0% encontró este documento útil (0 votos)
2K vistas176 páginas

Apuntes de Geometría Euclidiana

Este documento presenta apuntes sobre geometría euclidiana para competiciones matemáticas. Incluye secciones sobre ángulos, semejanza y congruencia de triángulos, cuadriláteros cíclicos, puntos notables de triángulos como el circuncentro y el ortocentro, y configuraciones útiles como conjugados isogonales y la recta de Simson. El objetivo es proporcionar los fundamentos y herramientas geométricas necesarias para resolver problemas complejos.

Cargado por

nnnnn
Derechos de autor
© © All Rights Reserved
Nos tomamos en serio los derechos de los contenidos. Si sospechas que se trata de tu contenido, reclámalo aquí.
Formatos disponibles
Descarga como PDF, TXT o lee en línea desde Scribd
0% encontró este documento útil (0 votos)
2K vistas176 páginas

Apuntes de Geometría Euclidiana

Este documento presenta apuntes sobre geometría euclidiana para competiciones matemáticas. Incluye secciones sobre ángulos, semejanza y congruencia de triángulos, cuadriláteros cíclicos, puntos notables de triángulos como el circuncentro y el ortocentro, y configuraciones útiles como conjugados isogonales y la recta de Simson. El objetivo es proporcionar los fundamentos y herramientas geométricas necesarias para resolver problemas complejos.

Cargado por

nnnnn
Derechos de autor
© © All Rights Reserved
Nos tomamos en serio los derechos de los contenidos. Si sospechas que se trata de tu contenido, reclámalo aquí.
Formatos disponibles
Descarga como PDF, TXT o lee en línea desde Scribd

Apuntes de Geometría Euclidiana para

Competiciones Matemáticas
Jafet Baca

23 de agosto de 2022
ÍNDICE GENERAL

I Fundamentos de geometría euclidiana 7


1. Manipulación angular 9
1.1. Ángulos en paralelas . . . . . . . . . . . . . . . . . . . . . . . . . . . . . . . . . . . . 9
1.2. Ángulos en circunferencias . . . . . . . . . . . . . . . . . . . . . . . . . . . . . . . . . 10
1.3. Problemas resueltos . . . . . . . . . . . . . . . . . . . . . . . . . . . . . . . . . . . . . 13
1.4. Problemas propuestos . . . . . . . . . . . . . . . . . . . . . . . . . . . . . . . . . . . . 14

2. Semejanza y congruencia de triángulos 17


2.1. Semejanza de triángulos . . . . . . . . . . . . . . . . . . . . . . . . . . . . . . . . . . 17
2.2. Congruencia de triángulos . . . . . . . . . . . . . . . . . . . . . . . . . . . . . . . . . 18
2.3. Teorema de Tales . . . . . . . . . . . . . . . . . . . . . . . . . . . . . . . . . . . . . . 19
2.3.1. La idea del paralelogramo . . . . . . . . . . . . . . . . . . . . . . . . . . . . . 20
2.4. Problemas resueltos . . . . . . . . . . . . . . . . . . . . . . . . . . . . . . . . . . . . . 21
2.5. Problemas propuestos . . . . . . . . . . . . . . . . . . . . . . . . . . . . . . . . . . . . 23

3. Potencia y cuadriláteros cíclicos 25


3.1. Potencia de puntos . . . . . . . . . . . . . . . . . . . . . . . . . . . . . . . . . . . . . 25
3.2. Cuadriláteros cíclicos . . . . . . . . . . . . . . . . . . . . . . . . . . . . . . . . . . . . 27
3.2.1. Propiedades fundamentales . . . . . . . . . . . . . . . . . . . . . . . . . . . . . 27
3.3. Problemas resueltos . . . . . . . . . . . . . . . . . . . . . . . . . . . . . . . . . . . . . 30
3.4. Problemas propuestos . . . . . . . . . . . . . . . . . . . . . . . . . . . . . . . . . . . . 32

II Rectas y puntos notables del triángulo 33


4. El circuncentro y el baricentro 35
4.1. Las mediatrices y el circuncentro . . . . . . . . . . . . . . . . . . . . . . . . . . . . . . 35
4.2. Las medianas y el baricentro . . . . . . . . . . . . . . . . . . . . . . . . . . . . . . . . 36
4.3. Problemas resueltos . . . . . . . . . . . . . . . . . . . . . . . . . . . . . . . . . . . . . 37
4.4. Problemas propuestos . . . . . . . . . . . . . . . . . . . . . . . . . . . . . . . . . . . . 39

5. Las alturas y el ortocentro 41


5.1. Hechos fundamentales . . . . . . . . . . . . . . . . . . . . . . . . . . . . . . . . . . . 41
5.1.1. Lemas relacionados . . . . . . . . . . . . . . . . . . . . . . . . . . . . . . . . . 44
5.2. Un círculo especial: El círculo de los nueve puntos . . . . . . . . . . . . . . . . . . . . 46
5.3. Problemas resueltos . . . . . . . . . . . . . . . . . . . . . . . . . . . . . . . . . . . . . 47
5.4. Problemas propuestos . . . . . . . . . . . . . . . . . . . . . . . . . . . . . . . . . . . . 50
6. Bisectrices, incentro y excentros 51
6.1. Preliminares . . . . . . . . . . . . . . . . . . . . . . . . . . . . . . . . . . . . . . . . . 51
6.2. Bisectrices externas y excentros . . . . . . . . . . . . . . . . . . . . . . . . . . . . . . 53
6.3. Los puntos de Gergonne y Nagel . . . . . . . . . . . . . . . . . . . . . . . . . . . . . . 55
6.3.1. Dos lemas adicionales . . . . . . . . . . . . . . . . . . . . . . . . . . . . . . . 57
6.4. Problemas resueltos . . . . . . . . . . . . . . . . . . . . . . . . . . . . . . . . . . . . . 58
6.5. Problemas propuestos . . . . . . . . . . . . . . . . . . . . . . . . . . . . . . . . . . . . 61

III Configuraciones útiles 63


7. Conjugados isogonales y simedianas 65
7.1. Conjugados isogonales . . . . . . . . . . . . . . . . . . . . . . . . . . . . . . . . . . . 65
7.2. Simedianas . . . . . . . . . . . . . . . . . . . . . . . . . . . . . . . . . . . . . . . . . 69
7.3. Problemas resueltos . . . . . . . . . . . . . . . . . . . . . . . . . . . . . . . . . . . . . 72
7.4. Problemas propuestos . . . . . . . . . . . . . . . . . . . . . . . . . . . . . . . . . . . . 74

8. Semejanza espiral 77
8.1. ¿Qué es la semejanza espiral? . . . . . . . . . . . . . . . . . . . . . . . . . . . . . . . . 77
8.2. Propiedades . . . . . . . . . . . . . . . . . . . . . . . . . . . . . . . . . . . . . . . . . 77
8.3. Un centro de semejanza espiral especial . . . . . . . . . . . . . . . . . . . . . . . . . . 79
8.4. Problemas resueltos . . . . . . . . . . . . . . . . . . . . . . . . . . . . . . . . . . . . . 79
8.5. Problemas propuestos . . . . . . . . . . . . . . . . . . . . . . . . . . . . . . . . . . . . 81

9. Recta de Simson 83
9.1. Configuración principal y hechos relacionados . . . . . . . . . . . . . . . . . . . . . . . 83
9.2. Problemas resueltos . . . . . . . . . . . . . . . . . . . . . . . . . . . . . . . . . . . . . 85
9.3. Problemas propuestos . . . . . . . . . . . . . . . . . . . . . . . . . . . . . . . . . . . . 87

10. Incírculos mixtilíneos 89


10.1. Preliminares . . . . . . . . . . . . . . . . . . . . . . . . . . . . . . . . . . . . . . . . . 89
10.2. Propiedades útiles . . . . . . . . . . . . . . . . . . . . . . . . . . . . . . . . . . . . . . 90
10.3. Problemas resueltos . . . . . . . . . . . . . . . . . . . . . . . . . . . . . . . . . . . . . 92
10.4. Problemas propuestos . . . . . . . . . . . . . . . . . . . . . . . . . . . . . . . . . . . . 95

IV Concurrencia y colinealidad 97
11. Teoremas de Ceva y Menelao 99
11.1. Colinealidad: Teorema de Menelao . . . . . . . . . . . . . . . . . . . . . . . . . . . . . 102
11.2. Problemas resueltos . . . . . . . . . . . . . . . . . . . . . . . . . . . . . . . . . . . . . 103
11.3. Problemas propuestos . . . . . . . . . . . . . . . . . . . . . . . . . . . . . . . . . . . . 107

12. Teorema del eje radical 109


12.1. Definición y propiedades . . . . . . . . . . . . . . . . . . . . . . . . . . . . . . . . . . 109
12.2. Problemas resueltos . . . . . . . . . . . . . . . . . . . . . . . . . . . . . . . . . . . . . 111
12.3. Problemas propuestos . . . . . . . . . . . . . . . . . . . . . . . . . . . . . . . . . . . . 113
13. Homotecia 115
13.1. Introducción . . . . . . . . . . . . . . . . . . . . . . . . . . . . . . . . . . . . . . . . . 115
13.2. Homotecia en polígonos . . . . . . . . . . . . . . . . . . . . . . . . . . . . . . . . . . 115
13.2.1. Propiedades . . . . . . . . . . . . . . . . . . . . . . . . . . . . . . . . . . . . . 116
13.3. Homotecia en circunferencias . . . . . . . . . . . . . . . . . . . . . . . . . . . . . . . . 116
13.3.1. Definición y construcción . . . . . . . . . . . . . . . . . . . . . . . . . . . . . 116
13.3.2. Puntos homólogos y antihomólogos . . . . . . . . . . . . . . . . . . . . . . . . 117
13.4. Algunos resultados útiles . . . . . . . . . . . . . . . . . . . . . . . . . . . . . . . . . . 118
13.5. Problemas resueltos . . . . . . . . . . . . . . . . . . . . . . . . . . . . . . . . . . . . . 119
13.6. Problemas propuestos . . . . . . . . . . . . . . . . . . . . . . . . . . . . . . . . . . . . 122

14. Teoremas selectos 125


14.1. Una breve introducción a la razón cruzada . . . . . . . . . . . . . . . . . . . . . . . . . 125
14.2. Teoremas selectos de concurrencia y colinealidad . . . . . . . . . . . . . . . . . . . . . 127
14.2.1. Teoremas de Pascal y Brianchon . . . . . . . . . . . . . . . . . . . . . . . . . . 127
14.2.2. Teorema de Pappus . . . . . . . . . . . . . . . . . . . . . . . . . . . . . . . . . 129
14.2.3. Teorema de Desargues . . . . . . . . . . . . . . . . . . . . . . . . . . . . . . . 130
14.3. Problemas resueltos . . . . . . . . . . . . . . . . . . . . . . . . . . . . . . . . . . . . . 132
14.4. Problemas propuestos . . . . . . . . . . . . . . . . . . . . . . . . . . . . . . . . . . . . 135

V Elementos de geometría proyectiva 137


15. División armónica 139
15.1. ¿Qué es la división armónica? . . . . . . . . . . . . . . . . . . . . . . . . . . . . . . . 139
15.1.1. Cuadriláteros armónicos . . . . . . . . . . . . . . . . . . . . . . . . . . . . . . 141
15.1.2. Círculos de Apolonio . . . . . . . . . . . . . . . . . . . . . . . . . . . . . . . . 143
15.2. Ejemplos . . . . . . . . . . . . . . . . . . . . . . . . . . . . . . . . . . . . . . . . . . 144
15.3. Problemas propuestos . . . . . . . . . . . . . . . . . . . . . . . . . . . . . . . . . . . . 147

16. Polos y polares 149


16.1. Introducción . . . . . . . . . . . . . . . . . . . . . . . . . . . . . . . . . . . . . . . . . 149
16.2. Resultados útiles . . . . . . . . . . . . . . . . . . . . . . . . . . . . . . . . . . . . . . 149
16.3. Polares y círculos ortogonales . . . . . . . . . . . . . . . . . . . . . . . . . . . . . . . 153
16.4. Más ejemplos . . . . . . . . . . . . . . . . . . . . . . . . . . . . . . . . . . . . . . . . 155
16.5. Problemas propuestos . . . . . . . . . . . . . . . . . . . . . . . . . . . . . . . . . . . . 157

17. Inversión 159


17.1. Primeros hechos . . . . . . . . . . . . . . . . . . . . . . . . . . . . . . . . . . . . . . . 159
17.2. Rectas y círculos . . . . . . . . . . . . . . . . . . . . . . . . . . . . . . . . . . . . . . 160
17.3. Un primer ejemplo . . . . . . . . . . . . . . . . . . . . . . . . . . . . . . . . . . . . . 162
17.4. Longitudes y ángulos . . . . . . . . . . . . . . . . . . . . . . . . . . . . . . . . . . . . 164
17.5. Inversión √
negativa . . . . . . . . . . . . . . . . . . . . . . . . . . . . . . . . . . . . . . 166
17.6. Inversión bc . . . . . . . . . . . . . . . . . . . . . . . . . . . . . . . . . . . . . . . . 168
17.7. Más ejemplos . . . . . . . . . . . . . . . . . . . . . . . . . . . . . . . . . . . . . . . . 171
17.8. ¿Cuándo es recomendable invertir? . . . . . . . . . . . . . . . . . . . . . . . . . . . . . 174
6

17.9. Problemas resueltos . . . . . . . . . . . . . . . . . . . . . . . . . . . . . . . . . . . . . 174


PARTE I

FUNDAMENTOS DE GEOMETRÍA
EUCLIDIANA
CAPÍTULO 1

MANIPULACIÓN ANGULAR

La primera técnica que todo concursante potencial debe desarrollar en geometría euclidiana es la ma-
nipulación de ángulos (angle-chasing). Pilar de otros tópicos fundamentales, como semejanza de trián-
gulos, encontrar ángulos adecuados en función de otros nos permite probar si un triángulo es isósceles o
si cuatro puntos yacen sobre un mismo círculo. Por esta razón, el objetivo de este capítulo es brindar las
técnicas más útiles y conocidas de manipulación angular.

1.1 Ángulos en paralelas


Consideremos dos rectas paralelas ℓ1 y ℓ2 como muestra la figura 1.1. Supongamos que otra recta
(comúnmente llamada transversal) corta a ℓ1 y ℓ2 . Los ángulos que se forman por estas rectas cumplen
las siguientes características.

Los ángulos opuestos por el vértice ∠PQC y ∠FQD son iguales; es decir, ∠PQC = ∠FQD.
Los ángulos alternos internos ∠PQC y ∠QPB tienen la misma medida; esto es, ∠PQC = ∠FQD.
∠EPA = ∠PQC, y son llamados ángulos correspondientes.
∠FQD = ∠EPA, y son ángulos alternos externos.

ℓ1 A P B

ℓ2 C Q D
F

Figura 1.1. Relaciones angulares en rectas paralelas.

Por supuesto, hay otras parejas de ángulos opuestos por el vértice, alternos internos, correspondientes
o alternos externos en esta configuración. ¿Puedes identificar al menos una pareja de cada tipo?
Observa que los ángulos ∠EPA y ∠EPB son suplementarios (tenemos que ∠EPA + ∠EPB = 180◦ ).
Esto nos ayuda a demostrar que la suma de los ángulos internos de un triángulo es 180◦ .

Teorema 1.1. Los ángulos internos de un triángulo suman 180◦ .


10 Manipulación angular

P A Q
α γ
β

α γ
B C

Demostración. Tracemos una paralela por A al lado BC (llamemósla ℓ). Sea P un punto sobre ℓ en distinto
semiplano con respecto al lado AB y C, y sea Q otro punto sobre ℓ en distinto semiplano con respecto a
AC y B. Sabemos que ∠PAB = ∠ABC y ∠QAC = ∠ACB al ser dos parejas de ángulos alternos internos.
Entonces:
∠ABC + ∠BCA + ∠CAB = ∠PAB + ∠QAC + ∠BAC = 180◦

como necesitábamos.

Para resolver problemas, es muy común utilizar letras griegas minúsculas para nombrar ángulos y
manipularlos (sumarlos, restarlos, etc.). Por lo general, las letras mayúsculas denotan circunferencias.

Cuadro 1.1. Correspondencia entre el alfabeto griego y el español.

Escritura Escritura
Nombre Español Nombre Español
Minúsc. Mayúsc. Minúsc. Mayúsc.
Alpha α A A Ni ν N N
Beta β B B Xi ξ Ξ X
Gamma γ Γ G Ómicron o O O
Delta δ ∆ D Pi π Π P
Épsilon ε E E Rho ρ R R
Dseta ζ Z Z Sigma σ Σ S
Eta η H E larga Tau τ T T
Theta θ Θ Th Ípsilon υ Y Y
Iota ι I I Fi φ Φ F
Kappa κ K K Ji χ X J
Lambda λ Λ L Psi ψ Ψ Ps
MI µ M M Omega ω Ω O larga

1.2 Ángulos en circunferencias

Definición 1.1. Un ángulo es central si su vértice coincide con el centro de un círculo. Su valor es
igual a la longitud del arco subtendido por sus rayos. Es decir, en la figura 1.2a ocurre que α = AB.
c
1.2 Ángulos en circunferencias 11

D
C A
A B α
α B
A
α
O O 2α
α
B C

(a) ∠AOB es central. (b) ∠ACB es inscrito. (c) ∠DBA es semiinscrito.


Figura 1.2. Ángulos en circunferencias.

Definición 1.2. Un ángulo es inscrito si su vértice yace sobre una circunferencia. Su medida es
equivalente a la mitad del arco que lo inscribe.

En la figura 1.2b sucede que ∠ACB = AB 2 . Por la definición 1 sabemos que ∠AOB = AB, así que
c c
también ocurre que la medida de todo ángulo inscrito es la mitad del ángulo central inscrito en su mismo
arco. La demostración de este hecho es como sigue:

Demostración. Como los triángulos COA y COB son isósceles en O, si tomamos en cuenta que el valor
de un ángulo central equivale a la longitud del arco que abarca, fácilmente obtenemos que:
  c c
∠COA ◦ ◦ ∠COB CB − CA AB
c
∠ACB = ∠OCA − ∠OCB = 90 − − 90 − = =
2 2 2 2

como requeríamos.

Definición 1.3. Un ángulo es semiinscrito si uno de sus rayos es tangente al círculo de referencia,
mientras que el otro es una secante. Su valor también es igual a la mitad del arco que interseca.

La gráfica 1.2c muestra un ejemplo de ángulo semiinscrito, el ángulo DBA. Su medida está dada por
∠DBA = AB/2.
c La lógica de este resultado es así: consideremos una secante BC3 , como muestra el gráfi-
co a continuación. Sabemos que ∠BC3 A = AB/2 c por ser un ángulo inscrito. A medida que movemos C3
de modo que se aproxima cada vez más a B, esta igualdad continúa siendo válida, como puede verificarse
para los ángulos ∠BC2 A y ∠BC1 A. Finalmente, cuando C3 coincide con B, la secante se “convierte” en
una tangente y el ángulo entre DB y AB continúa siendo igual a AB/2.
c Dicho de otra forma, el ángulo
∠BBA “inscribe” al arco AB, de donde surge la igualdad antes mencionada.
c

Además de estas tres categorías existen dos clasificaciones adicionales de ángulos en circunferencias
que son importantes pero no tan frecuentes en problemas de olimpiada como las recién abordadas. Estos
son los ángulos internos y externos.
12 Manipulación angular

D
A
α B
α
C1
α
α α C2
C C3

Figura 1.3. La medida de un ángulo semiinscrito equivale a la mitad del arco que subtiende.

Definición 1.4. Un ángulo es interno si su vértice yace en el interior de la circunferencia. Su valor


está dado por la semisuma de dos arcos formados por los puntos de intersección de los rayos del
ángulo con la circunferencia.

B
A
A P
α

P D C

C α

B D
(a) ∠BPD es interno. (b) ∠APC es externo.
Figura 1.4. Dos clases adicionales de ángulos en un círculo.

Definición 1.5. Un ángulo es externo si su vértice yace en el exterior de la circunferencia en cues-


tión. El valor del ángulo exterior corresponde a la mitad de la diferencia entre el arco mayor y el
arco menor determinado por los puntos de intersección de los rayos del ángulo con la circunferen-
cia.

En el panel izquierdo de la figura 1.4, el ángulo BPD es interno, con ∠BPD = (AC
c + BD)/2.
c Entre-
tanto, el panel derecho muestra al ángulo externo APC, cuya medida es igual a (BD − AC)/2.
c c

Demostración. Primero obtengamos la fórmula previa para el ángulo interno ∠BPD. Por la suma de
los ángulos internos de △CPB, es sencillo conseguir que ∠BPD = ∠PCB + ∠PBC. Esto implica que
∠BPD = BD/2c + AC/2c ya que ∠PCB y ∠PBC inscriben a los arcos BD c y AC,
c respectivamente.
Para el ángulo externo ∠APC, notemos que en el triángulo PAD ocurre que ∠APC = ∠BAD − ∠ADP
c − AC/2.
(¿por qué?). Desde luego, esto conlleva a que ∠APC = BD/2 c
1.3 Problemas resueltos 13

1.3 Problemas resueltos

Ejemplo 1.1. Dos circunferencias de centros O1 y O2 se cortan en dos puntos diferentes A y B. La


línea CD es tangente a ambas circunferencias. Demostrar que:
∠O1 AO2
∠CAD =
2

A
O1
O2

B
C D
Figura 1.5. Un primer ejemplo de manipulación angular.

Solución. Al ser △CO1 A isósceles en O1 , sabemos que ∠O1 AC = 90◦ −∠CO1 A/2 (1). Por un argumento
similar, ∠DAO2 = 90◦ − ∠DO2 A/2 (2).
Por otro lado, ∠ACD y ∠ADC son ángulos semiinscritos de las circunferencias con centros O1 y O2 ,
respectivamente, así que ∠ACD = ∠CO1 A/2 y ∠ADC = ∠DO2 A/2. Entonces, por la suma de los ángulos
internos de △CAD podemos obtener que

∠CO1 A + ∠DO2 A
∠CAD = 180◦ − ∠ACD − ∠ADC = 180◦ − (3)
2
tomando en cuenta las igualdades (1), (2) y (3) podemos inferir que ∠O1 AO2 = 360◦ − (∠CO1 A +
∠DO2 A). Comparando esta última expresión con (3) concluimos que ∠O1 AO2 = 2∠CAD, como de-
seábamos probar.

Ejemplo 1.2. Las circunferencias C1 y C2 se cortan en los puntos A y B. Se traza una recta ℓ que
corta a C1 en C y D, y a C2 en M y N, de tal manera que A y B quedan en distintos lados de ℓ.
Probar que ∠CAN + ∠MBD = 180◦ .

Solución. Notemos que en la circunferencia C1 los ángulos CAB y CDB son iguales, pues ambos sub-
tienden al arco CB. Análogamente, al enfocarnos en la circunferencia C2 los ángulos BMN y BAN son
congruentes. Luego:

∠CAN = ∠CAB + ∠BAN = ∠CDB + ∠BMN = ∠BDM + ∠BMD = 180◦ − ∠DBM

donde la última igualdad es verdadera por la suma de los ángulos internos de △BMD. La solución está
completa.
14 Manipulación angular

C M D N

Figura 1.6. Ángulos suplementarios en circunferencias.

Ejemplo 1.3. (Olimpiada de Mayo 2022, Nivel 2, P3). Sean ABCD un cuadrado, E un punto
del lado CD y F un punto en el interior del cuadrado tal que el triángulo BFE es isósceles y
∠BFE = 90◦ . Si DF = DE, calcular la medida del ángulo ∠FDE.

A B

45◦
D E C
Figura 1.7. Problema 3 de la Olimpiada de Mayo 2022, Nivel 2.

Solución. Sabemos que ∠BDC = 45◦ = ∠BFE/2. Junto al hecho BF = EF, lo anterior nos permite de-
ducir que F es el circuncentro de △BDE. Esto implica que EF = DF = DE, así que △DEF es equilátero
y ∠FDE = 60◦ .

1.4 Problemas propuestos


Problema 1.1. Sea Ω una circunferencia cualquiera con centro O. La recta ℓ es tangente a Ω en P.
Demostrar que OP es perpendicular a ℓ.
Problema 1.2. Demostrar que dos líneas paralelas cualesquiera que cortan a una circunferencia forman
arcos iguales entre ellas; es decir, los cuatro puntos de intersección determinan un trapecio isósceles.
Problema 1.3. Sea P un punto en el exterior de la circunferencia Γ. Las tangentes desde P a Γ tocan a la
circunferencia en A y B. Demostrar que PA = PB.
Problema 1.4. El lado BC del triángulo ABC coincide con un diámetro del circuncírculo de ABC. De-
mostrar que △ABC es rectángulo.
1.4 Problemas propuestos 15

Problema 1.5. Dos circunferencias son tangentes externamente en un punto A. La recta BC es una tan-
gente común externa, de modo que B está sobre la primera circunferencia y C en la segunda. Probar que
∠BAC = 90◦ .

Problema 1.6. (Olimpiada de Mayo 2014, Nivel 1, P4). Sea ABC un triángulo rectángulo e isósceles, con
∠C = 90◦ . Sean M el punto medio de AB y N el punto medio de AC. Sea P tal que MNP es un triángulo
equilátero con P en el interior del cuadrilátero MBCN. Calcular la medida del ángulo ∠CAP.

Problema 1.7. (Camino Báltico 2021, P13). Un punto P yace dentro del triángulo ABC. Los puntos K y
L son las proyecciones de P sobre AB y AC, respectivamente. El punto M está sobre la recta BC tal que
KM = LM, y el punto P′ es el simétrico de P con respecto a M. Probar que ∠BAP = ∠P′ AC.
16 Manipulación angular
CAPÍTULO 2

SEMEJANZA Y CONGRUENCIA DE TRIÁNGULOS

2.1 Semejanza de triángulos


En términos generales, dos triángulos son semejantes si poseen la “misma forma”. Esto quiere de-
cir que poseen tres pares de ángulos iguales y las razones de sus lados correspondientes son iguales.
En el caso de un par de triángulos semejantes △ABC y △DEF, lo anterior equivale a que ∠BAC =
∠EDF, ∠ABC = ∠DEF, ∠BCA = ∠EFD, y

AB BC CA
= =
DE EF FD

La relación de semejanza se denota △ABC ∼ △DEF. Por buen orden, es recomendable que los vérti-
ces correspondientes se coloquen en la misma posición. Por ejemplo, A y D son vértices correspondientes
y ocupan el primer puesto inicial al establecer la semejanza.

A
γ
D
γ

α β α β
B C E F

Figura 2.1. Dos triángulos semejantes △ABC y △DEF.

Pese a su relativa sencillez, la semejanza de triángulos constituye una herramienta crucial para re-
solver problemas de geometría euclidiana. Además, muchas de las propiedades y resultados a estudiar
en los capítulos posteriores pueden ser demostradas mediante triángulos semejantes. Ahí la importancia
de identificar cuándo un par (o más) de triángulos cuentan con dicha característica. Siendo así, veamos
cuáles son los tres criterios principales de semejanza.

Proposición 2.1 (Criterio Ángulo-Ángulo, AA). Si los triángulos ABC y DEF poseen dos pares
de ángulos correspondientes iguales (por ejemplo, ∠ABC = ∠DEF y ∠BCA = ∠EFD), entonces
son semejantes.
18 Semejanza y congruencia de triángulos

A
F
β
D
α α
B
β E
C

Figura 2.2. Criterio de semejanza AA.

Proposición 2.2 (Criterio Lado-Ángulo-Lado, LAL). Supongamos que △ABC y △DEF poseen
un par de ángulos iguales (digamos ∠BAC = ∠EDF), y que los lados adyacentes a estos ángulos
satisfacen la relación:
AB AC
=
DE FD
entonces, △ABC ∼ △DEF.

A D
E α
α

F
B C
Figura 2.3. Criterio de semejanza LAL.

Proposición 2.3 (Criterio Lado-Lado-Lado, LLL). Si los lados de los triángulos ABC y DEF
satisfacen que:
AB BC CA
= =
DE EF FD
debe ocurrir que △ABC ∼ △DEF.

2.2 Congruencia de triángulos

La congruencia de triángulos es un caso particular de semejanza. En esta, los triángulos en cuestión


no solo tienen la misma forma, sino el mismo tamaño. Por consiguiente, sus criterios de coinciden con
los criterios de semejanza pero incluyen al menos un elemento adicional que indique tal congruencia.
2.3 Teorema de Tales 19

Proposición 2.4 (Criterio Ángulo-Lado-Ángulo, ALA). Sean ABC y DEF dos triángulos tales
que poseen dos pares de ángulos iguales, ∠ABC = ∠DFE y ∠BCA = ∠DEF, y además un par de
sus lados correspondientes tienen la misma longitud. Entonces, △ABC ∼
= △DFE.

D
A

α β β α
B C E F
Figura 2.4. Criterio de congruencia ALA.

Proposición 2.5 (Criterio Lado-Ángulo-Lado, LAL). Los triángulos ABC y DEF poseen dos pares
de lados congruentes, digamos AB = DF y CA = DE. Además, los ángulos delimitados por estos
lados son iguales, ∠BAC = ∠EDF. Luego, △ABC y △DFE son congruentes.

A
α D
α

B C
F

Figura 2.5. Criterio de congruencia LAL.

Proposición 2.6 (Criterio Lado-Lado-Lado, LLL). Si los tres pares de lados de △ABC y △DEF
tienen la misma medida; es decir, AB = DE, BC = EF y CA = DF, entonces △ABC ∼ = △DEF.

B
E F

A C
D

2.3 Teorema de Tales


El primer ejemplo en que aplicaremos semejanza de triángulos es el conocido teorema de Tales.
20 Semejanza y congruencia de triángulos

Teorema 2.1 (Tales). Sean AB y CD dos segmentos. Las rectas CA y DB se intersecan en el punto
P. Entonces AB ∥ CD si y solo si:
PA PB
=
PC PD

A B

C D
Figura 2.6. Teorema de Tales.

Demostración. Si AB ∥ CD sucede que ∠PBA y ∠PDC son correspondientes, así que deben ser igua-
les. Similarmente podemos argumentar que ∠PAB = ∠PCD. Luego, por el criterio AA inferimos que
△PAB ∼ △PCD, así que PA/PC = PB/PD. La segunda dirección es completamente recíproca.

Una aplicación bastante particular (pero importante) del teorema de Tales ocurre cuando A y B coin-
ciden con los puntos medios de PC y PD. Por supuesto, el hecho PA/PC = PB/PD = 2 implica que
AB ∥ CD. Es más, la semejanza entre △PAB y △PCD implica que AB = CD/2. Por esto, AB recibe el
nombre de base media de CD. Evidentemente, también podemos construir las bases medias de PD y CD.
Es importante reconocer algunas “variantes” del teorema de Tales. Primero, si tomamos el recí-
proco de la expresión anterior, sustraemos 1 de ambos lados y extraemos el recíproco de la nueva
igualdad podemos obtener que PA/AC = PB/CD (algunas veces el teorema es enunciado de esta for-
ma). Por otro lado, la semejanza que empleamos en la demostración previa también proporciona que
PA/PC = AB/CD = PB/PD. Además, el teorema continúa siendo válido cuando P se encuentra en la
región delimitada por las rectas AB y CD, como se muestra a continuación.

A B

C D
Figura 2.7. Una variante común del teorema de Tales.

2.3.1 La idea del paralelogramo


Sea ABCD un paralelogramo. Recordemos la siguiente propiedad:

Proposición 2.7. Las diagonales AC y BD de ABCD se cortan en su punto medio; es decir, se


bisecan entre ellas.
2.4 Problemas resueltos 21

Como es de esperar, la prueba no involucra nada más que el teorema de Tales: si M es el corte de AC
y BD, sabemos que AM/MC = BM/MD = AB/DC = 1.
Pese a su simplicidad, esta propiedad es extremadamente útil. En una olimpiada, por supuesto, el
enunciado del problema no brindará explícitamente un paralelogramo: ¡debes construirlo! Lo que sí su-
cede es que proporcionan puntos medios y construcciones adicionales que los incluyen. En estas si-
tuaciones, considera simétricos con respecto a tales puntos medios; así, por el hecho 2.7, construyes
paralelogramos que pueden ser de amplia utilidad.

A B

D C

Figura 2.8. La idea del paralelogramo: terminar de construir la figura.

Enfatizaremos esta idea cuando resolvamos problemas mediante ella en los capítulos próximos.

2.4 Problemas resueltos

Ejemplo 2.1. Sean ABC y PQR dos triángulos semejantes. Demostrar que la razón de sus áreas es
igual al cuadrado de su razón de semejanza, es decir:

[ABC]
= r2
[PQR]

A
P

α α
B D C Q S R

Figura 2.9. La razón entre áreas entre triángulos semejantes es el cuadrado de la razón de semejanza.

Solución. Sean AD y PS las alturas trazadas desde A y P en △ABC y △PQR, respectivamente. Co-
mo △ABC ∼ △PQR, sabemos que ∠ACD = ∠ACB = ∠PRQ = ∠PRS. Además, tenemos que ∠ADC =
∠PSR = 90◦ . Siendo así, por el criterio AA concluimos que △ADC ∼ △PSR, lo que implica que AD/PS =
AC/PR. Entonces
[ABC] AD · BC/2 AD · BC AC BC BC2
= = = · = = r2
[PQR] PS · QR/2 PS · QR PR QR QR2
como se requiere.
22 Semejanza y congruencia de triángulos

Ejemplo 2.2. Sea ABCD un trapecio con AB ∥ CD. El punto P es la intersección de sus diagonales,
mientras que Q y Q′ son puntos sobre BC y AD, respectivamente, tales que QQ′ pasa por P y
QQ′ ∥ AB. Probar que P es punto medio de QQ′ .

A B

Q′ P
Q

D C
Figura 2.10. Una aplicación de semejanza y el teorema de Tales.

Solución. Como Q′ P ∥ DC, podemos cerciorarnos fácilmente que △AQ′ P ∼ △ADC, por tanto Q′ P =
AP BP
AC · CD (1). Análogamente, extraemos que PQ = BD · CD (2). Por el teorema de Tales sabemos que
AP/AC = BP/BD (3). Juntando (1), (2) y (3) concluimos que

AP BP
Q′ P = ·CD = ·CD = PQ
AC BD
como deseábamos.

Cuando la demostración de un hecho/resultado particular en la solución de un problema sea idéntico


a la prueba de otro, podemos emplear el vocablo “análogamente”. Esto se utiliza para ahorrar tiempo y
proporcionar soluciones más claras y elegantes.

Ejemplo 2.3. Las circunferencias Γ1 y Γ2 se cortan en dos puntos distintos P y Q. Los puntos A y
B yacen sobre Γ1 , y los puntos C y D yacen sobre Γ2 , de modo que las rectas AC y BD se cortan
en P. Demostrar que los triángulos QAB y QCD son semejantes.

C
B P

A D

Figura 2.11. Una semejanza “en espiral”.


2.5 Problemas propuestos 23

Solución. Observemos que ∠BQA = ∠BPA ya que ambos ángulos subtienden al arco AB
c en Γ1 . De forma
similar, ∠CPD = ∠CQD, pero ∠BPA y ∠CPD son opuestos por el vértice, entonces:

∠BQA = ∠BPA = ∠CPD = ∠CQD (1)

Por otro lado, ∠CPQ = 180◦ − ∠CDQ, ya que ambos subtienden los arcos CDQ
[ y CPQ
d cuya suma es

360 . Además, ∠ABQ = ∠APQ pues subtienden al arco AQ. Luego,

∠ABQ = ∠APQ = 180◦ − ∠CPQ = ∠CDQ (2)

Las igualdades (1) y (2) implican que, por el criterio AA, △AQB ∼ △CQD. La solución está completa.

2.5 Problemas propuestos


Problema 2.1. Sean F, G, H e I los puntos medios de los lados AB, BC,CD y DA del cuadrilátero ABCD,
respectivamente. Demostrar que el cuadrilátero FGHI es un paralelogramo.

Problema 2.2. En un triángulo △ABC, sobre el lado BC se toma un punto D de tal manera que ∠BAD =
∠ACB. Demostrar que AB2 = BD · BC.

Problema 2.3. En la siguiente figura los segmentos a, b, c y d son paralelos y dividen al lado BC en 4
segmentos iguales. Si a = 10, encontrar la suma a + b + c + d.

a
b
c
d
B C

Problema 2.4. Sea ABCD un paralelogramo en el que L y M son puntos medios de AB y CD, respectiva-
mente. Probar que los segmentos LC y AM dividen la diagonal BD en tres segmentos iguales.

Problema 2.5. En un paralelogramo ABCD se escogen los puntos E y F sobre la diagonal AC de manera
que AE = FC. Si BE se extiende hasta interceptar AD en H, y BF se extiende hasta interceptar DC en G,
probar que HG es paralelo a AC.

Problema 2.6. El segmento AM es la mediana hacia el lado BC de un triángulo ABC. Se toma un punto P
sobre AM. BP se extiende hasta interceptar AC en E, y CP se extiende hasta interceptar AB en D. Probar
que DE es paralelo a BC

Problema 2.7. En un cuadrilátero ABCD, sobre las rectas AC y BD se toman los puntos K y M de manera
que BK es paralelo a AD y AM es paralelo a BC. Demostrar que KM es paralelo a CD.
24 Semejanza y congruencia de triángulos

Problema 2.8. (Centroamericana 2002, P4). Sea ABC un triángulo, D el punto medio de BC, E un punto
sobre el segmento AC tal que BE = 2AD y F es el punto de intersección de AD con BE. Si ∠DAC = 60◦ ,
encontrar la medida del ángulo AEF.

Problema 2.9. (Olimpiada de Mayo 2021, Nivel 2, P3). Sean ABC un triángulo y D un punto en su
interior tal que ∠DBC = 60◦ y ∠DCB = ∠DAB = 30◦ . Si M y N son los puntos medios de AC y BC,
respectivamente, demostrar que ∠DMN = 90◦ .

Problema 2.10. (Olimpiada de Mayo 2019, Nivel 2, P3). En los lados AB, BC y CA de un triángulo
ABC se ubican los puntos P, Q y R, respectivamente, tales que BQ = 2QC, CR = 2RA y ∠PRQ = 90◦ .
Demostrar que ∠APR = ∠RPQ.

Problema 2.11. (Caucásica Junior 2022, P2). En el paralelogramo ABCD, los puntos E y F están sobre
los segmentos AD y CD de forma que ∠BCE = ∠BAF. Los puntos K y L yacen sobre los segmentos AD
y CD tales que AK = ED y CL = FD. Probar que ∠BKD = ∠BLD.
CAPÍTULO 3

POTENCIA Y CUADRILÁTEROS CÍCLICOS

3.1 Potencia de puntos


Comenzaremos con un resultado simple pero fundamental.

Proposición 3.1. Sean A, B, C y D puntos sobre una misma circunferencia Γ, en este orden. El
punto P es la intersección de las diagonales AC y BD. Luego,

AP · PC = BP · PD (3.1)

B X2 X3 B
X1
A A
P P

D O D O
Y3
C C
Y2

Y1

Figura 3.1. Potencia del punto interior P con respecto a Γ.

Demostración. Los ángulos ∠ADB y ∠ACB subtienden ambos al mismo arco AB,
c así que

∠ADP = ∠ADB = ∠ACB = ∠PCB

De forma similar, ∠DAP = ∠DBP. Por criterio AA, △APD ∼ △BPC, de donde la conclusión es inme-
diata.

Nótese que el resultado sigue siendo válido al trazar una cantidad arbitraria de cuerdas que pasen por
P (lado derecho de la figura 3.1), ya que podemos obtener pares de triángulos semejantes que conllevan
a la igualdad de los productos de los pares de segmentos en los que P divide a cada cuerda. Esto es,
X1 P · PY1 = X2 P · PY2 = X3 P · PY3 = AP · PC = BP · PD. Lo anterior permite intuir que la potencia de P,
como tal, es independiente de las cuerdas en consideración. La definición formal de potencia escenifica
esta sospecha.
26 Potencia y cuadriláteros cíclicos

P
B A
X
Y
C P
C
A B
O O
D

D X
Y

Figura 3.2. Potencia del punto P en dos configuraciones.

Definición 3.1. Consideremos una circunferencia Γ con centro O y radio r. Sea P ̸= O un punto
arbitrario. La potencia de P con respecto a Γ se define como

Pot(P, Γ) = PO2 − r2 (3.2)

Demostración. Cuando P yace en el exterior de Γ, el valor de la expresión (3.2) es positivo. En contraste,


cuando P pertenece al interior de Γ, dicho valor es negativo. En efecto, el signo de (3.2) permite identificar
si el punto yace o no dentro de Γ. Trabajemos con el primer caso (lado derecho del gráfico 3.2. Sean XY
el diámetro cuya prolongación contiene a P. Entonces

PA · PD = PX · PY = (PO − OX)(PO + OY ) = PO2 − r2

Queda como ejercicio al lector cerciorarse de que, cuando P yace en el interior de Γ, entonces Pot(P, Γ) =
r2 − OP2 .

Con la formulación de (3.2), queda claro que si P yace sobre Γ su potencia asociada es 0.
Podemos decir un poco más de la potencia de P cuando yace fuera de Γ.

Proposición 3.2. Cuando P es externo a Γ, la potencia de P respecto a este círculo equivale a la


longitud de las tangentes trazadas desde P hacia Γ.

Demostración. Sean Q y R los puntos donde las tangentes desde P trazadas hacia Γ tocan a esta cir-
cunferencia. Una aplicación inmediata del teorema de Pitágoras en △OQP u △ORP brinda el resultado
deseado.

Una implicación adicional de la propiedad 3.2 es:

Lema 3.1. Dos puntos son equidistantes de O si y solo si poseen igual potencia respecto a Γ.
3.2 Cuadriláteros cíclicos 27

P
Q

O
R

Figura 3.3. La potencia de Q es igual al cuadrado de la longitud de las tangentes trazadas hacia Γ.

3.2 Cuadriláteros cíclicos


Los cuadriláteros cíclicos constituyen una de las herramientas básicas que todo olímpico debe ma-
nejar. Casi todo problema de geometría euclidiana en olimpiadas involucra al menos un cuadrilátero
cíclico. No hace falta decir que esta clase de cuadriláteros se encuentran íntimamente relacionados con
la potencia de puntos, como visualizaremos en la exposición subsecuente.

Definición 3.2. Sea ABCD un cuadrilátero. Si los puntos A, B, C y D están sobre una misma
circunferencia, diremos que ABCD es un cuadrilátero cíclico.

B
C

Figura 3.4. El cuadrilátero ABCD es cíclico.

Alternativamente, podemos decir que ABCD está inscrito.


Es claro que a todo triángulo podemos trazarle su circunferencia circunscrita (es decir, su circun-
círculo); sin embargo, esto no necesariamente ocurre en el caso de los cuadriláteros. Por consiguiente,
identificaremos las condiciones a satisfacer para que un cuadrilátero posee un circuncírculo.

3.2.1 Propiedades fundamentales

Proposición 3.3. El cuadrilátero ABCD es cíclico si y solo si ∠ABD = ∠ACD.


28 Potencia y cuadriláteros cíclicos

C
B B
C E

D D

A A

Figura 3.5. Ángulos subtendidos en un mismo arco proporcionan cuadriláteros cíclicos.

Prueba. Inicialmente, supongamos que ABCD está inscrito en un círculo, luego:

AD
c
∠ABD = = ∠ACD
2

es decir, ∠ABD y ∠ACD deben ser iguales ya que subtienden al mismo arco AD
c en el círcuncírculo de
ABCD.
Pasemos a la otra dirección. Asumamos que ∠ABD = ∠ACD. Debemos probar que A, B, C y D yacen
sobre una misma circunferencia. Dibujemos el circuncírculo de △ABD y supongamos que interseca por
segunda vez a la recta DC en E ̸= C. Como ABCD es cíclico, tenemos que:

∠ABD = ∠AED

de este modo, concluimos que ∠AED = ∠ACD, lo cual implica que AE ∥ AC. Es claro que esto es
una contradicción, pues AE y AC comparten un punto común A. Por consiguiente, E ≡ C y ABCD es
cíclico.

En realidad, tenemos tres pares más de ángulos iguales de este tipo en un cuadrilátero cíclico, a saber:

∠DBC = ∠DAC
∠BCA = ∠BDA
∠CDB = ∠CAB

B
C

Figura 3.6. Pares de ángulos iguales en un cuadrilátero cíclico.


3.2 Cuadriláteros cíclicos 29

En la figura 3.6, los ángulos iguales están rellenos con un mismo color.

Proposición 3.4. Un cuadrilátero ABCD es cíclico si y solo algún par de ángulos opuestos suman
180◦ .

Prueba. Primero, asumamos que el cuadrilátero ABCD es cíclico. Luego

CDA ABC 360◦


= 180◦
d d
∠ABC + ∠CDA = + =
2 2 2
Similarmente, es posible demostrar que también ocurre que ∠BCD + ∠DAB = 180◦ .

B
C

A
D

Figura 3.7. Ángulos opuestos suplementarios brindan cuadriláteros cíclicos.

Ahora, supongamos que ∠AB + ∠CDA = 180◦ . Tracemos la circunferencia circunscrita al triángulo
ABC y asumamos que no pasa por el vértice D. Sea E = DC ∩ (ABC). Como el cuadrilátero ABCE es
cíclico tenemos que
∠ABC + ∠CEA = 180◦
por tanto ∠CEA = ∠EDA y entonces DA es paralela EA, lo cual es una contradicción ya que dos rectas
paralelas no se intersecan. Entonces D coincide con E y por lo tanto el cuadrilátero ABCD es cíclico.
La potencia de un punto y los cuadriláteros cíclicos van de la mano ya que, en términos generales, uno
implica al otro. Si P se define como el punto de intersección de las diagonales de un cuadrilátero inscrito
ABCD, es claro que este punto cumple la igualdad 3.1. El paso inverso podemos probarlo mediante el
criterio de semejanza AA. Lo mismo ocurre si consideramos las intersecciones de los lados opuestos de
ABCD.
Antes de pasar a resolver algunos problemas a modo de ejemplo, introduciremos el siguiente resultado
fundamental.

Teorema 3.1 (Teorema de Ptolomeo). El producto de las diagonales del cuadrilátero cíclico
ABCD equivale a la suma de los productos de los lados opuestos. Es decir,

AC · BD = AD · BC + AB · DC

La demostración se presenta más adelante, en el capítulo de inversión. Un camino posible consiste


en considerar un punto E sobre la diagonal AC tal que ∠EBC = ∠ABD, identificar triángulos semejantes
y manipular longitudes hasta arribar al resultado. Los detalles se dejan como ejercicio al lector. Por el
momento, lo tomaremos como verdadero.
30 Potencia y cuadriláteros cíclicos

3.3 Problemas resueltos

Ejemplo 3.1. (Centroamericana 2014, P2). Sea ABCD un trapecio con bases AB y CD, inscrito en
un círculo con centro O. Sea P la intersección de las rectas BC y AD. Un círculo por O y P interseca
los segmentos BC y AD en puntos internos F y G, respectivamente. Probar que BF = DG.

Solución. Como PGOF es cíclico, obtenemos que ∠DGO = ∠OFB. Asimismo, notemos que

∠DPB = ∠DPC = 180◦ − 2∠BCD = 180◦ − ∠BOD

por tanto PBOD es cíclico, lo cual implica que ∠OBF = ∠ODG. Por criterio AA concluimos que
△GOD ∼ △FOB, pero sabemos que los lados correspondientes OD y OB tienen igual medida, de modo
que estos triángulos deben ser congruentes y, por ende, DG = BF.

A B

G
O
D C

Figura 3.8. Segundo problema de la OMCC 2014.

Ejemplo 3.2. (Panamericana Femenil 2021, P2). Considere el triángulo rectángulo isósceles ABC
con ∠BAC = 90◦ . Sea ℓ la recta que pasa por B y el punto medio del lado AC. Sea Γ la circunfe-
rencia con diámetro AB. La recta ℓ y la circunferencia Γ se intersecan en el punto P, diferente de
B. Muestre que la circunferencia que pasa por los puntos A, C y P es tangente a la recta BC en C.

Solución. Sea M el punto medio de AC y Q un punto tal que BCQA es un paralelogramo (recuerda
la idea del paralelogramo). Evidentemente, B, P, M y Q son colineales. Como QC ∥ AB rápidamente
conseguimos que:
∠QCA = ∠BAC = 90◦ = ∠APQ
lo que implica que APCQ es cíclico. Ya que ∠AQC = ∠CBA = ∠ABC = 45◦ , concluimos que BC es
tangente al circuncírculo de APCQ en C.
3.3 Problemas resueltos 31

A Q
45◦

M
P

45◦
B C

Figura 3.9. Problema 2 de la PAGMO 2021.

Ejemplo 3.3. (Centroamericana 2015, P3). Sea ABCD un cuadrilátero cíclico con AB < CD, y sea
P el punto de intersección de las rectas AD y BC. El circuncírculo del triángulo PCD corta a la
recta AB en los puntos Q y R. Sean S y T los puntos donde las tangentes desde P al circuncírculo
de ABCD tocan a dicha circunferencia.

Pruebe que PQ = PR.


Muestre que QRST es un cuadrilátero cíclico.

Solución. Como CDPR es cíclico, observemos que:


∠PRC = 180◦ − ∠PDC = 180◦ − ∠ADC = ∠ABC = ∠PBR
También pasa que ∠RPB = ∠CPB; luego, por criterio AA inferimos que △RBP ∼ CRP, de donde
obtenemos fácilmente que PR2 = PB · PC. Análogamente, podemos probar que △QAP ∼ △DQP y
PQ2 = PA · PD. Como PA · PD = PB · PC, concluimos que PQ = PR.

P
R

B T
A
Q
S

D C

Figura 3.10. Problema 3 de la OMCC 2015.


Para el segundo ítem, basta recordar que también PT 2 = PB · PC = PA · PD = PS2 por la proposición
3.2. En conclusión, Q, R, S y T yacen en la circunferencia centrada en P y con radio PQ = PR = PS =
PT .
32 Potencia y cuadriláteros cíclicos

3.4 Problemas propuestos


Problema 3.1 (Teorema de Miquel). En un triángulo ABC sean M, N y P, puntos sobre los lados BC, CA
y AB, respectivamente. Se trazan las circunferencias circunscritas a los triángulos APN, BMP y CNM.
Demuestra que las tres circunferencias tienen un punto en común.
Problema 3.2 (Teorema de Brahmagupta). Sea ABCD un cuadrilátero cíclico y P el punto de intersección
de AC y BD. Suponga que AC es perpendicular a BD. Demuestre que la perpendicular trazada desde P
hacia el lado BC pasa por el punto medio de AD.
Problema 3.3. (Cono Sur 2018, P1). Sea ABCD un cuadrilátero convexo, donde R y S son puntos en DC
y AB, respectivamente, tales que AD = RC y BC = SA. Sean P, Q y M los puntos medios de RD, BS y
CA, respectivamente. Si ∠MPC + ∠MQA = 90◦ , probar que ABCD es cíclico.
Problema 3.4. (Centroamericana 2015, P5). Sea ABC un triángulo acutángulo y sea Γ su circuncírculo.
La bisectriz del ángulo A interseca a BC en D, a Γ en K (distinto
√ de A), y a la tangente a Γ por B en X.
AD
Demostrar que K es el punto medio de AX si y solo si DC = 2.
Problema 3.5. (Rioplatense 2018, Nivel 3, P2). Sea P un punto exterior a la circunferencia Γ, y sea PA
una de las tangentes desde P a Γ. La recta ℓ pasa por P e interseca a Γ en B y C, con B entre P y C. Sea D
el simétrico de B con respecto a P. Sean ω1 y ω2 los círculos circunscritos a los triángulos DAC y PAB,
respectivamente. El punto E ̸= A es la intersección de ω1 y ω2 . La recta EB interseca nuevamente a ω1
en F. Demostrar que CF = AB.
Problema 3.6. (Iraní de Geometría 2019, Nivel Avanzado, P1). Los círculos ω1 y ω2 se cortan en puntos
A y B. El punto C yace sobre la tangente desde A a ω1 tal que ∠ABC = 90◦ . Una recta arbitraria ℓ
pasa por C y corta a ω2 en puntos P y Q. Las rectas AP y AQ cortan a ω1 por segunda vez en X y Z,
respectivamente. Sea Y el pie de la altura desde A a ℓ. Probar que los puntos X,Y y Z son colineales.
Problema 3.7. (IMO 2014, P4). Los puntos P y Q están en el lado BC del triángulo acutángulo ABC de
modo que ∠PAB = ∠BCA y ∠CAQ = ∠ABC. Los puntos M y N están en las rectas AP y AQ, respectiva-
mente, de modo que P es el punto medio de AM, y Q es el punto medio de AN. Demostrar que las rectas
BM y CN se cortan en la circunferencia circunscrita del triángulo ABC.
Problema 3.8. (Hong Kong, Selectivo 2021, Prueba 2, P3). Sea ABC un triángulo acutángulo con cir-
cuncírculo Γ, y sea P el punto medio del arco menor BC de Γ. Las rectas AP y BC se cortan en D, y sea
M el punto medio de AB. También definamos a E como el punto tal que AE ⊥ AB y BE ⊥ MP. Probar
que AE = DE.
Problema 3.9. (Cuenca del Pacífico 2022, P2). Sea ABC un triángulo rectángulo con ∠B = 90◦ . Sea D
un punto de la recta BC tal que B está entre D y C. Sean E el punto medio de AD y F el segundo punto
de intersección de la circunferencia circunscrita del triángulo ACD son la circunferencia circunscrita del
triángulo BDE. Demostrar que al mover D, la recta EF pasa por un punto fijo.
Problema 3.10. (IMO 2019, P2). En el triángulo ABC, el punto A1 está en el lado BC y el punto B1
está en el lado AC. Sean P y Q puntos en los segmentos AA1 y BB1 , respectivamente, tales que PQ es
paralelo a AB. Sea P1 un punto de la recta PB1 distinto de B1 , con B1 entre P y P1 , y ∠PP1C = ∠BAC.
Análogamente, sea Q1 un punto en la recta QA1 , distinto de A1 , con A1 entre Q y Q1 , y ∠CQ1 Q = ∠CBA.
Demostrar que los puntos P, Q, P1 y Q1 son concíclicos.
PARTE II

RECTAS Y PUNTOS NOTABLES DEL


TRIÁNGULO
CAPÍTULO 4

EL CIRCUNCENTRO Y EL BARICENTRO

En la parte anterior desarrollamos técnicas preliminares que nos permitirán pasar a configuraciones
más elaboradas, como el caso de los puntos y rectas notables del triángulo.
Existe una amplia cantidad de puntos con propiedades interesantes relacionados al triángulo. Al ha-
blar de puntos notables en geometría olímpica, usualmente nos referimos al ortocentro, el baricentro, el
incentro y el circuncentro. Nos encargaremos de su definición y propiedades relacionadas, en detalle, a
lo largo de este y los próximos capítulos.

4.1 Las mediatrices y el circuncentro

Definición 4.1. La mediatriz de un segmento AB es el lugar geométrico de los puntos P equidis-


tantes a A y B.

A B

Figura 4.1. La mediatriz de AB.


Coincidentemente, ¡la mediatriz es una recta! Intenta cerciorarte de este resultado por tu propia cuen-
ta. Por definición, todo punto sobre una mediatriz nos brinda triángulos isósceles. Por ejemplo, en el
gráfico 4.1 tenemos que AP = PB y AQ = QB. Por supuesto, ℓ pasa por el punto medio de AB.
Ahora consideremos el caso de un triángulo. Ocurre que las mediatrices de sus tres lados se intersecan
en un mismo punto, el circuncentro de △ABC.

Definición 4.2. El circuncentro del triángulo ABC (usualmente denotado por O), es el punto donde
las mediatrices de los lados AB, BC y CA concurren.

En efecto, supongamos que las mediatrices de los lados AB y AC se cortan en O. Por definición,
tenemos que OB = OA = OC, por lo que O debe estar sobre la mediatriz del lado BC también. Esta
36 El circuncentro y el baricentro

igualdad implica la existencia de una circunferencia con centro en O, que pasa por los tres vértices del
triángulo. Esta circunferencia recibe el nombre de circuncírculo de △ABC. Nos referiremos al radio del
circuncírculo como circunradio. Naturalmente, OA, OB y OC son circunradios de △ABC.

B C

Figura 4.2. Existencia del circuncentro y el circuncírculo de ABC.


Cuando △ABC es acutángulo, O yace en su interior. Si el triángulo es obtusángulo, entonces O yace
en su exterior. Cuando ABC posee un ángulo recto, tenemos la siguiente propiedad.

Proposición 4.1. Sea ABC un triángulo rectángulo con ∠BAC = 90◦ . Entonces, su circuncentro
O coincide con el punto medio de su hipotenusa BC.

Dada su facilidad, la demostración se deja como ejercicio al lector.

4.2 Las medianas y el baricentro

Definición 4.3. El baricentro o centroide es el punto común de las rectas que unen los vértices
con el punto medio de sus respectivos lados opuestos en el triángulo ABC. Estas rectas son las
medianas de △ABC. El triángulo △LMN es el triángulo medial de △ABC.

Para probar la concurrencia, supongamos que L, M y N son los puntos medios de BC, CA y AB,
respectivamente. Definamos a G como el punto de intersección de BM y CN. Al ser MN base media
respecto a BC sabemos que MN ∥ BC, por lo que:
BG CG BC
= = =2
GM GN MN
Sea F el simétrico de G con respecto a L. Con esto, BGCF es un paralelogramo, lo que implica
CF = BG = 2MG, y así:
FC AC
= =2
GM AM
por consiguiente, A, G y F están sobre una misma línea, y por ende AG coincide con la A-mediana de
△ABC.
4.3 Problemas resueltos 37

N M

B L C

Figura 4.3. Las medianas de △ABC y su baricentro.

El teorema de Ceva, que abordaremos en un capítulo posterior, trivializa la prueba de la concurrencia


de las medianas. En la demostración anterior utilizamos (y verificamos) el siguiente hecho fundamental.

Proposición 4.2. El baricentro G divide a cada mediana en la razón 2 : 1.

Por ejemplo, AG
GL = 2, que implica la igualdad AL = 3GL; análogamente para las otras dos medianas.
Notemos algo importante. Como △BGL y △LGC comparten la misma altura desde G y sus bases BL
y CL son congruentes, obtenemos que [BGL] = [LGC]. De forma similar, [AGN] = [NGB] y [CGM] =
[MGA]. Pero veamos que las perpendiculares desde B y C a MN poseen igual longitud porque MN ∥ BC,
así que [MBN] = [NCM] y por ende [NGB] = [CMG]. Análogamente conseguimos que [AGN] = [LGC] y
[MGA] = [BGL]. En conclusión, hemos mostrado que

[BGL] = [NGB] = [AGN] = [MGA] = [CGM] = [LGC]

es decir:

Proposición 4.3. Las medianas de △ABC lo dividen en seis triángulos con áreas iguales y con el
baricentro como vértice común.

4.3 Problemas resueltos

Ejemplo 4.1. (Sharygin 2017, Ronda Final, Grado 8, P3). Sean AD, BE y CF las medianas del
triángulo ABC. Los puntos X e Y son las reflexiones de F con respecto a AD y BE, respectivamente.
Probar que los circuncírculos de los triángulos BEX y ADY son cocéntricos.

Solución. Como AD biseca a los segmentos EF y XF, por el teorema de Tales resulta que EX ∥ AD.
Además, sucede que XA = AF = ED por ser ED base media de AB; por tanto, AEXD es un trapecio
38 El circuncentro y el baricentro

X
E D
Y

A O F B

Figura 4.4. Los circuncentros de △BEX y △ADY coinciden.

isósceles con bases EX y AD. Similarmente, también lo es BY DE con bases BE y DY . Siendo así, las
mediatrices de AD y AY coinciden con las mediatrices de EX y BE. Por consiguiente, los circuncentros
de △BEX y △ADY coinciden.

Ejemplo 4.2. (IMO 2018, P1). Sea Γ la circunferencia circunscrita triángulo acutángulo ABC.
Los puntos D y E están en los segmentos AB y AC, respectivamente, y son tales que AD = AE.
Las mediatrices de BD y CE cortan a los arcos menores AB y AC de Γ en los puntos F y G,
respectivamente. Demostrar que las rectas DE y FG son paralelas (o son la misma recta).

S
A

G
T E
D
F

B C

Figura 4.5. Primer problema de la IMO 2018.

Solución. Si las rectas DE y FG coinciden no hay nada que probar, así que supongamos que son distintas.
4.4 Problemas propuestos 39

Como F y G están sobre las mediatrices de BD y CE, respectivamente, los triángulos BFD y CGE son
isósceles en F y G; de este modo, ∠FBA = ∠FBD = ∠FDB y ∠ACG = ∠ECG = ∠CEG.
Sea S = FD ∩ Γ, F ̸= S. Notemos que:
∠ASD = ∠ASF = ∠FBA = ∠FDB = ∠SDA
Es decir, △SAD es isósceles con AS = AD. Similarmente, si definimos a T como el segundo punto de
intersección de GE con Γ podemos conseguir que AT = AE. Hemos obtenido que AS = AD = AE = AT ,
lo cual implica que T, D, E y S están sobre la circunferencia de centro A y radio AD, es decir, T DES es
un cuadrilátero cíclico, por tanto:
∠T ED = ∠T SD = ∠T SF = ∠T GF
de modo que DE ∥ FG.

4.4 Problemas propuestos


Problema 4.1. Las medianas AD, BE y CF del triángulo ABC se cortan en el punto G. Si los cuadriláteros
AFGE y BDGF son cíclicos, probar que el triángulo ABC es equilátero.
Problema 4.2. (Sharygin 2019, Ronda Final, Grado 8, P1). Un trapecio con bases AB y CD está inscrito
en un círculo centrado en O. Sean AP y AQ las tangentes desde A al circuncírculo del triángulo CDO.
Probar que el circuncírculo del triángulo APQ pasa por el punto medio de AB.
Problema 4.3. (Suecia 2020, P2). Las medianas de los lados AC y BC del triángulo ABC son perpendi-
culares. Demostrar que 21 < BC
AC
< 2.
Problema 4.4. (Iberoamericana 2020, P1). Sea ABC un triángulo acutángulo y escaleno tal que AB < AC.
Los puntos medios de los lados AB y AC son M y N, respectivamente. Sean P y Q puntos en la recta MN
tales que ∠CBP = ∠ACB y ∠QCB = ∠CBA. La circunferencia circunscrita del triángulo ABP interseca
a la recta AC en D (D ̸= A) y la circunferencia circunscrita del triángulo AQC intersecta a la recta AB en
E (E ̸= A). Demostrar que las rectas BC, DP y EQ son concurrentes.
Problema 4.5. (Cuenca del Pacífico 2012, P1). Sea P un punto en el interior del triángulo ABC, y sean
D, E y F el punto de intersección de la recta AP y el lado CA, el de la recta BP y el lado CA, y el de la
recta CP y el lado AB, respectivamente. Probar que el área del triángulo ABC debe ser 6 si el área de cada
uno de los triángulos PFA, PDB y PEC es 1.
Problema 4.6. (IMO 2015, P4). El triángulo ABC tiene circunferencia circunscrita Ω y circuncentro O.
Una circunferencia Γ de centro A corta al segmento BC en los puntos D y E tales que B, D, E y C son
todos diferentes y están en la recta BC en este orden. Sean F y G los puntos de intersección de Ω y
Γ, tales que A, F, B, C y G están sobre Ω en este orden. Sea K el segundo punto de intersección de la
circunferencia circunscrita al triángulo BDF y el segmento AB. Sea L el segundo punto de intersección
de la circunferencia circunscrita al triángulo CGE y el segmento CA. Suponga que las rectas FK y GL
son distintas y se cortan en el punto X. Demostrar que X está en la recta AO.
Problema 4.7. (Sharygin 2019, Ronda Final, Grado 10, P1). Dado un triángulo ABC con ∠A = 45◦ , sea
A′ el punto diametralmente opuesto a A en el circuncírculo de ABC. Los puntos E y F yacen sobre los
segmentos AB y AC de modo que A′ B = BE y A′C = CF, respectivamente. Sea K la segunda intersección
de los circuncírculos de los triángulos AEF y ABC. Probar que EF biseca a A′ K.
40 El circuncentro y el baricentro
CAPÍTULO 5

LAS ALTURAS Y EL ORTOCENTRO

El tercer punto notable en nuestra lista corresponde al ortocentro. Como evidenciaremos a lo largo
de este capítulo, la configuración alturas-ortocentro posee una gran variedad de propiedades y por tal
motivo los problemas de olimpiada que las involucran son numerosos. Estas propiedades suelen incluir
al circuncentro y baricentro. Tal es el caso de la famosa recta de Euler.

5.1 Hechos fundamentales

Definición 5.1. El ortocentro es el punto de intersección de las rectas perpendiculares trazadas


desde cada vértice a su lado opuesto correspondiente, llamadas alturas.

F
H

B D C
Figura 5.1. El ortocentro H es el punto común de las alturas AD, BE y CF.

La demostración de la concurrencia de las alturas va de la siguiente manera. Definamos a E y F


como los pies de alturas desde B y C, respectivamente. Sea H el punto de intersección de CF y BE, y
construyamos el punto D = AH ∩ BC. Como ∠BFC = ∠BEC = 90◦ , los cuadriláteros BFEC y AFHE
son cíclicos, por tanto:
∠DBF = ∠CBA = ∠AEF = ∠AHF

lo que implica que BDHF también es cíclico, por consiguiente, ∠BDH = ∠HFA = 90◦ y AD es la A-
altura de △ABC.
El triángulo DEF es muy frecuente en olimpiadas. Cuando solamente se proporcionan dos pies de
alturas, es buena idea construir el tercero y así finalizar de construir la configuración.
42 Las alturas y el ortocentro

Definición 5.2. El triángulo formado por los pies de alturas del triángulo ABC recibe el nombre de
triángulo órtico de △ABC.

Para este triángulo, el ortocentro pasa a ser su incentro. Los vértices de △ABC también son puntos
notables de △DEF. En resumen:

Proposición 5.1. El ortocentro de △ABC es el incentro de su triángulo órtico △DEF. Además,


A, B y C son los excentros de △DEF.

Demostración. Véase la figura 5.1. Al ser AFDC y CEHD cuadriláteros cíclicos, notemos que:

∠HDF = ∠ADF = ∠ACF = ∠ECH = ∠EDH

por tanto, DA es bisectriz de ∠EDF. Análogamente podemos concluir que EB y CF son bisectrices
internas de ∠DEF y ∠EFD, respectivamente. De aquí el primer resultado.
Dado que BC es perpendicular a AD, entonces BD es la bisectriz externa de ∠EDF. Como C es la
intersección de BD y FH, entonces C es el F-excentro de △DEF. De forma similar, A y B son los D y
E-excentros de △ABC.

Analicemos otras definiciones y propiedades adicionales.

Definición 5.3. Los segmentos BH, CH y AH se nombran como segmentos de Euler del △ABC.

En lo que sigue, trabajaremos con un triángulo ABC acutángulo. Es posible modificar las demos-
traciones para el caso en que sea obtusángulo, en cuya situación el ortocentro yace en el exterior del
triángulo. Por supuesto, el ortocentro de un triángulo rectángulo es el vértice del ángulo recto.

Proposición 5.2. La reflexión de H con respecto a BC yace sobre el circuncírculo de △ABC.

Equivalentemente, también es cierto que los reflejos de H con respecto a los otros lados yacen sobre
(ABC).

Prueba. Sea H ′ el segundo punto de intersección de la altura AD con (ABC). Como BDEA es cíclico
sucede que:
∠H ′ BD = ∠H ′ BC = ∠H ′ AC = ∠DAE = ∠DBE = ∠DBH
lo que junto a ∠BDH = 90◦ indica que BC es la mediatriz de HH ′ ; de ahí el resultado.

Como los triángulos BHC y BH ′C resultan ser congruentes, los radios de sus circunferencias circuns-
critas coinciden en sus medidas; es decir, △ABC y △BHC tienen circunradios congruentes.

Proposición 5.3. Los circunradios del triángulo ABC y los tres triángulos formados por el orto-
centro y las tres duplas de vértices de △ABC son congruentes.

Existe una relación angular crucial entre el ortocentro y el circuncentro, como veremos a continua-
ción.
5.1 Hechos fundamentales 43

E
N

F
H
O

B D M C

H′ A′

Figura 5.2. Configuración alturas-ortocentro.

Proposición 5.4. Las rectas AH y AO forman ángulos iguales con los lados AB y AC, respectiva-
mente. Es decir, AH y AO son conjugadas isogonales del △ABC.

Una interpretación alternativa es que estas dos rectas son simétricas respecto a la bisectriz interna de
∠BAC. Ya que esta propiedad es cierta para BH, BO y CH, CO, lo que tenemos en realidad es que O y
H son puntos isogonales, uno del otro, en el triángulo ABC.

Prueba. Es suficiente ver que:


∠DAB = 90◦ − ∠ABC = ∠AOC

puesto que △AOC es isósceles en O y ∠AOC = 2∠ABC.

Consideremos el antípoda de A en (BAC), es decir, el punto diametralmente opuesto a A en el cir-


cuncírculo de △ABC. Como ∠A′ BA = ∠A′CA = 90◦ , inferimos que CH ∥ A′ B y BH ∥ A′C, de modo que
BHCA′ es un paralelogramo. Al ser M el punto medio de BC, esto indica que:

Proposición 5.5. El ortocentro H, el punto medio M del lado BC y el antípoda A′ de A en (BAC)


están alineados. Además, B, H, C y A′ forman un paralelogramo.

Sea N el punto medio del segmento de Euler AH. Es claro que AD ∥ OM. Como ON es base media
de HA′ en △HAA′ concluimos que OMHN es un paralelogramo. Esto implica que OM = NH = AH 2 , es
decir

Proposición 5.6. La distancia del circuncentro al punto medio de un lado es la mitad de la longitud
del segmento de Euler con vértice opuesto a ese lado.

¡Es momento de usar lo aprendido para resolver un problema!


44 Las alturas y el ortocentro

Ejemplo 5.1. (Rusia 2019, Grado 9, P3). El círculo Ω con centro O es el circuncírculo del triángulo
acutángulo ABC con AB < BC y ortocentro H. Sobre la recta BO yace un punto D tal que O está
entre B y D, y ∠ADC = ∠ABC. La semirrecta paralela a BO, que empieza en H y corta al segmento
AC, interseca a Ω en E. Probar que BH = DE.

Solución. Es suficiente demostrar que BDEH es un trapecio isósceles. Tenemos que ∠AHC = 180◦ −
∠ABC (¿por qué?), lo que junto a la hipótesis angular del problema implica que D está sobre el circun-
círculo de △AHC.
Sea P el simétrico de O con respecto al punto medio de AC, digamos M. Por la proposición 5.3,
inferimos que P es el circuncentro de △AHC. Es más, de acuerdo con la proposición 5.6 ocurre que OP =
BH. Puesto que BH ∥ OP, deducimos que BOPH es un paralelogramo, por ende, basta con demostrar que
ODEP es un trapecio isósceles. Se observa que:

PD = PC = OC = OE

Como OD ∥ PE, esto indica que OPED es un trapecio isósceles. ¡Estamos hechos!

O
H

A M C

P
E D

Figura 5.3. Trapecios isósceles y un paralelogramo

5.1.1 Lemas relacionados


En esta sección abordaremos algunos resultados mucho más particulares de la configuración alturas-
ortocentro cuyo reconocimiento ayuda significativamente a resolver completamente varios problemas de
olimpiada.

Lema 5.1. Si E y F corresponden a los pies de alturas desde B y C, y M es el punto medio del
lado BC, entonces ME y MF son tangentes al circuncírculo de △AEF.
5.1 Hechos fundamentales 45

A
P
E

F
H

B D M C

A′

Figura 5.4. Una tangencia oculta que involucra dos pies de alturas.

Prueba. Es claro que M es el circuncentro de BFEC. Observemos que:


∠EMF
∠FEM = 90◦ − = 90◦ − ∠ECF = ∠EAF
2
de donde surge el hecho deseado.
Otro hecho relacionado al círculo de diámetro AH es el siguiente:

Lema 5.2. El segundo punto de intersección del círculo de diámetro AH con el circuncírculo de
ABC, el ortocentro, el punto medio del lado BC y el antípoda de A en (ABC) están alineados.

Prueba. Sea P el segundo punto de intersección de ambos círculos. Por la proposición 5.5 ya sabemos
que H, M y A son colineales. Para terminar, notemos que:

∠APH = 90◦ = ∠APA′

de donde concluimos que P, H y A′ están alineados.


Resultados aparentemente inocentes como este permiten atacar problemas de dificultad alta, como el
problema 3 de la IMO 2015.
Otro lema a considerar es:

Lema 5.3 (Punto de Anti-Steiner). Sea ℓ una recta que pasa por el ortocentro H de △ABC. Luego,
las reflexiones de ℓ con respecto a BC, CA y AB concurren en un punto sobre el circuncírculo de
△ABC.

Prueba. Consideremos la figura 5.5. Definamos a HA , HB y HC como los reflejos de H en BC, CA y AB,
respectivamente. Sean P, Q y R los puntos de intersección de las reflexiones de ℓ con respecto a BC, AB
y AC, según corresponde. Por la proposición 5.2 sabemos que PHA , QHC y RHB son dichas reflexiones.
Basta con verificar que dos de las reflexiones concurren sobre (ABC). Sea S = HC Q ∩ HB R. Observe-
mos que (usando ángulos dirigidos):
46 Las alturas y el ortocentro

∡HB SHC = −(∡SQR + ∡QRS) = 2(∡RQA + ∡ARQ) = −2∡BAC = ∡HB BHC


así, S yace sobre (ABC), como necesitábamos.

A HB

HC R

H
Q

P B C

S
HA

Figura 5.5. El punto de Anti-Steiner correspondiente a la recta PQ.

Naturalmente, a cada recta que pasa por H le corresponde un punto de Anti-Steiner distinto. Verifica-
remos su utilidad en un problema de ejemplo.

5.2 Un círculo especial: El círculo de los nueve puntos


Ahora que hemos estudiado al circuncentro, centroide y ortocentro, es momento de explorar propie-
dades que relacionan estos tres puntos. En particular, probaremos la existencia de un círculo especial, a
saber, el círculo de los nueve puntos del triángulo ABC.

Definición 5.4. Los puntos medios de los lados del triángulo ABC, los puntos medios de sus
segmentos de Euler y los pies de sus alturas están sobre una misma circunferencia, llamada círculo
de los nueve puntos de ABC.

Prueba. Al ser OLDH un trapecio con DL perpendicular a sus bases, sigue que la mediatriz de DL pasa
por el punto medio de OH, digamos K. Por un argumento análogo se infiere que las mediatrices de NF y
EM pasan por K. Ya que MN ∥ BC y BFEC es cíclico deducimos que FNEM es cíclico. Similarmente,
LNFD y MLED son cuadriláteros inscritos. Esto implica que KD = KL = KM = KE = KN = KF, es
decir, los seis puntos D, L, M, E, N y F están sobre la circunferencia de centro K y radio KD.
Sean X, Y y Z los puntos medios de AH, BH y CH, respectivamente. Por supuesto, ZL ∥ BH y
Y L ∥ HC por ser bases medias, luego HY LZ es un paralelogramo. Por la proposición 5.5, se obtiene
que L está sobre (XY Z). Análogamente, M y N yacen en (XY Z), por lo que XNY LZM es un hexágono
cíclico. Como (DLMENF) y (XNY LZM) son los mismos círculos, concluimos que los nueve puntos son
concíclicos.
5.3 Problemas resueltos 47

X E
N M
O
G
K
F
H
Y Z
B D L C

Figura 5.6. El círculo de los nueve puntos de △ABC.

Puede conseguirse una demostración más sencilla mediante homotecia, una técnica que desarrollamos
en la parte de concurrencia y colinealidad. Como pudimos ver, el centro del círculo de los nueve puntos
coincide con el punto medio del segmento que une al ortocentro y circuncentro de △ABC. Sin embargo,
hay un punto notable adicional que también yace sobre OH: ¡el baricentro de △ABC!

Proposición 5.7 (Recta de Euler). El ortocentro H, el centroide G y el circuncentro O están sobre


una misma línea, llamada recta de Euler de △ABC. Además, G divide a OH en la razón 2 : 1.

Este hecho se obtiene a partir de las proposiciones 5.6 y 4.2. Debes ser capaz de probarlo por cuenta
propia.
Otras propiedades interesantes y que puedes verificar por tu propia parte son las siguientes:

Proposición 5.8. El circunradio de △XY Z es la mitad del circunradio de △ABC, es decir:


R
R△XY Z =
2

Proposición 5.9. El triángulo medial LMN y el triángulo XY Z son congruentes.

5.3 Problemas resueltos

Ejemplo 5.2. (Centroamericana 2017, P3). Dado un triángulo ABC, sean D el pie de la altura desde
A, y ℓ la recta que pasa por los puntos medios de AC y BC. Sea E la reflexión del punto D respecto
a ℓ. Demuestre que el circuncentro del triángulo ABC está sobre la recta AE.

Solución. Sea F el pie de altura desde C sobre AB. Por supuesto, AFDC es cíclico.
48 Las alturas y el ortocentro

O
D

B F A

Figura 5.7. Problema 3 de la OMCC 2017.

La recta ℓ es la mediatriz de ED; además, por coincidir con la base media de AB, también es la
mediatriz de CF. Esto implica que el cuadrilátero CEDF es un trapecio isósceles (y cíclico). Así, los
puntos A, F, D, E y C son concíclicos con diámetro CA.
Como ED ∥ CF surge que CE c = DF,
c de donde ∠CAE = ∠DAF = ∠DAB. Si definimos a O como el
circuncentro de △ABC, por la proposición 5.4 sabemos que ∠CAO = ∠DAB. Concluimos que ∠CAE =
∠CAO; por consiguiente, O yace sobre AE.

Ejemplo 5.3. (Balcánica Junior 2019, P3) El triángulo ABC es tal que AB < AC. La mediatriz del
lado AC corta a los lados AB y AC en P y Q, respectivamente. Sea H el ortocentro de ABC y sean
M y N los puntos medios de los segmentos BC y PQ, respectivamente. Probar que las rectas HM
y AN se cortan en el circuncírculo de ABC.

A N

R
Q
E

D
H

B M C

Figura 5.8. Dos rectas que se cortan sobre el circuncírculo de ABC.


5.3 Problemas resueltos 49

Solución. Notemos que:


∠APQ = 90◦ − ∠ABC = ∠HCB y ∠PQA = 90◦ − ∠BCA = ∠CBH
así que los triángulos PAQ y CHB son semejantes. Al ser HM y AN medianas correspondientes de estos
triángulos, podemos inferir que ∠PAN = ∠CHM.
Definamos a R como el punto de intersección de HM y AN, mientras que D es el pie de altura desde
C. Por la igualdad angular anterior obtenemos que ∠RHD = ∠CHM = ∠PAN = ∠RAD, por tanto R yace
en el circuncírculo de △ADH.
Finalmente, por el lema 5.2 concluimos que R también está sobre el circuncírculo de ABC.
Como último ejemplo, resolvamos un problema difícil.

Ejemplo 5.4. (Europea Femenil 2017, P6). Sea ABC un triángulo acutángulo en el que no hay dos
lados con la misma longitud. Las reflexiones del centroide G y el circuncentro O de ABC respecto
a sus lados BC, CA, AB se denotan por G1 , G2 , G3 y O1 , O2 , O3 , respectivamente. Demostrar
que los circuncírculos de los triángulos G1 G2C, G1 G3 B, G2 G3 A, O1 O2C, O1 O3 B, O2 O3 A y ABC
tienen un punto en común.

G2
O3 O2
G3 O
G

B C

D
G1
O1

Figura 5.9. D es el punto de Anti-Steiner de la recta de Euler de △ABC.

Solución. Sabemos que el ortocentro de △ABC está sobre la recta de Euler del mismo triángulo, que
también pasa por G y O. De acuerdo con el lema 5.3, las rectas O1 G1 , O2 G2 y O3 G3 concurren sobre
(ABC), en el punto de Anti-Steiner de la recta de Euler de ABC, digamos D. En términos de ángulos
dirigidos tenemos que:
∡O1 G1C = ∡CGO = ∡O2 G2C y ∡CO1 G1 = ∡GOC = ∡CO2 G2
luego G1CG2 D y O1CO2 D son cíclicos. Perfectamente conseguimos igualdades angulares análogas para
probar que el resto de circuncírculos pasan por D.
50 Las alturas y el ortocentro

5.4 Problemas propuestos


Problema 5.1. Sean AD, BE y CF las alturas de un triángulo acutángulo ABC y sea H su ortocentro. Sea
N el punto medio de AH y sea M el punto medio de BC. Probar que NM es perpendicular a FE.

Problema 5.2. (Caucásica 2021, Nivel Senior, P4). En el triángulo acutángulo ABC sean AHA y BHB las
alturas. La recta HA HB interseca al circuncírculo de ABC en P y Q. Sea A′ la reflexión de A en BC,y sea
B′ la reflexión de B en CA. Probar que A′ , B′ , P, Q forman un cuadrilátero cíclico.

Problema 5.3. (Iberoamericana 2011, P4). Sea ABC un triángulo acutángulo con AC ̸= BC y con cir-
cuncentro O. Sean P y Q puntos tales que BOAP y COPQ son paralelogramos. Demostrar que Q es el
ortocentro de ABC.

Problema 5.4. (Centroamericana 2019, P4). Sea ABC un triángulo, Γ su circuncírculo y ℓ la tangente a
Γ que pasa por A. Las alturas desde B y C se prolongan y cortan a ℓ en D y E, respectivamente. Las rectas
DC y EB cortan a Γ nuevamente en P y Q, respectivamente. Demostrar que el triángulo APQ es isósceles.

Problema 5.5. Sea AD la altura desde A en el triángulo ABC. Sean X e Y los puntos medios de las otras
dos alturas, H el ortocentro y M el punto medio de BC. Demostrar que:
El circuncírculo de DXY pasa por H y por M; y,
Los triángulos ABC y DXY son semejantes.

Problema 5.6. (Panafricana 2017, P6). Sea ABC un triángulo y H su ortocentro. El círculo de diámetro
AC corta al circuncírculo de ABH por segunda vez en K. Probar que la intersección de las rectas CK y
BH es el punto medio del segmento BH.

Problema 5.7. (Centroamericana 2019, P3). Sea ABC un triángulo y Γ su circuncírculo. Sea D el pie
de altura desde A al lado BC, M y N los puntos medios de AB y AC, y Q el punto en Γ diametralmente
opuesto a A. Sea E el punto medio de DQ. Demostrar que las perpendiculares a EM y EN que pasan por
M y N, respectivamente, se cortan sobre AD.

Problema 5.8. (IMO 2013, P4). Sea ABC un triángulo acutángulo con ortocentro H, y sea W un punto
sobre el lado BC, estrictamente entre B y C. Los puntos M y Nson los pies de las alturas trazadas desde B
y C respectivamente. Se denota por ω1 la circunferencia que pasa por los vértices del triángulo BW N, y
por X el punto de ω1 tal que W X es un diámetro de ω1 . Análogamente, se denota por ω2 la circunferencia
que pasa por los vértices del triángulo CW M, y por Y el punto de ω2 tal que WY es un diámetro de ω2 .
Demostrar que los puntos X, Y y H son colineales.

Problema 5.9. (Lista Corta IMO 2017, G3). Sea O el circuncentro de un triángulo escaleno ABC. La
recta OA interseca a las alturas de ABC que pasan por B y C en P y Q, respectivamente. Las alturas se
cortan en H. Probar que el circuncentro del triángulo PQH yace sobre una mediana del triángulo ABC.

Problema 5.10. (Sharygin 2016, Ronda de Correspondencia, P22). Sean MA , MB y MC los puntos medios
de los lados de un triángulo no isósceles ABC. Los puntos HA , HB , HC yacen en los lados correspondien-
tes, diferentes de MA , MB , MC , y satisfacen que MA HB = MA HC , MB HA = MB HC y MC HA = MC HB . Probar
que HA , HB y HC son los pies de las alturas del triángulo ABC.
CAPÍTULO 6

BISECTRICES, INCENTRO Y EXCENTROS

6.1 Preliminares

Definición 6.1. La bisectriz del ángulo BAC es el lugar geométrico de los puntos cuyas distancias
a las rectas AB y AC son congruentes.

Existen dos rectas que satisfacen esta definición, nombradas bisectriz interna y bisectriz externa. La
bisectriz interna del ángulo BAC coincide con la recta que lo divide en dos partes iguales. Entretanto, la
bisectriz externa es la recta que divide al suplemento de ∠BAC en dos partes iguales.

A partir de estas definiciones surge directamente que las bisectrices interna y externa son perpendi-
culares, ¡pruébalo! Es importante mantener este hecho en mente.

B P C Q

Figura 6.1. Las bisectrices interna y externa de ∠BAC forman un ángulo recto.

Consideremos al triángulo ABC, los segmentos en los que el lado BC está dividido por las bisectrices
de ∠BAC, y los lados AB y AC.

Teorema 6.1. (Teorema de la bisectriz). Sea Q un punto sobre el segmento BC y Q un punto sobre
la prolongación de BQ. Luego, AP y AQ son las bisectrices interna y externa de ∠BAC si y solo si:
BP AB BQ
= =
PC AC CQ

El teorema de Steiner brinda una prueba directa, al igual que la ley del seno aplicado a los triángulos
BAP y CAP. Los detalles se dejan como ejercicio al lector.
Al igual que las rectas notables tratadas en las secciones anteriores, las bisectrices internas poseen un
punto común.
52 Bisectrices, incentro y excentros

Definición 6.2. Las bisectrices internas de un triángulo ABC concurren en un punto, llamado el
incentro de △ABC.

Prueba. Sea I el punto de corte de las bisectrices internas de ∠ABC y ∠ACB, y definamos a D como el
punto donde AI interseca a BC. Por el teorema de la bisectriz

AB AI AC
= =
BD ID CD
Reordenando, concluimos por el recíproco del teorema de la bisectriz que ∠BAD = ∠DAC, por ende,
las bisectrices internas de △ABC concurren en I.

Junto a la definición 6.1, lo anterior implica la existencia de un círculo centrado en I y tangente


simultáneamente a los tres lados de △ABC.

Definición 6.3. El incírculo del triángulo ABC es la circunferencia con centro en I y tangente a
los lados BC, CA y AB. El radio de este círculo se conoce como inradio de △ABC, usualmente
denotado por r.

M′

A
E

I O

B D C

M
Figura 6.2. El incírculo de △ABC y algunos hechos relacionados.
Las bisectrices externas también están involucradas en ciertas concurrencias. De esto nos encargare-
mos más adelante. Por ahora, consideremos la siguiente propiedad.

Proposición 6.1. El ángulo AIC equivale a 90◦ + ∠BAC


2 .

La demostración es manipulación angular simple, por lo que queda como ejercicio al lector.
Supongamos que el rayo AI corta a (ABC) en M. Es claro que la bisectriz interna de ∠BAC determina
arcos iguales sobre (ABC), de ahí que BM = MC; pero nótese que:

∠MIC = ∠IAC + ∠ACI = ∠BAM + ∠ICB = ∠BCM + ∠ICB = ∠ICM


6.2 Bisectrices externas y excentros 53

lo que implica que IM = CM. En resumen:

Proposición 6.2. El punto medio del arco menor BC es el circuncentro de △BIC.

Inquiramos acerca de la potencia de I respecto a (ABC). Al ser un punto en el interior de △ABC y


por ende dentro de (ABC), tenemos:
R2 − OI 2 = AI · IM
Sea M ′ el antípoda de M. Como ∠AMM ′ = 90◦ , inferimos que AM ′ es la bisectriz externa de ∠BAC. Sea
E el punto de contacto del incírculo de △ABC con el lado AC. Es simple probar que △IAE ∼ △MM ′C,
de donde extraemos que:
AI · MI = 2rR
por consiguiente, hemos probado que:

Proposición 6.3. (Relación de Euler). La distancia del incentro al circuncentro, el inradio y el


circunradio están vinculados en la siguiente expresión

OI 2 = R(R − 2r)
En particular, se tiene que R ≥ 2r, con igualdad si y solo si △ABC es equilátero.

6.2 Bisectrices externas y excentros


Ahora es el turno de hablar un poco más acerca de las bisectrices externas, cuadriláteros cíclicos y
puntos de concurrencia relacionados. Mantengamos la notación utilizada en el apartado anterior.

Proposición 6.4. La bisectriz interna de ∠BAC y las bisectrices externas de ∠ABC y ∠ACB con-
curren en un punto, el excentro de △ABC opuesto al vértice A.

Es claro que △ABC cuenta con tres excentros. Por motivos de brevedad, usaremos el término A-
excentro para referirnos al excentro correspondiente al vértice A.

Prueba. Sea IA el punto de intersección de las bisectrices externas de ∠B y ∠C. Como

∠IBIA = ∠ICIA = 90◦

tenemos que BICIA es cíclico. Lo anterior implica que IA es el antípoda de I en (BICIA ). Por la proposición
6.2 concluimos que IA está sobre IM, por ende, AI también pasa por IA .

Esta concurrencia implica la existencia de un círculo centrado en IA , exterior a △ABC, tangente a sus
tres lados.

Definición 6.4. El excírculo de △ABC opuesto a A (brevemente nombrado como A-excírculo) es


el círculo con centro en el A-excentro y que toca al lado BC y las prolongaciones de AB y AC. El
A-exradio es el radio de este círculo, denotado comúnmente como rA .
54 Bisectrices, incentro y excentros

A
D′
E
F
I
B D T C
Q

P
IA

T′
Figura 6.3. El A-excírculo de △ABC y propiedades relacionadas.

Sea D′ el antípoda de D en el incírculo de △ABC, y T el punto de contacto del A-excírculo con BC.
AI r ID′ ′ ′
Como AI A
= rA = IA T (¿por qué?) y D I ∥ T IA , inferimos con ayuda del teorema de Tales que A, D y T
son colineales.

Lema 6.1. El antípoda del punto de tangencia del incírculo con BC, el punto de contacto del
A-excírculo y A están alineados.

Pese a su sencillez, el uso de este resultado es frecuente. Por un argumento análogo, A, D y T ′ , el


antípoda de T en el A-excírculo, están sobre la misma recta. Otro hecho de amplia utilidad es el siguiente.

Lema 6.2. Los puntos de tangencia del incírculo y A-excírculo con BC son simétricos respecto al
punto medio de este lado, es decir, BD = EC.

El ejemplo más evidente donde se usan estos lemas es el problema 6 de la IMO 2008, que resolvemos
en el capítulo de homotecia.
Prueba. Definamos a P y Q como los puntos de tangencia del A-excírculo con AB y AC, respectivamente.
Como es usual, denotemos por a, b, c y s las longitudes de BC, CA, AB y el semiperímetro de △ABC.
Observemos que

AP + AQ = AB + BP + AC +CQ = AB + BE + EC + AC = AB + BC +CA = 2s

así que AP = AQ = s. Por consiguiente CE = CQ = AQ − AC = s − b, pero

BD = AB + BC − BD − AF −CD = a + c − s = 2s − b − s = s − b

como requeríamos.
6.3 Los puntos de Gergonne y Nagel 55

IB
A
IC

I
O
V
B C

IA

Figura 6.4. Los tres excírculos del triángulo ABC.

Cuando dos puntos son simétricos respecto al punto medio de un lado del triángulo, diremos que son
puntos isotómicos. En este caso, D y T son isotómicos.
La configuración completa de △ABC y sus excírculos se muestra en la figura 6.4. El triángulo formado
por IA , IB e IC es el triángulo excentral de △ABC. Notemos que, para △IA IB IC , el incentro de △ABC es
su ortocentro, el circuncírculo de △ABC es su círculo de los nueve puntos y el circuncentro de △ABC es
su correspondiente centro de los nueve puntos. Esto quiere decir que el simétrico de I con respecto a O,
digamos V , es el circuncentro de △IA IB IC . A este punto se le conoce como el punto de Bevan de △ABC.

6.3 Los puntos de Gergonne y Nagel


La configuración incírculo-excírculos guarda dos concurrencias que hasta ahora no habíamos consi-
derado.

Definición 6.5. El punto de Gergonne es el punto de concurrencia de los segmentos que unen cada
vértice de △ABC con el punto de tangencia del incírculo y el lado opuesto correspondiente.

En nuestro caso, las rectas AD, BE y CF se cortan en el mismo punto J, el punto de Gergonne de
△ABC.
56 Bisectrices, incentro y excentros

F′ E
F E′
J N

B D D′ C

Figura 6.5. Los puntos de Gergonne y Nagel del triángulo ABC.

Definición 6.6. El punto de Nagel corresponde al punto de intersección de los segmentos que unen
cada vértice de △ABC con el punto de tangencia del excírculo y el lado opuestos a ese vértice.

En efecto, el punto de Nagel de △ABC está dado por N, el punto de corte de AD′ , BE ′ y CF ′ , como
se muestra en la figura 6.5.
Por ahora, tomaremos estas concurrencias como ciertas. Como BD = BF, AF = AE y CE = CD, por
el teorema de Ceva (que estudiaremos más adelante), la prueba de la intersección común de AD, BE
y CF ′ es simple. Como los puntos de tangencia del incírculo y el excírculo correspondiente a un lado
son isotómicos según el lema 6.2, este mismo teorema asegura la concurrencia de AD′ , BE ′ y CF ′ . De
hecho, el lema recién utilizado implica que los puntos de Nagel y Gergonne son conjugados isotómicos
de △ABC.
El punto de Nagel está involucrado en una colinealidad especial.

Proposición 6.5 (Recta de Nagel). El incentro I, el baricentro G y el punto de Nagel N del trián-
gulo ABC yacen sobre una misma recta, llamada recta de Nagel de △ABC. Además, G divide al
segmento NI en la razón 2 : 1.

Prueba. Consideremos la figura 6.6. Definamos a L como el punto medio de BC y R el antípoda de D en


el incírculo de △ABC.
De acuerdo con el lema 6.1, A, R y D′ están alineados. Nótese que IL es base media respecto a RD′ ,
así que LI ∥ D′ A, pero AG = 2GL, entonces si N ′ = IG ∩ AD′ tenemos que GN ′ = 2IG. Un punto N ′
con esta propiedad se define de manera única, así que también debe estar sobre BE ′ y CF ′ , por tanto
N = N ′.

Por la proposición 6.5, es claro que AN = 2IL = RD′ , así que AR = ND′ . Este resultado es básicamente
el problema 2 de la Olimpiada Matemática de Estados Unidos (USAMO por sus siglas en inglés) de 2001.
6.3 Los puntos de Gergonne y Nagel 57

R
F′

E′
I G N

B D L D′ C

Figura 6.6. La recta de Nagel de △ABC.

6.3.1 Dos lemas adicionales


Continuemos considerando a D, E y F como los puntos de tangencia del incírculo con los lados
BC, CA y AB de un triángulo ABC. Tomemos a M como el punto medio de BC. Tenemos la siguiente
relación especial:

Lema 6.3. Las rectas EF, DI y AM son concurrentes.

E
S P T
F
I

B D M C
Figura 6.7. Una concurrencia extra que involucra la A-mediana.

Prueba. Sea P = AM ∩ DI. Los puntos S y T están sobre AB y AC, respectivamente, tales que ST pasa
por P y ST ∥ BC. De esta forma, P es el punto medio de ST e ∠IPT = 90◦ , así que △SIT es isósceles con
IS = IT . Como IF = IE y ∠IFS = ∠IET = 90◦ , ocurre que △IFS ∼ = △IET . Además, los cuadriláteros
IPSF e IT EP son cíclicos, por consiguiente:

∠IT E = ∠ISF = ∠IPF

luego, P también está sobre EF.

El segundo hecho concierne a una perpendicularidad y concurrencia que suelen estar ocultas en los
problemas.
58 Bisectrices, incentro y excentros

Lema 6.4. La recta EF, la bisectriz interna de ∠B y la base media respecto al lado AB son
concurrentes en un punto P. Es más, ∠BPC = 90◦ .

P
E

F
I

B D M C

Figura 6.8. Perpendiculares ocultas.

Prueba. Tomemos a P como la intersección de EF y BI. Primero probaremos la perpendicularidad. En


términos de ángulos dirigidos, observemos que:

∡CIP = ∡CBI + ∡ICB = 90◦ − ∡IAC = ∡AEF = ∡CEP

lo que implica que E y P yacen sobre la circunferencia de diámetro CI, que también contiene a D.
Por su parte, como P también está sobre la circunferencia de diámetro BC, extraemos que:

∠BPM = ∠CBP = ∠PBA

por ende PM ∥ AB y por tanto PM es la recta que une los puntos medios de BC y AC.

6.4 Problemas resueltos

Ejemplo 6.1. (IMO 2006, P1). Sea ABC un triángulo con incentro I. Un punto P en el interior
del triángulo satisface que ∠PBA + ∠PCA = ∠PBC + ∠PCB. Demostrar que AP ≥ AI, y que la
igualdad se cumple si y solo si P = I.

Solución. Por suma de ángulos internos, la condición angular es equivalente a:


∠BAC
∠PBC + ∠PCB = 90◦ −
2
por tanto, ∠BPC = 90◦ + ∠BAC/2. Como el ángulo ∠AIC también es igual al último valor según la
proposición 6.1, el cuadrilátero BPIC es cíclico. Ya que el circuncentro de △BIC (llamémoslo M) está
6.4 Problemas resueltos 59

I
P

B C

Figura 6.9. Problema 1 de la IMO 2006.

dado por la segunda intersección de AI con el circuncírculo de △ABC, por la desigualdad triangular en
△APM podemos obtener que:
AP + PM ≥ AI + IM
de donde el resultado sigue. La igualdad ocurre cuando A, P y M están alineados; es decir, cuando P =
I.

Ejemplo 6.2. ([Link]. 2015, Selectivo IMO, P1) Sea ABC un triángulo escaleno con incentro I
cuyo incírculo es tangente a BC, CA y AB en D, E y F, respectivamente. Denote por M el punto
medio de BC. Sea Q un punto sobre el incírculo tal que ∠AQD = 90◦ . Sea P un punto dentro del
triángulo sobre la recta AI tal que MD = MP. Probar que ∠PQE = 90◦ o bien ∠PQF = 90◦ .

K
E

F
I Q
P

B D M D′ C

Figura 6.10. Primer problema del Selectivo IMO 2015 de [Link].

Solución. Sin pérdida de generalidad, supongamos que AC > AB. Probaremos que ∠PQE = 90◦ .
Sea K el antípoda de D en el incírculo, y D′ el simétrico de D respecto a M. Tenemos que ∠KQD =
∠AQD = 90◦ . Además, A, K y D′ son colineales por los lemas 6.1 y 6.2. Concluimos que A, Q y D′ están
60 Bisectrices, incentro y excentros

alineados. Como ∠DQD′ = 90◦ y MP = MD, se infiere que DPQD′ es un cuadrilátero cíclico con centro
M y diámetro DD′ .
Sea P′ = AI ∩ ED. Por el lema 6.4, sabemos que MP′ ∥ AC. Como △ECD es isósceles en C, el
paralelismo anterior implica que △P′ MD también lo es en M, así que P = P′ .
Finalmente, ya que ID ⊥ DM deducimos que ID es tangente a (DPQD′ ), por tanto:

∠EQA = ∠EQK = ∠EDK = ∠PQD

luego, ∠EQP = ∠AQD = 90◦ , como requeríamos.

Ejemplo 6.3. (IMO 2013, P3). Supongamos que el excírculo del triángulo ABC opuesto al vértice
A es tangente al lado BC en el punto A1 . Análogamente, se definen los puntos B1 en CA y C1 en
AB, utilizando los excírculos opuestos a B y C, respectivamente. Supongamos que el circuncentro
del triángulo A1 B1C1 pertenece a la circunferencia que pasa por los vértices A, B y C. Demostrar
que el triángulo ABC es rectángulo.

A M

C1
B1

B D A1 C
Figura 6.11. Tercer problema de la IMO 2013.

Solución. Sea M el punto medio de BAC d y M ′ el circuncentro de △A1 B1C1 . Como M ′ está sobre (ABC)
y exterior a △A1 B1C1 , este triángulo es obtusángulo. Sin pérdida de generalidad, sea ∠B1 A1C1 > 90◦ .
Tenemos que MC = MB y BC1 = s − a = CB1 , además:

∠MBC1 = ∠MBA = ∠MCA = ∠MCB1

por lo que △BMC1 ∼ = △CMB1 , es decir MC1 = MB1 , de donde M = M ′ .


Sean D y E los puntos de contacto del incírculo y C-excírculo de △ABC con BC y AB, respectiva-
mente. Por simetría y según el lema 6.2 concluimos que D está sobre (A1 B1C1 ). Además, como MA es
mediatriz de C1 E, también E yace sobre (A1 B1C1 ). Concluimos que EB1 A1 D es cíclico, por tanto:

s(s − a) = CE ·CB1 = CD ·CA1 = (s − c)(s − b)

Simplificando la expresión anterior se extrae que a2 = b2 + c2 . Por el recíproco del teorema de Pitá-
goras concluimos que △ABC es recto en A, como deseábamos.
6.5 Problemas propuestos 61

6.5 Problemas propuestos


Problema 6.1. Sea ABCD un cuadrilátero cíclico. Sean IA , IB , IC e ID los incentros de los triángulos
BCD, CDA, DAB y ABC, respectivamente. Demostrar que IA IB IC ID es un rectángulo.
Problema 6.2. Sea ABC un triángulo acutángulo con ∠BAC = 60◦ . Probar que IH = IO, donde I, H y
O son el incentro, ortocentro y circuncentro de ABC, respectivamente.
Problema 6.3. (Centroamericana 2020, P4). Considere un triángulo ABC con BC > AC. El círculo con
centro C y radio AC corta al segmento BC en D. Sea I el incentro del triángulo ABC y Γ el círculo que
pasa por I y es tangente a la recta CA en A. La recta AB y Γ se cortan en un punto F con F ̸= A. Probar
que BF = BD.
Problema 6.4. (Iberoamericana 2013, P4). Sean Γ una circunferencia de centro O, AE un diámetro de Γ
y B el punto medio de uno de los arcos AE de Γ. El punto D ̸= E está sobre el segmento OE. El punto C
es tal que el cuadrilátero ABCD es un paralelogramo con AB paralelo a CD y BC paralelo a AD. Las rectas
EB y CD se cortan en el punto F. La recta OF corta al arco menor EB de Γ en el punto I. Demostrar que
la recta EI es la bisectriz del ángulo BEC.
Problema 6.5. (Hong Kong 2020, 2do Selectivo IMO, P1). Sea ABC un triángulo acutángulo con incentro
I y ortocentro H. La recta AI corta al circuncírculo de ABC por segunda vez en M. Suponga que el
segmento IM es igual en longitud al circunradio de ABC. Probar que AH ≥ AI.
Problema 6.6. (IMO 2012, P1). Dado un triángulo ABC, el punto J es el centro del excírculo opuesto al
vértice A. Este excírculo es tangente al lado BC en M, y a las rectas AB y AC en K y L, respectivamente.
Las rectas LM y BJ se cortan en F, y las rectas KM y CJ se cortan en G. Sea S el punto de intersección
de las rectas AF y BC, y sea T el punto de intersección de las rectas AG y BC. Demostrar que M es el
punto medio de ST .
Problema 6.7. (Cuenca del Pacífico 2017, P2). Sea ABC un triángulo con AB < AC. Sea D el punto
de intersección de la bisectriz interior del ángulo BAC y el circuncírculo de ABC. Sea Z el punto de
intersección de la mediatriz de AC con la bisectriz exterior del ángulo BAC. Probar que el punto medio
del segmento AB yace sobre el circuncírculo de ADZ.
Problema 6.8. (Lista Corta IMO 2016, G4). Sea ABC un triángulo con AB = AC ̸= BC y sea I su incentro.
La recta BI corta a AC en D, y la recta por D perpendicular a AC corta a AI en E. Probar que la reflexión
de I respecto a AC yace en el circuncírculo del triángulo BDE.
Problema 6.9. (Iraní de Geometría 2021, Nivel Intermedio, P4) Sea ABC un triángulo escaleno acután-
gulo con incentro I y circuncírculo Γ. La recta AI corta a Γ por segunda vez en M. Sea N el punto medio
de BC y sea T el punto sobre Γ tal que IN ⊥ MT . Finalmente, sean P y Q los puntos de intersección de
T B y TC con la perpendicular a AI por I, respectivamente. Demostrar que PB = CQ.
Problema 6.10. (Lista Corta IMO 2005, G1). En el triángulo ABC con AB + AC = 3AC, el incírculo tiene
centro I y toca a los lados AB y BC en D y E, respectivamente. Sean K y L los simétricos de D y E con
respecto a I. Probar que el cuadrilátero ACKL es cíclico.
Problema 6.11. (Centroamericana 2011, P6). Sea ABC un triángulo acutángulo y D, E y F los pies de
alturas desde A, B y C, respectivamente. Sean Y y Z los pies de las perpendiculares desde B y C hacia FD
y DE, respectivamente. Sea F1 el simétrico de F con respecto a E y E1 el simétrico de E con respecto a
F. Si 3EF = FD + DE, probar que ∠BZF1 = ∠CY E1 .
62 Bisectrices, incentro y excentros

Problema 6.12. (Centroamericana 2012, P6). Sea ABC un triángulo con AB < BC, y sean E y F puntos
en AC y AB tales que BF = BC = CE, ambos en el mismo semiplano de A con respecto a BC. Sea G la
intersección de BE con CF. Sea H un punto sobre la paralela por G a AC tal que HG = AF (con H y C
en semiplanos opuestos con respecto a BG). Demostrar que ∠EHG = ∠BAC 2 .

Problema 6.13. (Lista Corta IMO 2012, G6) Sea ABC un triángulo con circuncentro O e incentro I.
Los puntos D, E y F yacen en los lados BC, CA y AB, respectivamente, tales que BD + BF = CA
y CD + CE = AB. Los circuncírculos de los triángulos BFD y CDE se cortan en P ̸= D. Probar que
OP = OI.

Problema 6.14. (Lista Corta IMO 2019, G6). Sea I el incentro del triángulo acutángulo ABC. El incírculo
toca a BC, CA y AB en D, E y F, respectivamente. La recta EF corta al circuncírculo del triángulo en P
y Q, tal que F yace entre E y P. Probar que ∠DPA + ∠AQD = ∠QIP.
PARTE III

CONFIGURACIONES ÚTILES
CAPÍTULO 7

CONJUGADOS ISOGONALES Y SIMEDIANAS

En la parte anterior abordamos las propiedades más conocidas de los puntos y rectas notables del
triángulo. En esta, exploraremos algunas configuraciones que aparecen con regularidad en olimpiadas.
Comenzaremos con conjugados isogonales (que introducimos brevemente en el capítulo de alturas y el
ortocentro) y simedianas.

7.1 Conjugados isogonales


Consideremos un triángulo ABC. Dos rectas ℓ1 y ℓ2 son conjugadas isogonales del △ABC si ambas
pasan por el mismo vértice del triángulo y una es la reflexión de la otra respecto a la bisectriz interna de
△ABC; esto es, que formen ángulos iguales con los lados del triángulo que comparten ese vértice.

B C

Figura 7.1. Un par de conjugadas isogonales de △ABC.

En la segunda sección hablaremos acerca de un par particular de conjugadas isogonales: la mediana y


la simediana. La última contiene propiedades interesantes y útiles que valen la pena estudiar por separado.
Otro ejemplo común de rectas isogonales son las alturas de △ABC y los circundiámetros correspondientes
a cada vértice.
Cuando tres rectas concurren, sus respectivas conjugadas isogonales también poseen un mismo punto
de intersección.1 El punto de concurrencia es el conjugado o punto isogonal del punto de concurrencia de
nuestras rectas originales. Por ejemplo, el ortocentro es el conjugado isogonal del ortocentro y viceversa.
Por supuesto, el incentro es su propio conjugado isogonal.
En lo que sigue de este apartado, centraremos nuestra atención en propiedades cruciales de rectas y
puntos isogonales.
1 La prueba viene por el Teorema de Ceva en su forma trigonométrica.
66 Conjugados isogonales y simedianas

Teorema 7.1 (Steiner). Sean D y E dos puntos sobre el segmento BC del triángulo ABC.
Luego,∠BAE = ∠DAC si y solo si:
BD BE AB2
· =
CD CE AC2

Prueba. Dos aplicaciones del teorema de la bisectriz generalizado son más que suficiente.

El siguiente resultado es menos simple de demostrar.

Proposición 7.1 (Triángulos pedales de conjugados isogonales). Sean P y Q dos puntos copla-
nares con △ABC. Definamos a X1 , Y1 y Z1 como las proyecciones de P sobre BC, CA y AB,
respectivamente. Los puntos X2 , Y2 y Z2 se construyen de forma análoga para el punto Q. Luego,
si P y Q son conjugados isogonales entonces los triángulos X1Y1 Z1 y X2Y2 Z2 comparten el mismo
circuncírculo. Es más, el circuncentro de esta circunferencia común es el punto medio de PQ.

Y1
Z2
Y2
Q
Z1 M
P

B X1 X2 C

Figura 7.2. Los triángulos pedales de dos puntos isogonales comparten su circuncírculo.

Prueba. Veamos que △Z1 AP ∼ △Y2 AQ y △Z2 AQ ∼ △Y1 AP por criterio AA, por consiguiente:

AZ1 AP AY1
= = ∴ AZ1 · AZ2 = AY1 · AY2
AY2 AQ AZ2

por ende Z1 Z2Y1Y2 es cíclico. Como PQY2Y1 y PQZ2 Z1 son trapecios, el circuncírculo de Z1 Z2Y1Y2 es el
punto medio de PQ. De forma similar podemos demostrar que Z2 Z1 X1 X2 y X1 X2Y2Y1 son cíclicos con
circuncentro en el punto medio de PQ, así que estos seis puntos están sobre una misma circunferencia
centrada en dicho punto.

Si Q es el circuncentro de △ABC podemos deducir que P es su ortocentro. Básicamente, hemos


redescubierto seis puntos del círculo de los 9 puntos de ABC.
Vale mencionar que el recíproco de este resultado también es válido.
7.1 Conjugados isogonales 67

Proposición 7.2. Sean P y Q conjugados isogonales del △ABC. Los circuncentros de los triángu-
los BPC (OP ) y BQC (OQ ) satisfacen que:

OOP · OOQ = r2

donde O es el circuncentro y r el circunradio de △ABC.

Demostración. Para comenzar, O, OP y OQ yacen sobre la mediatriz de BC; es decir, son colineales.
Por otro lado, notemos que ∠PBC +∠PCB+∠QBC +∠QCB = (∠PBC +∠ABP)+(∠PCB+∠ACP) =
∠ABC + ∠ACB = 180◦ − ∠BAC. Entonces:

∠BPC + ∠BQC = 360◦ − (∠PBC + ∠PCB + ∠QBC + ∠QCB) = 180◦ + ∠BAC

Además, podemos conseguir que ∠BOP O = 180◦ −∠BPC y ∠OBOQ = ∠BQC −∠BAC = 180◦ −∠BPC =
∠BOP O. Luego, por criterio AA inferimos que △OBOP ∼ △OOQ B, de donde surge directamente la pro-
piedad deseada.

P
O Q

B C
OP

OQ

Figura 7.3. Los circuncentros de △BPC y △BQC son inversos respecto a (ABC).

Cuando dos puntos X e Y son colineales con O y satisfacen que OX · OY = r2 , estos puntos reciben el
nombre de inversos con respecto a (ABC). En este caso, el resultado 7.2 indica que OP y OQ son inversos
respecto a (ABC). La inversión es tan importante que le dedicamos un capítulo exclusivo en la parte de
introducción a la geometría proyectiva.
El resultado a continuación también es útil para resolver problemas olímpicos.

Lema 7.1. (Reino Unido 2012, Ronda 2, P2). El punto P yace en el interior del triángulo ABC
tal que ∠ABP = ∠CPA. El punto Q es tal que PQBC es un paralelogramo. Probar que ∠QAB =
∠CAP.
68 Conjugados isogonales y simedianas

A′

B C

Figura 7.4. Una generalización de la proposición 5.5.

Demostración. Sea A′ el punto tal que AA′ BP es un paralelogramo. Como AA′ = BP = CQ y AA′ ∥ CQ,
también AA′ QC es un paralelogramo. Ya que los lados correspondientes de los triángulos BA′ Q y PAC
son paralelos, tenemos que:
∠BQA′ = ∠PCA = ∠ABP = ∠BAA′
por tanto AQBA′ es cíclico. Luego, ∠BAQ = ∠BA′ Q = ∠PAC.

Notemos que esta propiedad es una generalización de la proposición 5.5: cuando P coincide con el
ortocentro de △ABC, Q es el antípoda de A en (ABC). Como el circuncentro de △ABC yace sobre AQ,
la isogonalidad sigue por el hecho 5.4.
Establezcamos dos resultados adicionales de conjugados√isogonales en triángulos. Las pruebas se
realizan mediante división armónica (primer ítem) e inversión bc (segundo ítem). Estas herramientas las
abordamos en la parte de introducción a la geometría proyectiva. Es posible probarlos con trigonometría.

Proposición 7.3. Sean P y Q puntos tales que AP y AQ son conjugadas isogonales en el triángulo
ABC. Se tiene que:
Si X es la intersección de BQ y CP, y Y el corte de BP y CQ luego AX y AY son también
conjugados isogonales de △ABC.
Si V es la segunda intersección de (BAP) y (CAQ), mientras que W es el segundo corte de
(BAQ) y (CAP), entonces V y W son conjugados isogonales en △ABC.

También es posible contar con conjugados isogonales en cuadriláteros. La proposición a continuación


establece el criterio para que esto ocurra.

Proposición 7.4 (Puntos isogonales en cuadriláteros). Sea ABCD un cuadrilátero convexo y P un


punto en su mismo plano. Luego, P posee un conjugado isogonal respecto a ABCD si y solo si
∠APB + ∠CPD = 180◦ .

Prueba. Consideremos a E = AD ∩ BC. Sean K, L, M y N los pies de altura desde P hacia DA, CD, BC
7.2 Simedianas 69

N
A

K P∗
M

C L B E

y AB, respectivamente. Sea P1 el conjugado isogonal de P respecto a △EAB y P2 el conjugado isogonal


respecto a △ECD.
Obsérvese que el conjugado isogonal de P respecto a ABCD existe si y solo si P1 = P2 . No es difícil
ver que esto ocurre si y solo si los triángulos pedales de P respecto a △EAB y △ECD poseen un mismo
circuncírculo. Por el lema 7.1 esto equivale a probar que KLMN es cíclico, pero notemos que

∠APB + ∠CPD = ∠APM + ∠MPB + ∠CPK + ∠KPD


= ∠ANM + ∠MLB + ∠CLK + ∠KND
= 360◦ − (∠KNM + ∠KLM)

Es decir, el cuadrilátero KLMN tiene un circuncírculo si y solo si ∠APB y ∠CPD son suplementarios.

7.2 Simedianas

Definición 7.1. La simediana correspondiente al vértice A se define como la reflexión de la A-


mediana con respecto a la bisectriz interna del ángulo BAC. En otras palabras, es la recta isogonal
correspondiente a la mediana que parte del mismo vértice de referencia.

B L M C

Figura 7.5. La recta AL es la A-simediana del triángulo ABC.


70 Conjugados isogonales y simedianas

En el caso de la figura 7.5, la recta AL es la A-simediana del △ABC. Es decir, ∠BAL = ∠MAC. No
hace falta decir que todo triángulo contiene tres simedianas. Por el teorema de Ceva en su forma trigo-
nométrica, sabemos que, al concurrir las medianas en el baricentro de △ABC, las simedianas comparten
también un punto en común, llamado el punto de Lemoine del triángulo ABC.

Proposición 7.5. Manteniendo la notación de la figura 7.5, se tiene que


 2
BL AB
=
LC AC

si y solo AL coincide con la A-simediana del △ABC. Es decir, la simediana divide a su lado
opuesto respectivo en una razón equivalente al cuadrado del cociente de los dos lados adyacentes.

Prueba. Es una aplicación directa del teorema de Steiner.

De inmediato, la propiedad 7.5 también garantiza la concurrencia de las simedianas al aplicar el


teorema de Ceva en su versión estándar.
Posiblemente, el lema a continuación constituye el resultado más conocido y de mayor utilidad en
problemas que involucran simedianas.

Lema 7.2. Las tangentes por B y C al circuncírculo del triángulo ABC se intersecan en P. Luego,
AP es la A-simediana de △ABC.

B C
L M

S P T
Figura 7.6. Una forma de construir e identificar la A-simediana.

Prueba. Supongamos que las prolongaciones de los lados AB y AC cortan a la paralela a BC por P en
S y T , respectivamente. Es claro que ∠PSB = ∠CBA y ∠PTC = ∠BCA. Además, ∠PBS = ∠BCA y
∠PCT = ∠CBA por la condición de tangencia de PB y PC. De este modo, △BPS ∼ △CAB ∼ △T PC, lo
que a su vez implica que:
 2  2
SP SP PC AB AS
= · = =
PT PB PT BC AT
por consiguiente, por el hecho 7.5 AP es simediana de △SAT y, de paso, también de △BAC.
7.2 Simedianas 71

Dénotese por d(X,Y Z) la distancia del punto X a la recta Y Z. El lema 7.5 nos sirve para demostrar la
propiedad a continuación.

Proposición 7.6. Sea P un punto en el mismo plano que △ABC. Entonces P está sobre la A-
simediana de △ABC si y solo si:
d(P, AB) AB
=
d(P, AC) AC

B N C

Figura 7.7. Distancias desde un punto arbitrario de la A-simediana.

Demostración. De acuerdo con el segundo resultado y por el teorema de la bisectriz generalizado obte-
nemos que, para un punto N sobre BC, AN es la A-simediana si y solo si:
sin ∠BAN AB
=
sin ∠NAC BC
Es claro que, por el teorema de Tales, para cualquier punto P sobre AN sucede que:
d(P, AB) d(N, AB) sin ∠BAN
= =
d(P, AC) d(N, AC) sin ∠NAC
Desde luego, por transitividad extraemos la conclusión deseada.
Esto indica que podemos definir a la A-simediana como el lugar geométrico de los puntos cuyas
razones de distancias hacia AB y AC coinciden con la razón de las longitudes de estos dos lados. Una
caracterización intuitiva es la siguiente.

Proposición 7.7. La A-simediana es el lugar geométrico de los puntos medios de los segmentos
formados por las antiparalelas al lado BC de un triángulo ABC y sus puntos de intersección con
los otros dos lados.

Demostración. Construyamos puntos D y E sobre AB y AC tales que BDEC sea un cuadrilátero cíclico.
Como es usual, definamos a M como el punto medio de BC, mientras que J es el punto medio de DE.
Como △DAE ∼ △CAB, también debe pasar que △DAJ ∼ △CAM. El resultado sigue.
72 Conjugados isogonales y simedianas

J
D

B M C

Figura 7.8. Una caracterización adicional de la A-simediana.

7.3 Problemas resueltos

Ejemplo 7.1. (Balcánica 2009, P2). Sea MN una recta paralela al lado BC de un triángulo ABC,
con M en el lado AB y N en el lado AC. Las rectas BN y CM se cortan en P. Los circuncírculos
de los triángulos BMP y CNP se cortan en dos puntos distintos P y Q. Demostrar que AQ es la
A-simediana de △ABC.

M N
P

B C

Figura 7.9. Otra caracterización de la A-simediana.

Solución. Notemos que ∠BQM = ∠BPM = ∠CPN = ∠CQN y ∠MBQ = ∠CPQ = ∠CNQ, así que
7.3 Problemas resueltos 73

△BQM ∼ △NQC. De este modo, conseguimos que:


d(Q, AB) d(Q, MB) BM AB
= = =
d(Q, AC) d(Q, NC) CN AC
Por la proposición 7.6, concluimos que AQ es simediana de △ABC.

Ejemplo 7.2. (Lista Corta IMO 2012, G2). Sea ABCD un cuadrilátero cíclico cuyas diagonales AC
y BD se cortan en E. Las extensiones de los lados AD y BC más allá de A y B se cortan en F. Sea G
el punto tal que ECGD es un paralelogramo, y H la reflexión de E con respecto a AD. Demostrar
que D, H, F y G son concíclicos.

Solución. Por supuesto, ∠ADG = ∠AEC = ∠DBF = ∠EBF. Ya que ∠EDA = ∠ECB, por el lema 7.1
reconocemos que FE y FG son isogonales del △DFC, es decir, ∠EFB = ∠DFG. Luego, por criterio
AA concluimos que △EBF ∼ △GDF, así que:

∠FGD + ∠FHD = ∠FEB + ∠FED = 180◦

con lo que concluimos que DHFG es cíclico.

H D
A

E
G

F B C

Figura 7.10. Problema G2 de la Lista Corta de la IMO 2012.

Ejemplo 7.3. (Iberoamericana 2016, P5). Las circunferencias C1 y C2 se cortan en puntos diferen-
tes A y K. La tangente común a C1 y C2 más cercana a K toca a C1 en B y a C2 en C. Sea Q el pie
de altura desde B a AC, y sea P el pie de altura desde C a AB. Si E y F son los simétricos de K con
respecto a las rectas PQ y BC, respectivamente, demostrar que A, E y F son colineales.

Solución. Sea M el corte de AK con BC. Por potencia, MB2 = MK · MA = MC2 ; es decir, M es el punto
medio de BC. Observemos que:

∠BFC = 180◦ − (∠MBK + ∠MCK) = 180◦ − (∠BAK + ∠KAC) = 180◦ − ∠BAC

por consiguiente, ABFC es cíclico y así ∠BAF = ∠BCF = ∠KCB = ∠KAC, de modo que AF es sime-
diana de △ABC.
74 Conjugados isogonales y simedianas

Por otro lado, según el lema 5.1 MP y MQ son tangentes a (APQ), así que por el hecho 7.5 concluimos
que AK es simediana de △PAQ. Además, PM 2 = BM 2 = AM · KM, por lo que K está sobre (PAQ).
También AK es simediana de △PKQ.
Sea N el punto medio de PQ. Por lo anterior obtenemos que:
∠NAQ = ∠KAP = ∠NQK y ∠AQN = ∠AKP = ∠QKN
de donde extraemos que ∠ANQ = ∠QNK, así que E debe estar sobre AN. Por último, como PQ es
antiparalela a BC concluimos por el lema 7.7 que AF pasa por N, y por ende E también está sobre
AF.

E Q

N
K
P

B M C

F
Figura 7.11. Problema 5 de la OIM 2016.

7.4 Problemas propuestos


Problema 7.1. (Teorema del triángulo pedal de Lemoine). Sea K el punto simediano del triángulo ABC.
Demostrar que K es el único punto en el mismo plano de ABC que es baricentro de su propio triángulo
pedal.
Problema 7.2. (Primer círculo de Lemoine). Sean x, y, z las antiparalelas trazadas por K a las rectas
BC, CA y AB, respectivamente. Probar que los seis puntos determinados por x, y, z en los lados de ABC
yacen en una misma circunferencia, llamada el primer círculo de Lemoine de △ABC.
Problema 7.3. (Segundo círculo de Lemoine). Esta vez, sean x, y, z las paralelas trazadas por K a las
rectas BC, CA y AB. Mostrar que los seis puntos determinados por x, y, z en los lados de ABC yacen en
un mismo círculo, llamado el segundo círculo de Lemoine de △ABC.
Problema 7.4. (Balcánica 2022, P1). Sea ABC un triángulo acutángulo tal que CA ̸= CB con circuncírculo
ω y circuncentro O. Sean tA y tB las tangentes a ω por A y B respectivamente, que se cortan en X. Sea
Y el pie de altura de la perpendicular desde O al segmento CX. La paralela por C a AB corta a tA en Z.
Probar que la recta Y Z pasa por el punto medio del segmento AC.
7.4 Problemas propuestos 75

Problema 7.5. (Iberoamericana 2013, P2). Sean X, Y los extremos de un diámetro de una circunferencia
Γ y N el punto medio de uno de los arcos XY de Γ. Sean A y B dos puntos en el segmento XY . Las rectas
NA y NB cortan nuevamente a Γ en los puntos C y D, respectivamente. Las tangentes a Γ en C y D se
cortan en P. Sea M el punto de intersección del segmento XY con el segmento NP. Demostrar que M es
el punto medio del segmento AB.

Problema 7.6. (Centroamericana 2016, P2). Sea ABC un triángulo acutángulo, Γ su circuncírculo y M
el punto medio de BC. Sea N el punto en el arco BC de Γ que no contiene a A tal que ∠NAC = ∠BAM.
Sea R el punto medio de AM, S el punto medio de AN y T el pie de altura desde A. Probar que R, S y T
están alineados.

Problema 7.7. (Europea Femenil 2016, P2). Sea ABCD un cuadrilátero cíclico, y X la intersección de
las diagonales AC y BD. Sean C1 , D1 y M los puntos medios de los segmentos CX, DX y CD, respecti-
vamente. Las rectas AD1 y BC1 se intersecan en Y , la recta MY interseca a las diagonales AC y BD en
dos puntos distintos, que llamamos respectivamente E y F. Demostrar que la recta XY es tangente a la
circunferencia que pasa por E, F y X.

Problema 7.8. Sea D el punto de Anti-Steiner de la recta de Euler del triángulo ABC. Definamos a E
como el pie de la perpendicular desde A a la recta de Euler del triángulo. Demostrar que AD y AE son
conjugadas isogonales del △ABC.

Problema 7.9. (Bulgaria 2011, P4). El punto O está en el interior de ABC. Los pies de las perpendiculares
desde O a BC, CA y AB son D, E y F, respectivamente. Las perpendicualres desde A y B hacia EF y FD
se cortan en P. Sea H el pie de altura desde P a AB. Demostrar que D, E, F y H son concíclicos.

Problema 7.10. (ELMO 2012, P5). Sea ABC un triángulo actuángulo con AB < AC, y sean D y E puntos
en el lado BC tal que BD = CE y D yace entre B y E. Suponga que existe un punto P en el interior de
ABC tal que PD ∥ AE y ∠PAB = ∠EAC. Probar que ∠PBA = ∠PCA

Problema 7.11. ([Link]. 2010, Selectivo IMO, P7). En el triángulo ABC, sean P y Q puntos en su
interior tales que ∠ABP = ∠QBC y ∠ACP = ∠QCB. El punto D está sobre el segmento BC. Probar que
∠APB + ∠DPC = 180◦ si y solo si ∠AQC + ∠DQB = 180◦ .

Problema 7.12. (IMO 2018, P6) Un cuadrilátero convexo ABCD satisface AB ·CD = BC · DA. El punto X
en el interior de ABCD es tal que ∠XAB = ∠XCD y ∠XBC = ∠XDA Demostrar que ∠BXA + ∠DXC =
180◦ .

Problema 7.13. (Lista Corta Iberoamericana 2017). Sea ABC un triángulo acutángulo con AC > AB,
circuncírculo Γ y M punto medio del lado BC. Se escoge un punto N interior a ABC, de modo que si D y
E son sus pies de alturas a AB y AC, respectivamente, entonces DE ⊥ AM. El circuncírculo de ADE corta
a Γ en L (L ̸= A) y las rectas AL y DE se cortan en K. La recta AN corta a Γ en F (F ̸= A). Demostrar
que si N es el punto medio del segmento AF entonces KA = KF.
76 Conjugados isogonales y simedianas
CAPÍTULO 8

SEMEJANZA ESPIRAL

8.1 ¿Qué es la semejanza espiral?


Una clase particular de semejanza que se ha convertido en un método frecuente para atacar problemas
es la semejanza espiral, definida como sigue.

Definición 8.1. Una semejanza espiral o rotohomotecia con centro O es una composición de ho-
motecia y rotación respecto a un punto O.

Abordaremos con mayor detalle el concepto de homotecia en la parte de concurrencia y colinealidad.


En términos generales, la homotecia es un “reescalamiento” de una figura geométrica por un factor k ̸= 0.
Cuando k = 1, la semejanza espiral coincide con una rotación pura. Más que polígonos en general, la
semejanza espiral aplicada para segmentos es el caso más común en problemas de olimpiada.

B C
B′
A′
C′

A
O

Figura 8.1. Una semejanza espiral centrada en O que manda △ABC a △A′ B′C′ .

8.2 Propiedades

Lema 8.1 (Unicidad de la semejanza espiral). Dados cuatro puntos distintos A, B, C y D en el


plano tales que el cuadrilátero ABCD no es un paralelogramo, existe una y solo una semejanza
espiral que manda el segmento AB al segmento CD.

A C
D

O
78 Semejanza espiral

Demostración. Sean a, b, c, d los números complejos correspondientes a los puntos A, B, C, D. Una


semejanza espiral tiene la forma T (x) = x0 + α(x − x0 ), donde |α| es la razón de homotecia y arg α el
ángulo de rotación. Luego, deseamos encontrar x0 y α tales que:
T (a) = x0 + α(a − x0 ) = c; T (b) = x0 + α(b − x0 ) = d
c−d ad−bc
Resolviendo el sistema de ecuaciones anterior, podemos deducir que α = d−b y x0 = a+d−b−c . Ya que
ABCD no es paralelogramo y A, B,C, D son todos distintos, entonces a + d ̸= b + c y b ̸= d por lo que x0
y α están siempre bien definidos. Dado que obtuvimos una única solución al sistema original, entonces
existe una única semejanza espiral que manda el segmento AB al segmento CD.
Si bien el resultado anterior nos asegura la existencia de un único centro de semejanza espiral para
puntos que no forman un paralelogramo, aún no conocemos cómo construir o identificar tal punto. La
siguiente propiedad resuelve este problema.

Lema 8.2 (Excesivamente útil). Si A, B, C y D son cuatro puntos distintos en el plano tales que
AC ∦ BD, X = AC ∩ BD y los circuncírculos de △ABX y △CDX se cortan por segunda vez en O,
entonces O es el centro de semejanza espiral que manda AB a CD.

X
B
C X C

A D

D
O
A
B
O
Figura 8.2. Dos configuraciones posibles de semejanza espiral.

Demostración. Notemos que hay varias posibles configuraciones de la semejanza espiral, por lo que
utilizaremos ángulos dirigidos. En efecto, vemos que:
∡BOA = ∡BXA = ∡DXC = ∡DOC; ∡OBA = ∡OXA = ∡ODC
Por consiguiente, los triángulos AOB y COD son semejantes en la misma orientación.
De forma análoga, podemos probar que △AOC y △BOD son directamente similares, por lo que:

Lema 8.3 (La semejanza espiral viene en pares). Si O es el centro de semejanza espiral que manda
el segmento AB al segmento CD, entonces también es el centro de semejanza espiral que manda el
segmento AC al segmento BD.

Vale mencionar que si X e Y son puntos correspondientes en los triángulos AOB y COD, tendríamos
que △AOX ∼ △COY y △XOB ∼ Y OD, por ende, O es el centro de semejanza espiral que manda el
segmento AO a CO y el segmento BO a DO.
8.3 Un centro de semejanza espiral especial 79

8.3 Un centro de semejanza espiral especial


Centremos nuestro interés en el siguiente resultado:

Lema 8.4. Sea ABC un triángulo y D el segundo punto de intersección de la A−simediana con
su circuncírculo. Luego M, el punto medio de la cuerda AD, es el centro de semejanza espiral que
manda el lado AB al lado AC.

B C

Figura 8.3. Centro de semejanza espiral de BA y AC.

Demostración. Ya que AD es simediana del △ABC, entonces BC es simediana de △ABD y △ACD, por
tanto, ∠MBA = ∠CBD = ∠CAD = ∠CAM y ∠ACM = ∠BCD = ∠BAD = ∠BAM, por ende △BMA y
△AMC son semejantes en la misma orientación y se obtiene el resultado.

Como AD también es simediana del triángulo BDC, fácilmente se puede demostrar que M manda el
segmento BD a DC.

8.4 Problemas resueltos

Ejemplo 8.1. (Lista Corta IMO 2006, G3). Sea ABCDE un pentágono convexo tal que:

∠BAC = ∠CAD = ∠DAE; ∠ABC = ∠ACD = ∠ADE

Las diagonales BD y CE se cortan en P. Probar que la recta AP biseca el lado CD.

Solución. Ya que ∠CBA = ∠EDA y ∠CAB = ∠EAD, entonces △CAB y △EAD son semejantes con una
misma orientación, por lo que A debe ser el centro de semejanza espiral que manda BC a DE. Por el lema
8.2, inferimos que P es el segundo punto de intersección de los circuncírculos de ABC y ADE.
Notemos que ∠PCD = ∠ACD − ∠ACP = ∠ABD − ∠ABP = ∠PBC, luego, CD es tangente al cir-
cuncírculo de △ABC. Similarmente podemos probar que CD es tangente a circuncírculo del △ADE. Si
M = AP ∩CD, obtenemos que MC2 = MP · MA = MD2 , por ende MC = MD.
80 Semejanza espiral

C M
D
B P

Figura 8.4. Problema G3 de la Lista Corta de la IMO 2006.

Ejemplo 8.2. (EE. UU. 2008, Selectivo IMO, P7). Sea ABC un triángulo con baricentro G. P es
un punto variable sobre el segmento BC. Los puntos Q y R yacen en los lados AC y AB, respec-
tivamente, de modo que PQ ∥ AB y PR ∥ AC. Demostrar que, a medida que P varía a lo largo del
segmento BC, el circuncírculo del △AQR pasa por un punto fijo X tal que ∠BAG = ∠CAX.

Solución. Sea X ′ el punto medio de la A-cuerda simediana; claramente, ∠BAG = ∠CAX ′ . De acuerdo al
lema 8.4, X ′ envía BA a AC.
CQ
Por el teorema de Tales, AR CP
RB = PB = QA , es decir, R y Q son puntos correspondientes en BA y AC;
luego, X ′ también traslada AR a CQ, lo que significa que △X ′ RA ∼ △X ′ QC, por ende ∠X ′ RA = ∠X ′ QC
y se concluye que X ′ yace sobre el círculo (AQR). Ya que X ′ es independiente de la posición de P, se
deduce que este es el punto fijo deseado; es decir, X ≡ X ′ .

R
Q
X′
G

B P C
Figura 8.5. Problema 7 del Selectivo IMO 2008 de [Link].
8.5 Problemas propuestos 81

Ejemplo 8.3. (Irán 2017, Tercer Selectivo IMO, P6). En el triángulo ABC sean O y H su circun-
centro y ortocentro, respectivamente. El punto P es la reflexión de A con respecto a OH. Asuma
que P no está en el mismo lado de BC que A. Los puntos E, F yacen sobre AB, AC respectivamente
tales que BE = PC ,CF = PB. Sea K la intersección de AP y OH. Probar que ∠EKF = 90◦ .

Solución. Evidentemente, P yace sobre (ABC). Sean M y N los puntos medios de BC y EF, y D el
simétrico de P con respecto a M. Como BHDC es cíclico según la proposición 5.3, conseguimos:
∠OHC = ∠HAK + ∠HCB = ∠BAP = ∠BCP = ∠DBC = ∠DHC
por tanto, D yace sobre OH.

Q
F
N
E O
K
H D

B M C

Figura 8.6. Solución de un problema iraní con semejanza espiral.


Además, los triángulos EBD y FCD son isósceles en B y C. Ya que ∠EBD + ∠FCD = ∠ABC +
∠ACB − (∠DBC + ∠DCB) = 180◦ + 2∠BAC, entonces ∠EDB + ∠FDC = 90◦ − ∠BAC y podemos con-
cluir que ∠EDF = 90◦ .
Sea Q = PD ∩ (BAC), Q ̸= P. Observemos que:
QB BM MC QC
= = =
CP MP MP PB
por ende, QB : QC = PC : PB = BE : CF. Junto a ∠QBE = ∠QCF, esto implica que △QBE ∼ △QCF;
es decir, Q es el centro de semejanza espiral que manda EB a FC y EF a BC según el lema 8.3, por tanto
Q yace sobre (AEF) también.
Como M y N son puntos correspondientes en △QEF y △QBC, Q también intercambia EB y FC con
NM, así que ∠QMN = ∠QBE = ∠QPA, lo que indica que MN ∥ PA y a su vez MN ⊥ OH. Siendo M y
N los circuncentros de △KPD y △EDF, podemos inferir que MN es la mediatriz del segmento DK, por
tanto EFKD es cíclico y ∠EKF = 90◦ .

8.5 Problemas propuestos


Problema 8.1. (Iberoamericana 2018, P2) Sea ABC un triángulo tal que ∠BAC = 90◦ y BA = CA. Sea
M el punto medio de BC. Un punto D ̸= A es elegido en la semicircunferencia de diámetro BC que
82 Semejanza espiral

contiene a A. La circunferencia circunscrita a △DAM interseca a las rectas DB y DC en los puntos E y


F, respectivamente. Demostrar que BE = CF.
Problema 8.2. (Cuenca del Pacífico 2015, P1). Sea ABC un triángulo y sea D un punto sobre el lado BC.
Una recta que pasa por D corta al lado AB en X y a la semirrecta AC en Y . La circunferencia circunscrita
del triángulo BXD interseca a la circunferencia circunscrita ω del triángulo ABC nuevamente en el punto
Z ̸= B. Las rectas ZD y ZY vuelven a cortar a ω en V y W , respectivamente. Probar que AB = VW .
Problema 8.3. (Lista Corta Iberoamericana 2016, G3). Las circunferencias Φ1 y Φ2 se intersecan en dos
puntos diferentes A y B. La tangente a Φ1 por A intersecta a Φ2 en M y la tangente a Γ2 por A que corta
a Φ1 en N. Sea P la reflexión de A con respecto a B. Sean S y T los puntos de intersección de las rectas
PM y PN con Φ1 y Φ2 , respectivamente. Demostrar que los puntos S, B, T son colineales.
Problema 8.4. (Europea Femenil 2022, P1). Sea ABC un triángulo acutángulo con BC < AB y BC < AC.
Considere los puntos P y Q en los segmentos AB y AC, respectivamente, tales que P ̸= B, Q ̸= C y
BQ = BC = CP. Sea T el circuncentro del triángulo APQ, H el ortocentro del triángulo ABC y S el punto
de intersección de las rectas BQ y CP. Pruebe que los puntos T , H y S están en una misma recta.
Problema 8.5. (IMO 2017, P4). Sean R y S puntos distintos sobre la circunferencia Ω tales que RS
no es un diámetro de Ω. Sea l la recta tangente a Ω en R. El punto T es tal que S es el punto medio
del segmento RT . El punto J se elige en el menor arco RS de Ω de manera que Γ, la circunferencia
circunscrita al triángulo JST , intersecta a l en dos puntos distintos. Sea A el punto común de Γ y l más
cercano a R. La recta AJ corta por segunda vez a Ω en K. Demostrar que la recta KT es tangente a Ω.
Problema 8.6. (Lista Corta IMO 2015, G3). Sea ABC un triángulo con ∠C = 90◦ , y sea H el pie de
altura desde C. Un punto D es escogido dentro del triángulo CBH tal que CH biseca AD. Sea P el punto
de intersección de las rectas BD y CH. Sea ω la semicircunferencia con diámetro BD que corta a CB en
un punto interno. Una recta por P es tangente a ω en Q. Probar que CQ y AD se cortan en ω.
Problema 8.7. (Irán 2016, Tercera Ronda, G2). Sea ABC un triángulo ABC arbitrario. Sean E y F dos
puntos sobre los lados AB y AC, respectivamente, tal que sus distancias al punto medio de BC son iguales.
Los circuncírculos de ABC y AEF se cortan en otro punto P. Las tangentes por E y F al circuncírculo de
AEF se cortan en K. Probar que ∠KPA = 90◦ .
Problema 8.8. (Lista Corta IMO 2015, G3). Sea ABC un triángulo con ∠C = 90◦ , y sea H el pie de
altura desde C. Un punto D es escogido dentro del triángulo CBH tal que CH biseca AD. Sea P el punto
de intersección de las rectas BD y CH. Sea ω la semicircunferencia con diámetro BD que corta a CB en
un punto interno. Una recta por P es tangente a ω en Q. Probar que CQ y AD se cortan en ω.
Problema 8.9. ([Link]. 2008, P2). Sea ABC un triángulo acutángulo y escaleno. Sean M, N, P los
puntos medios de BC,CA, AB respectivamente. Las mediatrices de AB y AC cortan al rayo AM en D y E,
respectivamente, y las rectas BD y CE se cortan en F, dentro del triángulo ABC. Probar que A, N, F, P
yacen sobre una misma circunferencia.
Problema 8.10. (Lista Corta IMO 2006, G9). Los puntos A1 , B1 ,C1 son escogidos en los lados BC,CA, AB
de un triángulo ABC, respectivamente. Los circuncírculos de los triángulos AB1C1 , BC1 A1 , CA1 B1 interse-
can el circuncírculo del triángulo ABC nuevamente en A2 , B2 ,C2 , respectivamente. Los puntos A3 , B3 ,C3
son simétricos a A1 , B1 ,C1 con respecto a los puntos medios de los lados BC,CA, AB, respectivamente.
Demostrar que los triángulos A2 B2C2 y A3 B3C3 son semejantes.
CAPÍTULO 9

RECTA DE SIMSON

9.1 Configuración principal y hechos relacionados


Consideremos un triángulo ABC con circuncírculo Γ. Sea P un punto arbitrario. Supongamos que las
perpendiculares desde P hacia los lados BC, CA y AB cortan a estos segmentos en D, E y F, respectiva-
mente. Entonces, se tiene que:

Teorema 9.1 (Teorema de Simson). Los puntos D, E y F están alineados si y solo si P yace sobre
Γ. La recta que los contiene recibe el nombre de recta de Simson de P con respecto al triángulo
ABC (o simplemente P-recta de Simson del △ABC).

F R
F
A A
P P

E E

H
Q′

B D C B D C

(a) Recta de Simson de P. (b) La P-recta de Simson biseca a HP.

Demostración. Notemos que los cuadriláteros BDPF y CEDP son cíclicos. Observemos que D, E y F
son colineales si y solo si

∠ABP = ∠FBP = ∠FDP = ∠EDP = ∠ECP = ∠ACP

es decir, si y solo si el cuadrilátero ABPC es cíclico, como queríamos demostrar.

Como dato interesante, P es el centro de semejanza espiral que envía FB a EC. ¿Puedes notar por
qué? De hecho, es posible llegar a una solución mediante la técnica de semejanza espiral.
84 Recta de Simson

Por otro lado, el teorema de Simson implica básicamente que el triángulo pedal de cualquier punto
sobre el circuncírculo de ABC es degenerado.
Examinemos dos resultados adicionales relacionados al teorema de Simson.

Proposición 9.1. Si H es el ortocentro de △ABC, la recta de Simson de P biseca al segmento HP.

Demostración. Sea R el punto de intersección de AH con DF y Q el segundo punto de intersección de


PD con Γ (véase la lámina 9.1b).
Es claro que AR ∥ DQ. Además, ∠EDP = ∠ECP = ∠AQP, lo que implica que AQ ∥ RD, así que
ARDQ es un paralelogramo.
Sea Q′ la reflexión de Q respecto al lado BC. Al ser Q la segunda intersección de la perpendicular
por P a BC con Γ, sabemos que Q′ coincide con el ortocentro de △BPC. Además, observemos que
PQ′ = AH, ya que ambos equivalen a la distancia del circuncentro de △ABC al punto medio del lado BC.
Como también pasa que AH ∥ PQ′ , podemos concluir que AHQ′ P también es un paralelogramo.
En resumen, inferimos que RH = RA + AH = DQ + PQ′ = DQ′ + PQ′ = PD, lo que junto a RH ∥ PD
conduce a que RHDP es un paralelogramo. De aquí que la recta DF biseca al segmento HP.

Proposición 9.2. Sean ℓP y ℓQ las rectas de Simson de dos puntos arbitrarios P y Q sobre el
circuncírculo del triángulo ABC. Luego, el ángulo formado por ℓP y ℓQ equivale a la mitad del
arco PQ.
c

F′
Q
A

E′

E
R

B D D′ C

Figura 9.2. Una propiedad interesante acerca del ángulo entre dos rectas de Simson.

Demostración. Consideremos a D′ , E ′ , F ′ como las proyecciones de Q sobre los lados BC, CA y AB,
9.2 Problemas resueltos 85

respectivamente. Sea R la intersección de DF y D′ F ′ . Observemos que en el cuadrilátero RECD′ sucede:

∠D′ RE = 360◦ − ∠RD′C − ∠BCA − ∠CER


= 180◦ − ∠BCA − ∠RD′ Q − ∠REP
= 180◦ − ∠BCA − ∠ACQ − ∠BAP
PAQ PCQ
= 180◦ −
d d
=
2 2
El resultado sigue.

En particular, la proposición 9.2 implica que cuando P y Q son antípodas (diametralmente opuestos)
en el circuncírculo de ABC, entonces sus rectas de Simson con respecto a △ABC son perpendiculares.

9.2 Problemas resueltos

Ejemplo 9.1. (Hong Kong, 1998) Sea PQRS un cuadrilátero cíclico con ∠PSR = 90◦ y sean H y
K los pies de alturas desde Q a PR y PS. Probar que HK biseca a QS.

P H R

Solución. Notemos que HK es la Q-recta de Simson del triángulo PSR. Como S es el ortocentro de
△PSR, por la proposición 9.1 concluimos que HK pasa por el punto medio de QS.

Ejemplo 9.2. (EE. UU., 2014) Sea ABC un triángulo acutángulo y X un punto variable sobre el
arco menor BC.
c Sean P y Q los pies de las perpendiculares desde X hacia los lados CA y CB,
respectivamente. Sea R la intersección de la recta PQ y la perpendicular por B a AC. Sea ℓ la recta
por P paralela a XR. Probar que a medida que X varía sobre el arco menor BC,c la recta ℓ siempre
pasa por un punto fijo.

Solución. Es claro que PQ es la X-recta de Simson de △ABC. Demostraremos que el ortocentro H de


△ABC es el punto fijo requerido.
86 Recta de Simson


H P

Q
B C

R X

Recordemos que, de acuerdo con la proposición 9.1, PQ biseca al segmento HX. Además, por cons-
trucción de R, sabemos que XP ∥ HR. De este modo, por el teorema de Tales HX también biseca a PR,
por lo que podemos concluir que XPHR es un paralelogramo. Esto conlleva a concluir que H yace sobre
ℓ.
Como H no depende de la elección de X, deducimos que es el punto fijo sobre ℓ que deseábamos.

Ejemplo 9.3. (EGMO 2021, P4) Sea ABC un triángulo con incentro I y sea D un punto arbitrario
en el lado BC. La recta que pasa por D y es perpendicular a BI interseca a CI en el punto E. La recta
que pasa por D y es perpendicular a CI interseca a BI en el punto F. Demuestre que la reflexión de
A sobre la recta EF está en la recta BC.

F
A

P Q R

I E

P′ B D Q′ C R′

Solución. Observemos que:


∠BAC + ∠ABC ∠BCA
∠AIF = = 90◦ −
2 2
9.3 Problemas propuestos 87

así que ∠AIB = ∠FDB. Junto a ∠ABI = ∠FBD, la igualdad anterior implica que △ABI ∼ △FBD. Siendo
así, B es el centro de semejanza espiral que manda AI a FD, por lo que también transforma a AF en BD
y los triángulos ABF e IBD resultan ser semejantes también. Similarmente, △ACE ∼ △ICD. Luego,

∠AEI = 180◦ − ∠AEC = 180◦ − ∠IDC = ∠IDB = ∠AFI

por lo que el cuadrilátero AIEF es cíclico.


Finalmente, consideremos las proyecciones de A sobre las rectas IE, IF y FE, digamos P, Q y R.
Al ser IB e IC bisectrices internas de △AC, los simétricos de A respecto a P y Q yacen sobre BC. Por el
teorema de Simson, sabemos que P, Q y R son colineales. Siendo así, el simétrico de A con respecto a R
también va a estar sobre BC, como requeríamos.

9.3 Problemas propuestos


Problema 9.1. (Macedonia 2018, P4). El triángulo acutángulo ABC tiene ortocentro H. El punto H ′ es
el simétrico de H con respecto a la recta AB. El círculo que pasa por A, N y H ′ corta por segunda vez a
AC en M, mientras que el círculo que pasa por B, N y H ′ corta a BC por segunda vez en P. Probar que
M, N y P están alineados.

Problema 9.2. (Sharygin 2017, Ronda Final, Grado 9, P5). El centro de un círculo ω2 yace sobre un
círculo ω1 . Las tangentes XP y XQ hacia ω2 desde un punto arbitrario X sobre ω1 (P y Q son los puntos
de contacto) cortan a ω1 por segunda vez en R y S, respectivamente. Probar que la recta PQ biseca al
segmento RS.

Problema 9.3. (Sharygin 2014, Ronda de Correspondencia, P8). Dado un rectángulo ABCD, dos rectas
perpendiculares entre ellas tienen al vértic B como punto de intersección. Una de ellas corta al segmento
AD en K, y la segunda corta a la prolongación del lado CD en L. Sea F la intersección de KL y AC.
Demostrar que BF ⊥ KL

Problema 9.4. (Reino Unido 2015, P5). Sea ABC un triángulo, y sean D, E y F los pies de las per-
pendiculares desde A, B y C hacia BC, CA y AB, respectivamente. Sean P, Q, R y S los pies de las
perpendiculares desde D a BA, BE, CF y CA, respectivamente. Probar que P, Q, R y S son colineales.

Problema 9.5. (Resultado útil). Sea ABC un triángulo. Sean D, E y F los puntos de tangencia del
incírculo con los lados BC, CA y AB, respectivamente. Probar que la A-mediana de △ABC, EF y el
diámetro del incírculo de △ABC que pasa por D concurren.

Problema 9.6. Sea ABCD un cuadrilátero cíclico. Probar que la intersección de la A-recta de Simson del
△BCD con la B-recta de Simson del △ACD es colineal con C y el ortocentro de △ABD.

Problema 9.7. (China 2022, Selectivo IMO, P2). Dado un triángulo no rectángulo ABC con BC > AC >
AB, dos puntos P1 ̸= P2 en el plano satisfacen que, para i = 1, 2, si APi , BPi y CPi cortan al circuncírculo
del triángulo ABC en Di , Ei , y Fi , respectivamente, entonces Di Ei ⊥ Di Fi y Di Ei = Di Fi ̸= 0. La recta P1 P2
interseca al circuncírculo de ABC en Q1 y Q2 . Las rectas de Simson de Q1 , Q2 con respecto a ABC se
cortan en W . Probar que W yace sobre el círculo de los nueve puntos de ABC.
88 Recta de Simson
CAPÍTULO 10

INCÍRCULOS MIXTILÍNEOS

10.1 Preliminares

Definición 10.1. Sea ABC un triángulo. El A-incírculo mixtilíneo (ωA ) es el círculo tangente inter-
namente al circuncírculo del △ABC y tangente a los lados AB y CA.

A A

B C B C
T1
R1
TA R2
T2
Figura 10.1. El incírculo mixtilíneo correspondiente al vértice A.
Naturalmente, también existen incírculos mixtilíneos correspondientes a B (ωB ) y C (ωC ). Nótese el
uso del artículo “el”. En efecto, no hay más que un incírculo mixtilíneo para cada vértice de △ABC.
La intuición acerca de la existencia de estos círculos va por estas líneas. En el caso del A-incírculo
mixtilíneo, es claro que su centro OA debe estar sobre la bisectriz interna de ∠BAC. Así, podemos mover
OA a lo largo de esta recta de forma que la distancia entre los puntos de intersección del círculo centrado
en OA y tangente a los segmentos AB, AC sea cada vez menor, como lo muestra el panel derecho de la
figura 10.1. Eventualmente, obtendremos un círculo que es también tangente a (BAC). Más importante
aún, solo existe un punto OA que satisfaga esto. Para cualquier otro punto sobre la bisectriz de ∠BAC
diferente de OA , el círculo trazado tendrá dos puntos de intersección con (BAC) o ninguno. De manera
más formal, podemos construir el A-incírculo mixtilíneo de la siguiente forma:

Trazar la perpendicular ℓ a la recta AI por I. Esta recta corta a los segmentos AB y AC en D y E,


respectivamente. D y E son los puntos de tangencia del A-incírculo mixtilíneo con AB y AC.
Dibujar la perpendicular a AB por D o a AC por E y considerar su corte con AI, digamos OA . Este
punto es el centro del A-incírculo mixtilíneo.
90 Incírculos mixtilíneos

La siguiente sección permitirá cerciorarnos que, en efecto, esta construcción funciona. Pasaremos a
indagar los resultados más importantes relacionados a la configuración en cuestión.

10.2 Propiedades útiles


Consideremos el siguiente hecho.

Lema 10.1. Sean Ω y ω dos circunferencias tangentes en D, de modo que ω se encuentra en el


interior de Ω. La cuerda AB de Ω es tangente a ω en C. Entonces, DC biseca el arco AB.
c

D A C B

Figura 10.2. Un círculo tangente internamente a otro implica punto medio del arco.

Demostración. Sea D el corte de AB y la tangente común por T a ω y Ω. Es claro que △T DC es isósceles


en D. Notemos que:
∠MCB = ∠DCT = ∠DTC = ∠DT M = ∠MBT
lo que indica que MB es tangente al circuncírculo de △BCT . Así, ∠MT B = ∠CBM = ∠ABM = ∠AT M.
El resultado sigue.

Existe una prueba instructiva de este resultado mediante homotecia en circunferencias. En efecto,
observa el problema 13.4.
Definamos a D y E como los puntos de contacto del A-incírculo mixtilíneo con los lados AB y AC,
respectivamente. Sea I el incentro de △ABC. Entonces sucede que:

Lema 10.2. El incentro I yace sobre DE. Es más, I es el punto medio del segmento DE.

Demostración. Si I es colineal con D y E, es evidente que I coincide con el punto medio de DE y que
DE ⊥ AI, pues △DAE es isósceles en A. Por el lema 10.1, sabemos que T D y T E bisecan a los arcos AB
c
y AC
c que no contienen a T en puntos X e Y , respectivamente.
Recordemos que ∠BIC = 90◦ + ∠BAC/2. Sea I ′ el punto medio de DE. Por el lema 7.2 reconocemos
que TA es simediana de △DT E. Así:

∠BT I ′ = ∠BT D + ∠DT I ′ = ∠DTA + ∠AT E = ∠DT E = ∠ADE


10.2 Propiedades útiles 91

por tanto, BT I ′ D es cíclico. Similarmente podemos demostrar que CT I ′ E también lo es. En este caso:
∠BAC
∠BI ′C = ∠BI ′ T + ∠T I ′C = ∠BDT + ∠CET = ∠DET + ∠EDT = ∠DT E = 90◦ +
2
lo que fuerza a que I ′ = I. El resultado sigue.
El teorema de Pascal simplifica considerablemente la prueba anterior. Una vez que se reconoce que
T D y T E bisecan a los arcos AB
c y AC c que no contienen a T , es cuestión de aplicar tal resultado al
hexágono T XCABY para conseguir la colinealidad deseada.
M
A
Y

X
E

D I

B C

Figura 10.3. El incentro es punto medio de DE y la recta T I biseca a BAC.


d

Igual de importante es que:

Lema 10.3. El punto de tangencia del A-incírculo mixtilíneo, el incentro de ABC y el punto medio
del arco BAC
d están alineados; es decir, la recta T I biseca al arco BAC.
d

Demostración. Según la demostración del lema 10.2, BDIT y CEIT son cíclicos. Por consiguiente:
∠IT B = ∠IDA = ∠IEA = ∠CT I
lo que indica que T I es bisectriz de ∠BTC y por consiguiente pasa por el punto medio de BAC.
d

Esta construcción aparece una y otra vez en olimpiadas: el enunciado proporciona el punto medio
d y el incentro de △ABC, y casualmente definen el segundo punto de intersección de la recta
de BAC
formada por estos dos puntos con (ABC). De antemano, ya sabemos que este es el punto de tangencia del
A-incírculo mixtilíneo y (ABC). ¡No dejes que el problema te atrape por no recordar este resultado!
Antes de pasar a algunos problemas de ejemplo, abordemos dos propiedades más.

Proposición 10.1. Sea F el punto de tangencia del incírculo de △ABC con el lado BC. Luego, TA
y T F son conjugadas isogonales con respecto al △BTC. Es más, si F es el punto de contacto del
A-excírculo de △ABC con BC, también AT y AG son conjugadas isogonales, esta vez respecto a
△ABC.
92 Incírculos mixtilíneos

M
A

B F L G C

K
Figura 10.4. Dos pares de conjugadas isogonales relacionadas al A-incírculo mixtilíneo.

Demostración. Definamos a K, L y M como los puntos medios de BTC, d BC y BAC,d respectivamente.


Por el teorema del cateto en el triángulo rectángulo en △KCM y la proposición 6.2 sabemos que KI 2 =
KL · KM, así que KI es tangente a (MLI) y concluimos que:

∠T IK = ∠T IF + ∠FIK = ∠IMK + ∠FIK = ∠KIL + ∠FIK = ∠FIL

Como ∠IT K = ∠IFL = 90◦ , por criterio AA deducimos que △FIL ∼ △T IK. Esto es, I es el centro de
semejanza espiral que manda FL a T K. Por el lema 8.3, reconocemos que I también envía FT a LK con
△IFT ∼ △ILK. Por consiguiente:

∠IT F = ∠IKL = ∠AKM = ∠AT I

donde hemos utilizado el hecho 10.3. Como T I también es bisectriz de ∠BTC, inferimos que ∠BTA =
∠FTC.
La igualdad anterior implica rápidamente que △BT F ∼ △ATC. Usando el hecho de que BF = GC,
es sencillo obtener que △BAT ∼ △GAC, de donde surge que ∠BAT = ∠GAC. Esto completa la demos-
tración.

10.3 Problemas resueltos

Ejemplo 10.1. (EGMO 2013, P5). Sea Ω el circuncírculo de △ABC. El círculo ω es tangente a
los lados AC y BC, y es internamente tangente al círculo Ω en P. Una recta paralela a AB que pasa
por el interior de △ABC es tangente a ω en Q. Probar que ∠ACP = ∠QCB.

Solución. Sean D y E los puntos de contacto de ω con CA y CB, respectivamente. Por construcción, ω
corresponde al C-incírculo mixtilíneo, así que de antemano sabemos que el incentro de △ABC (digamos
I) es el punto medio del segmento DE.
10.3 Problemas resueltos 93

Por el lema 10.1, sabemos que PQ biseca al arco determinado por la tangente a ω por Q y (ABC).
Como esta tangente es paralela a AB, también pasa por el punto medio de BAC
d y según el hecho 10.3
concluimos que I está sobre PQ.
Ya que CP es simediana de △DPE y △DQE (¿por qué?), obtenemos que:

∠QEC = ∠QPE = ∠IPE = ∠DPC y ∠CQE = 180◦ − ∠DQP = 180◦ − ∠ADP = ∠CDP

por tanto △CDP ∼ △CQE y conseguimos que ∠ACP = ∠DCP = ∠QCE = ∠QCB.

Q
E
I
D

A B

Figura 10.5. Problema 5 de la EGMO 2013.

Ejemplo 10.2. (Sharygin 2021, Ronda de Correspondencia, P13). En el triángulo ABC con cir-
cuncírculo Ω e incentro I, el punto M biseca al arco BAC y la recta AI corta a Ω en N ̸= A. El
excírculo opuesto a A toca al lado BC en E. El punto Q ̸= I sobre el circuncírculo de △MIN es tal
que QI ∥ BC. Probar que las rectas AE y QN se cortan en Ω.

Solución. Sea P = AE ∩ Ω, P ̸= A y Q′ = NP ∩ (MIN), Q′ ̸= N. Probaremos que Q = Q′ , para lo que es


suficiente concluir que IQ′ ∥ BC.
Sabemos que MI corta por segunda vez a Ω en el punto de tangencia del A-incírculo mixtilíneo
con Ω (lema 10.3), digamos T , y que ∠BAT = ∠EAC = ∠PAC (proposición 10.1). De este modo, si
R = BC ∩ MI, obtenemos que:

∠Q′ IM = ∠Q′ NM = ∠MNP = ∠MNC + ∠CNP = ∠MBC + ∠CAP = ∠MBR + ∠BMR = ∠CRM

por tanto IQ′ ∥ BC y Q′ coincide con la definición original de Q, es decir Q = Q′ .

Ejemplo 10.3. (Centroamericana 2016, P6). Sea ABC un triángulo con incentro I y circuncírculo
Γ. Sean M y N los puntos de intersección de las rectas BI y CI con Γ. La paralela a MN por I corta
a AB, AC en P y Q. Demostrar que los circunradios de △BNP y △CMQ son iguales.

Solución. Por la proposición 6.2, reconocemos que M y N son los circuncentros de △AIC y △AIB, res-
pectivamente. Como AN = NI y AM = MI concluimos que MN es la mediatriz de AI; por consiguiente,
94 Incírculos mixtilíneos

M
A

I Q

B R E C

T P

Figura 10.6. Otra aplicación de las propiedades de incírculos mixtilíneos.

AI ⊥ PQ. Según el lema 10.2, P y Q son los puntos correspondientes de tangencia del A-incírculo mixti-
líneo con AB y AC. Esto nos asegura que NP y MQ se cortan sobre el punto de tangencia de este círculo
y (ABC), digamos T .
Sean r1 y r2 los circunradios de △BNP y △CMQ, respectivamente. Por la ley del seno en estos
triángulos, conseguimos:
r1 NB sin ∠MQC NB sin ∠T QC NB sin ∠T PQ NB T Q
= = = =
r2 MC sin ∠NPB MC sin ∠BPT MC sin ∠PQT MC T P
pero △T IN ∼ △TCM ya que ∠T NI = ∠T MC y ∠NT I = ∠PT I = ∠PBI = ∠IBC = ∠QTC. Además, P
y Q son puntos correspondientes en estos triángulos al saber que ∠T PI = ∠T QC, por tanto:
NB NI TP
= =
MC MC T Q
En definitiva, r1 = r2 y la solución queda completa.

A M

N Q

I
P

B C

Figura 10.7. Problema 6 de la OMCC 2016.


10.4 Problemas propuestos 95

10.4 Problemas propuestos


Problema 10.1. Sea T el punto de tangencia del A-incírculo mixtilíneo y el circuncírculo del triángulo
ABC. El incentro de △ABC es I. Demostrar que:
△BT I ∼ △T IC, es decir, que T es el centro de semejanza espiral que envía BI a IC.
El simétrico de I con respecto a T yace sobre el circuncírculo de △BIC.

Problema 10.2. El círculo β con centro J toca internamente al círculo Ω con centro O en el punto T . La
recta T J interseca a β por segunda vez en M. La tangente a β por M interseca a Ω en A y B. La segunda
tangente desde B hacia β corta a Ω por segunda vez en C. Probar que BA + AC = 3AB.

Problema 10.3. (Japón 2009, P4). Sea Γ el circuncírculo del triángulo ABC. Un círculo con centro O
toca al segmento BC en P y toca al arco BC de Γ que no contiene a A en Q. Si ∠BAO = ∠CAO, probar
que ∠PAO = ∠QAO.

Problema 10.4. (Lista Corta IMO 2017, G4). En el triángulo ABC, sea ω el A-excírculo. Sean D, E, F
los puntos donde ω es tangente a las rectas BC, CA, AB, respectivamente. El círculo (AEF) interseca a
la recta BC en P, Q. Sea M el punto medio de AD. Probar que el círculo (MPQ) es tangente a ω.

Problema 10.5. (EE. UU. 2017, P3). Sea ABC un triángulo escaleno con circuncírculo Omega e incentro
I. El rayo AI corta a BC en D y a Ω por segunda vez en M. El círculo con diámetro DM corta a Ω por
segunda vez en K. Las rectas MK y BC se cortan en S, y N es el punto medio de IS. Los circuncírculos
de △KID y △MAN se intersecan en dos puntos L1 y L2 . Probar que Ω pasa por el punto medio de IL1 o
IL2 .

Problema 10.6. (Lista Corta IMO 2016, G2). Sea ABC un triángulo con circuncírculo Γ e incentro I y
sea M el punto medio de BC. Los puntos D, E, F se seleccionan sobre los segmentos BC, CA, AB tals que
ID ⊥ BC, IE ⊥ AI, e IF ⊥ AI. Suponga que el circuncírculo del triángulo △AEF interseca a Γ en X ̸= A.
Probar que las rectas XD y AM se cortan sobre Γ

Problema 10.7. (Tailandia 2011, Cuarto Selectivo IMO, P3). Un triángulo △ABC es dado, con circun-
círculo Γ e incírculo de radio unitario. Sea ΓA el círculo tangente a los lados AB y AC y al círculo Γ
internamente. Defina ΓB y ΓC similarmente. Sean RA , RB y RC los radios de ΓA , ΓB y ΓC , respectivamen-
te. Probar que RA + RB + RC ≥ 4.

Problema 10.8. Sea ABC un triángulo con AC > AB, circuncírculo Γ e incentro I. El incírculo de ABC
toca a BC, CA y AB en D, E y F, respectivamente. Sea M un punto sobre Γ tal que ∠MAI = 90◦ . Las
rectas DI y AM se cortan en P. El punto K yace sobre el segmento BC tal que MD = MK. El circuncírculo
de AMK corta a BC por segunda vez en Q. Probar que el punto medio del segmento EF yace sobre PQ.
96 Incírculos mixtilíneos
PARTE IV

CONCURRENCIA Y COLINEALIDAD
CAPÍTULO 11

TEOREMAS DE CEVA Y MENELAO

Los teoremas de Ceva y Menelao constituyen dos de las principales herramientas para abordar pro-
blemas de concurrencia y colinealidad. En realidad, es poco común que la aplicación directa de estos
teoremas proporcionen una solución completa. En la mayor parte de los casos es necesario retomar ele-
mentos de manipulación de longitudes (como el teorema de Tales, semejanza de triángulos, potencia de
puntos, incluso trigonometría).
Comencemos estableciendo el primer hecho fundamental de la parte de concurrencia y colinealidad:

Teorema 11.1 (Teorema de Ceva). En el triángulo ABC, los puntos D, E y F están sobre los lados
BC, CA y AB, respectivamente. Luego, las rectas AD, BE y CF concurren si y solo si:
BD CE AF
· · =1 (11.1)
DC EA FB

F
E
P

B D C

Figura 11.1. Teorema de Ceva.

Demostración. Primero, supongamos que las rectas AD, BE y CF concurren en un punto P y verifique-
mos que la expresión 11.1 es cierta. Notemos que los triángulos BPD y DPC comparten la altura que
parte de P hacia el lado BC, mientras que △BAD y △DAC hacen lo mismo con la altura que parte desde
A, entonces:1
[BPD] BD [BAD] BD
= y =
[DPC] DC [DAC] DC
por tanto:2  
[BAP] [BAD] − [BDP] BD [DAC] − [DPC] BD
= = =
[PAC] [DAC] − [DPC] DC [DAC] − [DPC] DC
1 Esta idea es muy útil al realizar manipulación de longitudes.
2 En este paso aplicamos el resultado siguiente: si ab = xy , entonces a−x
b−y = a
b = xy .
100 Teoremas de Ceva y Menelao

De forma similar podemos obtener que [CBP] : [PBA] = CE : EA y [ACP] : [PCB] = AF : FB, así que:

BD CE AF [BAP] [CBP] [ACP]


· · = · · =1
DC EA FB [CAP] [PBA] [PCB]

Para la segunda dirección, asumamos que la igualdad 11.1 es cierta y definamos a P como el punto
de intersección de BE y CF. Basta demostrar que D yace sobre AP. Sea D′ = AP ∩ BC. Como AD′ , BE y
CF concurren en P, por la primera parte de la prueba sabemos que:

BD′ CE AF
· · =1 (11.2)
D′C EA FB

siendo así, al comparar 11.1 y 11.2 concluimos que

BD′ BD
= (11.3)
D′C DC

Si BD > BD′ , esto implicaría que DC < D′C y tendríamos que BD′ : D′C < BD : DC, lo que contradice
el hallazgo 11.3. Análogamente, llegamos a una contradicción si suponemos que BD < BD′ , así que
inferimos que BD = BD′ y CD = CD′ , lo que a su vez conlleva a que D ≡ D′ , como requeríamos.

Un resultado inmediato del teorema de Ceva en su versión original es la concurrencia relacionada


a conjugados isotómicos. Recordemos que dos rectas AK y AL son conjugadas isotómicas (con K y L
sobre el lado BC) si tenemos que BK = CL; es decir, K y L son reflexiones uno del otro con respecto
al punto medio de BC. Entonces, si tres rectas que parten de los vértices del △ABC son concurrentes,
sus conjugadas isotómicas también concurren. Los puntos de concurrencia se conocen como conjugados
isotómicos. El ejemplo más claro de un par de puntos que son conjugados isotómicos son el punto de
Gergonne de △ABC y su punto de Nagel.

Definición 11.1 (Cevianas y triángulo ceviano). Las rectas AD, BE y CF son cevianas del △ABC.
Una ceviana corresponde al segmento que une a un vértice de △ABC con un punto en el lado
opuesto correspondiente.
El triángulo formado por D, E y F recibe el nombre de triángulo ceviano de P con respecto a
△ABC.

Si aplicamos el teorema de la bisectriz generalizado en △ABC con cevianas AD, BE y CF, podemos
obtener una formulación adicional (e igual de útil) del teorema de Ceva.

Teorema 11.2 (Teorema de Ceva, versión trigonométrica). Manteniendo la notación del enunciado
del teorema de Ceva original, las rectas AD, BE y CF concurren si y solo si:
sin ∠BAD sin ∠CBE sin ∠ACF
· · =1
sin ∠DAC sin ∠EBA sin ∠FCB

Esta versión del teorema de Ceva implica que si tres rectas que pasan por los vértices de △ABC
concurren, sus conjugadas isogonales también concurren. Este es el caso de las medianas (que concurren
en el baricentro) y las simedianas (que concurren en el punto de Lemoine).
101

Lema 11.1 (Nido ceviano). Sean D, E y F puntos sobre los lados BC, CA y AB de un triángulo
△ABC. Los puntos X, Y y Z yacen sobre los lados EF, FD y DE del △DEF. Si dos de las tripletas
de rectas (AD, BE, CF), (AX, BY, CZ) y (DX, EY, FZ) son concurrentes, la tercera también lo
es.

Demostración. Supongamos que las tripletas (AD, BE, CF) y (DX, EY, FZ) concurren. Probaremos
que (AX, BY, CZ) también concurren. Los dos casos restantes se pueden demostrar de forma análoga.
De acuerdo con el teorema de la bisectriz generalizado, sabemos que:
sin ∠FAX FX AE
= ·
sin ∠XAE XE AF
Si extraemos las razones correspondientes para los otros dos pares de ángulos obtenemos:
  
sin ∠FAX sin ∠ECZ sin ∠DBY FX EZ DY AE CD FB
· · = · · · · =1
sin ∠XAE sin ∠ZCD sin ∠Y BA XE ZD Y F EF CE BD
Los dos productos entre paréntesis son iguales a 1 por el teorema de Ceva. Luego, por la versión trigono-
métrica del teorema de Ceva, AX, BY y CZ concurren.

F X
E

Y Z

B D C

Figura 11.2. Teorema del nido ceviano.

Una versión menos conocida pero igualmente interesante e importante de conocer del teorema es la
siguiente:

Teorema 11.3 (Ceva sobre la circunferencia). Sean A, B, C, D, E y F seis puntos sobre la cir-
cunferencia ω. Luego, las cuerdas AD, BE y CF son concurrentes si y solo si

AB ·CD · EF = BC · DE · FA (11.4)

Demostración. Primero supongamos que AD, BE y CF concurren en un punto P. Notemos que △APB ∼
△EPD, △BPC ∼ △FPE y △CPD ∼ △APF. De aquí obtenemos las siguientes relaciones:
AB PA EF PF CD PC PC PB
= , = , = y =
DE PE BC PB FA PA PE PF
102 Teoremas de Ceva y Menelao

F
B

P
C
E

Figura 11.3. Teorema de Ceva aplicado a cuerdas.

Multiplicando estas expresiones obtenemos la igualdad deseada.


Para la segunda parte, asumamos la validez de la relación (11.5). Sabemos que al menos uno de los
arcos ABC,
d CDE[ y EFA d de ω debe ser menor de 180◦ . Asumamos, sin pérdida de generalidad, que el arco
d y definamos a P como la intersección de AD y BE. Sea F ′ = CP ∩ ω, F ′ ̸= C.
con esta propiedad es EFA
Por la primera parte de la prueba conocemos que

AB ·CD · EF ′ = BC · DE · F ′ A

que al comparar con (11.5) permite concluir que EF · F ′ A = EF ′ · FA. Si EF > EF ′ necesariamente
F ′ A > FA, de modo que EF · F ′ A > EF ′ · FA, lo que contradice la igualdad que acabamos de conseguir.
Similarmente, si EF < EF ′ entonces F ′ A < FA y llegamos a una contradicción. Siendo así, F ≡ F ′ y la
demostración queda completa.

11.1 Colinealidad: Teorema de Menelao

Teorema 11.4 (Menelao). Los puntos D, E y F yacen sobre los lados BC, CA y AB del triángulo
ABC (posiblemente en sus prolongaciones). Entonces, D, E y F están alineados si y solo si:
DC BF AE
· · =1 (11.5)
DB FA EC

Demostración. Seguiremos una estrategia similar a la demostración del teorema de Ceva. Primero, su-
pondremos que D, E y F son colineales y obtendremos la expresión 11.5. Veamos que:
[DEC] DC [BDE] BF [AED] AE
= , = y =
[DEB] DB [EDA] FA [EDC] EC
Multiplicando los lados derecho e izquierdo de las expresiones anteriores conseguimos la igualdad 11.5.
Para la dirección restante, asumamos que (11.5) es válida y demostremos que D, E y F son colineales.
En efecto, sea D′ = EF ∩ BC. Por la primera sección de la prueba sabemos que:
D′C BF AE
· · =1 (11.6)
D′ B FA EC
11.2 Problemas resueltos 103

La comparación directa de (11.5) y (11.6) conlleva a inferir que D′C : D′ B = DC : DB, lo que en última
instancia implica que D ≡ D′ y la colinealidad deseada.

F
E

B C D

Figura 11.4. Teorema de Menelao.

El teorema de Menelao continúa siendo válido cuando los tres puntos yacen sobre las prolongaciones
de los lados BC, CA y AB (panel izquierdo de la 11.5) o cuando D yace en el interior del lado BC (panel
derecho de la 11.5). La demostración es similar en estos casos.

A A

B C D B D C
E

F
F

Figura 11.5. Configuraciones adicionales del teorema de Menelao.

11.2 Problemas resueltos


Es momento de solucionar algunos ejemplos con las herramientas recién abordadas.

Ejemplo 11.1. (Sharygin 2016, Ronda Final, Grado 10, P6). Un triángulo ABC es dado. El punto
K es la base de la bisectriz externa del ángulo A. El punto M es el punto medio del arco AC del
circuncírculo. El punto N yace sobre la bisectriz del ángulo C de modo que AN ∥ BM. Probar que
los puntos M, N, K están alineados.
104 Teoremas de Ceva y Menelao

M
A

N J

K B C

Solución. Por el teorema de Menelao aplicado al triángulo ICB y transversal K, N, M es suficiente probar
que:
MI BK CN
· · =1 (11.7)
MB KC NI
Por el teorema de la bisectriz, sabemos que BK : KC = AB : AC (a). Sea J = BI ∩ AC. Como AN ∥ IJ, por
el teorema de Tales conseguimos que CN : NI = AC : AJ (b). Además, notemos que ∠BAJ = ∠BMC y
∠JBA = ∠CBM, asi que △BAJ ∼ △BMC por criterio AA, de modo que MI : MB = MC : MB = AJ : AB
(c), donde hemos usado que MI = MC por ser M el circuncentro de △AIC. Al combinar (a), (b) y (c)
concluimos que:
MI BK CN AJ AB AC
· · = · · =1
MB KC NI AB AC AJ
como deseábamos probar.

Ejemplo 11.2. (Cuenca del Pacífico 2012, Problema 1). Sea P un punto en el interior de un trián-
gulo ABC, y sean D, E, F los puntos de intersección de la recta AP con el lado BC del triángulo,
de la recta BP y el lado CA, y de la recta CP y el lado AB, respectivamente. Probar que el área del
triángulo ABC debe ser 6 si el área de cada uno de los triángulos PFA, PDB y PEC es 1.

F 1 b E

c 1
P
1 a
B D C
11.2 Problemas resueltos 105

Solución. Denotemos por a, b y c las áreas de los triángulos △DPC, △EPA y △APF, respectivamente.
Por el teorema de Ceva, sabemos que:

CD BF AE
· · =1 (11.8)
DB FA EC
Además, notemos que a = [CPD] : [DPB] = CD : DB. Análogamente, b = AE : EC y c = BF : FA. Susti-
tuyendo estos valores en (11.8) descubrimos que abc = 1 (a).
Por otro lado, es sencillo obtener que

1 + a [DPB] + [CPD] [BPC] BF


= = = =c
1 + b [EPC] + [APE] [CPA] FA

y de forma similar (1 + b) : (1 + c) = a y (1 + c) : (1 + a) = b. Sumando estas tres expresiones se llega a:

ab + bc + ca = 3 (11.9)

Es claro que a, b y c son reales positivos. Luego, por (a) y la desigualdad MA-MG deducimos que:
√3
ab + bc + ca ≥ 3 a2 b2 c2 = 3

Al cumplirse la igualdad en (11.9) y tomar en cuenta (a), podemos concluir que a = b = c = 1 y por ende
[ABC] = 6.

Ejemplo 11.3. (Sharygin 2019, Ronda de Correspondencia, Problema 19). Sean ALa , BLb , CLc
las bisectrices del triángulo ABC. Las tangentes al circuncírculo de ABC en B y C se cortan en el
punto Ka . Los puntos Kb y Kc se definen similarmente. Probar que las rectas Ka La , Kb Lb y Kc Lc
concurren.

Kb

A
Kc

Lc Lb

B La C

Ka
106 Teoremas de Ceva y Menelao

Solución. Sabemos que ALa , BLb y CLc concurren en el incentro de △ABC. Asimismo, AKa , BKb y CKc
concurren en el punto de Lemoine de △ABC. Luego, por el lema del nido ceviano tenemos que Ka La ,
Kb Lb y Kc Lc tienen un mismo punto de intersección.

Ejemplo 11.4. (Lista Corta IMO 2006, G4). Se escoge un punto D sobre el lado AC de un triángulo
ABC con ∠C < ∠A < 90◦ de forma que BD = BA. El incírculo de ABC es tangente a AB y AC en
puntos K y L , respectivamente. Sea J el incentro del triángulo BCD. Probar que la recta KL corta
a J en su punto medio.

L
M
P
N
K D
I
J
B C

Solución. Definamos a M como la intersección de AJ y KL, P el corte de AI y KL, y N el punto común de


KL e IC. Los puntos I y J yacen sobre la bisectriz de ∠C, así que son colineales con C. Según el teorema
de Menelao aplicado a △IAJ y transversal N, P y M sabemos que:

NI JM AP
· · =1
NJ MA PI
por consiguiente, es suficiente probar que NI : NJ = PI : AP. Al ser △ABD isósceles en B, es sencillo
obtener que
∠BDC ∠A
∠BJC = 90◦ + = 180◦ −
2 2
así que ∠IJB = ∠A : 2. Además, sabemos que ∠BNC = 90◦ (¿por qué?)3 , así que
 
◦ ◦ ∠C + ∠B ∠A
∠IBN = 90 − ∠NIB = 90 − =
2 2

Al ser AKIL cíclico, también sucede que ∠ILP = ∠A : 2. De este modo, por criterio AA tenemos que
△IBN ∼ △ILP y △JBN ∼ △ALP, de donde concluimos que:

NI NI BN PI LP PI
= · = · =
NJ BN NJ LP PA PA
como deseábamos probar.
3 Recuérdese el lema 6 del folleto de puntos notables, las perpendiculares ocultas.
11.3 Problemas propuestos 107

11.3 Problemas propuestos


Problema 11.1. Sea ABC un triángulo. Demostrar que:
a) Las medianas de △ABC concurren (en el baricentro).
b) Las alturas de △ABC concurren (en el ortocentro).
c) Las bisectrices de △ABC concurren (en el incentro).
d) Las simedianas de △ABC concurren (en el punto de Lemoine).
e) Las cevianas que unen cada vértice con el punto de contacto del incírculo con el lado opuesto
concurren (este punto se conoce como punto de Gergonne).
f) Las cevianas que unen cada vértice con el punto de contacto del excírculo correspondiente con el
lado opuesto concurren (este punto se conoce como punto de Nagel).

Problema 11.2. Sea D el punto medio del lado BC del triángulo ABC. Sea P un punto arbitrario sobre
AM. La recta BP corta al lado AC en E, y la recta CP corta al lado AB en F. Demostrar que EF ∥ BC.

Problema 11.3. Sea ABCD un trapecio con AD ∥ BC. Las rectas AB y CD se cortan en P, mientras que
los segmentos AC y BD se intersecan en Q. Sean M y N los puntos medios de AD y BC, respectivamente.
Demostrar que M, N, P y Q son colineales.

Problema 11.4. Sea ABC un triángulo y sea A1 el punto de intersección de la tangente al circuncírculo
de ABC por A con la recta BC. Los puntos B1 y C1 se definen análogamente. Probar que A1 , B1 y C1 están
alineados.

Problema 11.5. (EE. UU. 2003, P4). Sea ABC un triángulo. Un círculo que pasa por A y B interseca a
los segmentos AC y BC en D y E, respectivamente. Las rectas AB y DE se cortan en F, mientras que las
rectas BD y CF se cortan en M. Probar que MF = MC si y solo si MB · MD = MC2 .

Problema 11.6. (Centroamericana 2015, Problema 5). Sea ABC un triángulo tal que AC = 2AB. Sea D el
punto de intersección de la bisectriz del ángulo CAB con BC. Sea F el punto de intersección de la paralela
a AB que pasa por C con la perpendicular a AD que pasa por A. Probar que FD pasa por el punto medio
del segmento AC.

Problema 11.7. (Rumanía 2007, Selectivo para la Olimpiada Balcánica Junior, P3). Sea ABC un triángulo
con ∠A = 90◦ , y sea D un punto sobre AC. Denote por E la reflexión de A en la recta BD y F el punto de
intersección de CE con la perpendicular por D a BC. Probar que AF, DE y BC son concurrentes.

Problema 11.8. (Sharygin 2017, Ronda de Correspondencia, Problema 6). Sea ABCD un cuadrilátero
convexo con AC = BD = AD; E y F los puntos medios de AB y CD, respectivamente; O el punto común
de las diagonales. Probar que EF pasa por los puntos de contacto del incírculo del triángulo △AOD con
AO y OD.
108 Teoremas de Ceva y Menelao
CAPÍTULO 12

TEOREMA DEL EJE RADICAL

12.1 Definición y propiedades

Definición 12.1. El eje radical constituye el lugar geométrico de los puntos P que cuentan con
potencias iguales respecto a dos círculos de referencia Γ1 y Γ2 , y es una recta perpendicular a la
recta formada por los centros de ambas circunferencias.

O1 O2

Figura 12.1. El eje radical de Γ1 y Γ2 .

Demostración. Sean P y Q dos puntos con igual potencia con respecto a Γ1 y Γ2 . Es decir,

PO21 − r12 = PO22 − r22 , y


QO21 − r12 = QO22 − r22

lo que implica que PO21 − PO22 = QO21 − QO22 , un conocido criterio de perpendicularidad. De esta forma,
P y Q yacen sobre una recta perpendicular a O1 O2 , y al ser arbitrarios, debe ocurrir que el eje radical es
precisamente una recta perpendicular a O1 O2 .

Cuando Γ1 y Γ2 se intersecan, su eje radical coincide con la recta que pasa por los dos puntos de
intersección (¿por qué?). Cuando son tangentes externamente, el eje radical está dado por su tangente
interna; en tanto, cuando son tangentes internamente, el eje radical corresponde a su única tangente
común. Si estas circunferencias no se intersecan una a la otra, la construcción se complica un poco.
Previo a abordar el resultado principal concerniente a ejes radicales, dilucidaremos un método sencillo
de identificación para dos circunferencias no secantes.
110 Teorema del eje radical

O1 O2 O1 O2

Figura 12.2. Dos casos especiales de ejes radicales.

Proposición 12.1. Supóngase que Γ1 y Γ2 son dos circunferencias sin puntos de intersección. El
eje radical biseca las tangentes internas y externas de Γ1 y Γ2 .

P
M
Q

O1 O2

Figura 12.3. Una forma de construir el eje radical de dos círculos no secantes.

Demostración. Supongamos que una de las tangentes externas toca a Γ1 y Γ2 en P y Q, respectivamente,


y que el eje radical corta a PQ en M. Luego:

MP2 = Pot(M, Γ1 ) = Pot(M, Γ2 ) = MQ2

de donde MP = MQ. La demostración para la otra tangente externa y las tangentes internas es completa-
mente análoga.
El hecho 12.1 proporciona un camino para trazar el eje radical de dos circunferencias no secantes:
basta trazar dos tangentes comunes, localizar y unir sus puntos medios. Otra forma es ubicar el punto
medio de una tangente común y trazar la perpendicular hacia el segmento que une los centros de las
circunferencias en consideración. El segundo método lo provee el principal resultado de esta sección.

Teorema 12.1 (Teorema del eje radical). Sean Γ1 , Γ2 y Γ3 tres circunferencias diferentes (¡posi-
blemente de radio 0!). Luego:
1. Los tres ejes radicales determinados por Γ1 , Γ2 y Γ3 concurren en un mismo punto, a saber,
el centro radical de estas tres circunferencias; o,
2. Los tres ejes radicales son paralelos.
12.2 Problemas resueltos 111
Curve

O3

P
O1
O2
O1 O2 O3

Figura 12.4. Teorema del eje radical.

Demostración. Supongamos que los centros correspondientes O1 , O2 y O3 no están alineados. Sea P el


punto de intersección del eje radical de Γ1 y Γ2 con el eje radical de Γ2 y Γ3 . De este modo,

Pot(P, Γ1 ) = Pot(P, Γ2 ) = Pot(P, Γ3 )

luego, por transitividad, P también yace sobre el eje radical de Γ3 y Γ1 .


Cuando O1 , O2 y O3 yacen sobre una misma recta, los tres ejes radicales son perpendiculares a O1 O3 ,
por consiguiente, son paralelos.

Un error común al aplicar el teorema del eje radical es excluir el caso extremo en que los ejes radicales
son paralelos. ¡Asegúrate siempre de no olvidar esta situación especial! Tomando en cuenta este teorema,
el eje radical de dos circunferencias no secantes Γ1 y Γ2 también puede construirse de la siguiente forma:
se traza un tercer círculo lo suficientemente grande de modo que corte a Γ1 y Γ2 . Con esto, podemos
encontrar el centro radical P de las tres circunferencias en cuestión y el eje radical de las dos primeras
puede identificarse al trazar la perpendicular desde P hacia los centros de Γ1 y Γ2 .

12.2 Problemas resueltos

Ejemplo 12.1. (Cuenca del Pacífico 2020, P1). Sea Γ el circuncírculo del △ABC. Sea D un punto
sobre el lado BC. La tangente a Γ por A interseca la paralela a BA por D en E. El segmento CE
interseca a Γ de nuevo en F. Suponga que B, D, F, E son concíclicos. Probar que AC, BF, DE son
concurrentes.

Solución. Por la condición DE ∥ BA y el teorema del ángulo semiinscrito sabemos que:

∠CDE = ∠CBA = ∠CAE

lo que implica que AECD es un cuadrilátero cíclico. Luego, por el teorema del eje radical aplicado a
(ABCF), (BDFE) y (AECD), concluimos que BF, DE y AC son concurrentes.
112 Teorema del eje radical

A
F

B D C

Figura 12.5. Problema 1 de la APMO 2020.

Ejemplo 12.2. (Iberoamericana 2017, P2). Sea ABC un triángulo acutángulo y Γ su circuncírculo.
Sea D un punto sobre el segmento BC, diferente de B y C, y sea M el punto medio de AD. La
perpendicular a AB que pasa por D corta a AB en E y a Γ en F, con el punto D entre E y F. Las
rectas FC y EM se cortan en X. Si ∠DAE = ∠AFE, demostrar que AX es tangente a Γ.

X
A

M
E

B D C

Y
F

Figura 12.6. Problema 2 de la OIM 2017.

Solución. Sea Y un punto sobre el rayo AD más allá de D. La condición angular implica que ∠ADE =
∠AFE + ∠DAF = ∠EAD + ∠DAF = ∠EAF, así que:

∠Y DF = ∠ADE = ∠EAF = ∠BAF = ∠BCF = ∠DCF

lo que indica que AD es tangente al circuncírculo del triángulo CDF. Por la proposición 12.1 inferimos
que M está sobre el eje radical de (CDF) y la circunferencia con centro A y radio 0. Es más, la condición
angular implica que EA2 = ED · EF, por lo que E también yace sobre tal recta. Concluimos que ME es
12.3 Problemas propuestos 113

el eje radical de estas dos circunferencias; luego, por el teorema del eje radical aplicado a Γ, (CDF) y
O(A, 0), deducimos que la tangente a Γ por A pasa por el punto de intersección de EM y FC, es decir
X.

Ejemplo 12.3. (Irán 2011, Selectivo IMO, P1). En el triángulo acutángulo ABC el ángulo B es
mayor que el ángulo C. Sea M el punto medio de BC. Los puntos F y E son los pies de alturas
desde C y B, respectivamente. Sean K y L los puntos medios de ME y MD, respectivamente. Si
KL corta a la paralela a BC por A en T , probar que TA = T M.

A
T

F
K
L
B M C

Figura 12.7. Un problema clásico de ejes radicales.

Solución. Recordemos que BFEC es un cuadrilátero cíclico (con diámetro BC), así que:

∠TAE = ∠ECB = ∠EFA

es decir, TA es tangente al circuncírculo de △EAF. Dicho de otro modo, TA es el eje radical de (EAF)
y O(A, 0).
Por otro lado, es bien conocido que MF y ME son tangentes a (EAF) (¿por qué?). Como LF 2 = LM 2 y
KE = KM 2 , descubrimos que KL es el eje radical de (EAF) y la circunferencia O(M, 0). En conclusión,
2

por el teorema del eje radical aplicado a O(A, 0), (EAF) y O(M, 0) inferimos que T es el centro radical
de estos círculos, por lo que TA2 = T M 2 . La solución está completa.

12.3 Problemas propuestos


Problema 12.1. Utilizando ejes radicales, demostrar que:
a) La existencia del circuncentro del triángulo ABC. (Pista. Considerar las circunferencias centradas
en los vértices y que tienen radios iguales a 0.)
b) Las alturas de △ABC son concurrentes.

Problema 12.2. Sea C un punto sobre un semicírculo con diámetro AB y sea D el punto medio del arco
AC. Sea E la proyección de D sobre la recta BC y F la intersección de AE con el semicírculo. Probar que
BF biseca al segmento DE.
114 Teorema del eje radical

Problema 12.3. Sobre los lados AB y AC del triángulo ABC se colocan los puntos P y Q, respectivamente,
tal que PQ ∥ BC. Probar que los círculos con diámetros BQ y CP se cortan sobre la recta que pasa por A
y es perpendicular a BC.

Problema 12.4. (Iraní de Geometría 2015, Nivel Intermedio, P3). En el triángulo ABC, M, N y K son
los puntos medios de los lados BC, AC y AB, respectivamente. Se construyen dos semicírculos con
diámetros AB y AC fuera del triángulo ABC. Suponga que MK y MN intersecan a estos semicírculos en
X e Y , respectivamente. Las tangentes a los semicírculos por X e Y se cortan en Z. Probar que AZ ⊥ BC.

Problema 12.5. (IMO 1995, P1). Sean A, B, C y D cuatro puntos distintos sobre una línea, en ese orden.
Los círculos con diámetros AC y BD se cortan en X y Y . La recta XY interseca a BC en Z. Sea P un punto
sobre la recta XY diferente de Z. La recta CP corta al círculo de diámetro AC en C y M, y la recta BP
corta al círculo de diámetro BD en B y N. Probar que las rectas AM, DN y XY son concurrentes.

Problema 12.6. (IMO 2000, P1). Dos círculos G1 y G2 se cortan en dos puntos M y N. Sea AB la
tangente a estos círculos en A y B, respectivamente, de forma que M está más cerca de AB que N. Sea CD
la paralela a AB por M, con C sobre G1 y D sobre G2 . Las rectas AC y BD se cortan en E; las rectas AN
y CD se intersecan en P; las rectas BN y CD se encuentran en Q. Probar que EP = EQ.

Problema 12.7. (IMO 2008, P1). Un triángulo acutángulo ABC tiene ortocentro H. La circunferencia
con centro en el punto medio de BC que pasa por H corta a la recta BC en A1 y A2 . La circunferencia con
centro en el punto medio de CA que pasa por H corta a la recta CA en B1 y B2 . La circunferencia con
centro en el punto medio de AB que pasa por H corta a la recta AB en C1 y C2 . Demostrar que A1 , A2 , B1 ,
B2 , C1 , C2 están sobre una misma circunferencia.

Problema 12.8. (Centroamericana 2021, P6). Sea ABC un triángulo con AB < AC y sea M el punto
medio de AC. Un punto P (diferente de B) se escoge en el segmento BC de modo que AB = AP. Sea
D la intersección de AC con el circuncírculo de △ABP distinta de A, y E la intersección de PM con el
circuncírculo de △ABP distinto de P. Sea K la intersección de las rectas AP y DE. Sea F un punto sobre
BC (diferente de P) tal que KP = KF. Demostrar que C, D, E y F yacen sobre la misma circunferencia.

Problema 12.9. (Iberoamericana 2011, P3). Sea ABC un triángulo y sean X, Y , Z los puntos de tangencia
de su circunferencia inscrita con los lados BC, CA, AB, respectivamente. Suponga que C1 , C2 , C3 son
circunferencias con cuerdas Y Z, ZX, XY , respectivamente, tales que C1 , C2 se cortan sobre la recta CZ
y que C1 , C3 se cortan sobre la recta BY . Suponga que C1 corta a las cuerdas XY y ZX en J y M,
respectivamente; C2 corta a las cuerdas Y Z y XY en L e I, respectivamente; y que C3 corta a las cuerdas
Y Z y ZX en K y N, respectivamente. Demostrar que I, J, K, L, M, N están sobre una misma circunferencia.
CAPÍTULO 13

HOMOTECIA

13.1 Introducción
La homotecia constituye otra herramienta de amplia utilidad para la resolver problemas de concurren-
cia y colinealidad. En esta ocasión, nos concentraremos en la homotecia aplicada a polígonos (triángulos,
por lo general). Antes de resolver algunos ejercicios, brindaremos algunas definiciones y propiedades
fundamentales.

13.2 Homotecia en polígonos

Definición 13.1. La homotecia es una transformación geométrica la que, dado un punto P en el


plano y un número real k ̸= 0, traslada cada punto Q del plano a un punto Q′ de modo que:
Los puntos P, Q y Q′ son colineales, y
Q′ P
=k
QP

El punto P se llama centro de homotecia, k es la razón de homotecia y los puntos Q y Q′ son nom-
brados elementos homólogos.
Si k es un número real positivo, diremos que la homotecia es directa; en este caso, los elementos
homólogos yacen a un mismo lado del centro de homotecia P; en contraste, la homotecia es inversa si k
es negativo, en la que los elementos homólogos yacen en lados opuestos con respecto a P.
En caso de aplicar una homotecia con centro P y razón k a n puntos P1 , P2 , . . . , Pn y obtener los puntos
homólogos P1′ , P2′ , . . . Pn′ , se dice que los polígonos (no necesariamente convexos) P1 P2 . . . Pn y P1′ P2′ . . . Pn′
son homotéticos. En esta situación todos los elementos correspondientes (tales como lados, diagonales,

A′

A′′ A
B B′
B′′ P
C C′

C′′

Figura 13.1. Una homotecia positiva que envía △ABC a △A′ B′C, y una negativa que manda △ABC a
△A′′ B′′C′′ .
116 Homotecia

otros polígonos) también son elementos homólogos.

13.2.1 Propiedades
Entre las propiedades más importantes de la homotecia tenemos:
Los lados correspondientes de dos figuras homotéticas son paralelos.
La razón de las áreas de dos figuras homotéticas es igual al cuadrado de la razón de homotecia.
Los puntos notables de las figuras homotéticas son siempre colineales con el centro de homotecia.
Por ejemplo, los circuncentros de triángulos homotéticos son puntos notables de ellos y deben ser
colineales con el centro de homotecia.
La homotecia preserva ángulos y por ende tangencia.
Un caso particular es que si dos figuras homotéticas poseen circuncírculos y tienen un vértice en
común, entonces estos círculos son tangentes en ese vértice común.

A
C′
B′ P B

A′ C

Figura 13.2. Tangencia entre circuncírculos de figuras homotéticas.

13.3 Homotecia en circunferencias


13.3.1 Definición y construcción

Definición 13.2. Sean Ω1 y Ω2 dos círculos con centros O1 , O2 y radios r1 , r2 , respectivamente.


Considere los puntos P y Q tales que P es el centro de homotecia H + con razón rr12 que traslada Ω1
a Ω2 y Q es el centro de homotecia H − con coeficiente − rr21 que transforma Ω1 en Ω2 . Entonces,
P y Q son el exsimilicentro e insimilicentro de Ω1 y Ω2 , respectivamente. Además, son puntos
únicos.

¿Cómo construir el exsimilicentro y el insimilicentro? Un método general es el siguiente: Trazamos


dos diámetros AB y CD en Ω1 y Ω2 , respectivamente, tales que AB ∥ CD. Luego, P = AC ∩BD y Q = AD∩
BD. Ya debes intuir qué pasa cuando los radios de ambas círculos tienen la misma medida. Casos límite
ocurren cuando una de las circunferencias tiene radio cero, son concéntricas, entre otros; sin embargo, no
nos ocuparemos de ellos aquí.
Una situación particular y muy frecuente en competiciones matemáticas sucede cuando Ω1 y Ω2
están en “posición general”, es decir, no son cocéntricas, sus radios poseen distinta longitud positiva y
no poseen puntos comunes. En este contexto, podemos obtener el exsimilicentro P como el punto común
de las tangentes externas de Ω1 y Ω2 , mientras tanto, Q sería el punto de intersección de las tangentes
internas de Ω1 y Ω2 (de aquí los prefijos ex- e in-), como lo muestra la figura 13.3.
13.3 Homotecia en circunferencias 117

Ω2

Ω1 A F C
E
P O1 Q O2
B
D

Figura 13.3. El exsimilicentro P e insimilicentro Q de Ω1 y Ω2 .


A C

A
C

O1 Q O2 P P O1 Q O2
D
B

B D
Figura 13.4. Trazado general del exsimilicentro e insimilicentro de dos circunferencias.

13.3.2 Puntos homólogos y antihomólogos

Definición 13.3 (Puntos homólogos). Sea A un punto sobre Ω1 y C = H + (A), es decir, C es un


punto sobre Ω2 tal que CO2 ∥ AO1 y yace al mismo lado que A con respecto a O1 O2 . Luego, C es
el homólogo de A respecto a P. Por el contrario, si D = H − (A), entonces D es el homólogo de A
con respecto a Q.

En otras palabras, los puntos correspondientes son siempre puntos homólogos, según sea el caso. Es
claro que AC pasa por P y AD por Q. En la primera figura, los puntos B, D y E, F también son pares
homólogos respecto a P (E = PA ∩ Ω1 , E ̸= A; F = PC ∩ Ω2 , F ̸= C). En adición, AB ∥ CD, EB ∥ FD.1
Por supuesto, existen también puntos antihomólogos.

Definición 13.4. Sean E y F los segundos puntos de intersección de AC con Ω1 y Ω2 , respectiva-


mente. Los pares de puntos A, F y E, C son llamados antihomólogos con respecto a P.

Obsérvese la principal diferencia entre los primeros y los segundos: mientras AO1 ∥ CO2 y EO1 ∥
FO2 , los segmentos determinados por puntos antihomólogos y sus respectivos centros no son paralelos,
1 Nota:Si los pares X1 ∈ Ω1 , Y1 ∈ Ω2 y X2 ∈ Ω1 , Y2 ∈ Ω2 son homólogos respecto a P, X1Y2 y X2Y2 no necesariamente
deben pasar por Q y si lo son respecto a Q, tampoco es cierto que su punto de intersección siempre sea P.
118 Homotecia

por ejemplo, EO1 ∦ CO2 y AO1 ∦ FO2 . En la siguiente sección, volveremos a encontrarnos con estos
puntos.

13.4 Algunos resultados útiles

Lema 13.1. Sean X1 , Y1 y X2 ,Y2 dos parejas de puntos antihomólogos tales que X1Y1 , X2Y2 son
rectas distintas, X1 , X2 yacen sobre Ω1 y Y1 , Y2 están sobre Ω2 . Entonces, el cuadrilátero X1Y1Y2 X1
es cíclico.

Demostración. Supongamos que estos pares de puntos son antihomólogos respecto al exsimilicentro
P. Sean Z1 = H + (Y1 ), Z2 = H + (Y2 ), luego Z1 Z2 ∥ Y1Y2 ; por consiguiente, debido a que Z1 X1 X2 Z2
es cíclico, también X1Y1Y2 X2 debe serlo. En tanto, si X1 , Y1 y X2 , Y2 son antihomólogos respecto al
insimilicentro Q, entonces definamos Z1 = H − (Y1 ), Z2 = H − (Y2 ) y un paso similar al previo nos
permite deducir que X1 , Y1 , Y2 y X2 están sobre una misma circunferencia.

Z1

Z1 X1 Y1
X1 Y2
P Q Q P
Z2 X2
X2
Y2 Y1
Z2

Figura 13.5. Puntos antihomólogos involucran cuadriláteros cíclicos.


Debido a este resultado, podemos deducir que para dos parejas de puntos antihomólogos respecto
al exsimilicentro o insimilicentro X1 , Y1 y X2 , Y2 , tendremos que PX1 · PY1 = PX2 · PY2 o QX1 · QY1 =
QX2 · QY2 , respectivamente.

Lema 13.2. Sean R y S los puntos de intersección de dos circunferencias no congruentes Ω1 y


Ω2 , sea P su exsimilicentro y A1 ∈ Ω1 , A2 ∈ Ω2 tal que A1 A2 es tangente externamente a ambas
circunferencias y A1 A2 está más cerca de S que de R. Luego,
i) PR es tangente al circuncírculo del △A1 RA2 .
ii) Sea T la reflexión de S en la recta A1 A2 ; entonces PT es tangente al circuncírculo del
△A1 RA2 .

Demostración. Notemos que A1 y A2 son puntos antihomólogos (¿por qué?) y R es su propio antihomó-
logo; por ende, gracias al lema 13.1 inferimos que PR2 = PA1 · PA2 , de donde surge el primer hecho. Por
otra parte, observemos que,
∠A1 TA2 = 180◦ − ∠TA1 A2 − ∠TA2 A1 = 180◦ − ∠SA1 A2 − ∠SA2 A1
= 180◦ − ∠SRA1 − ∠SRA2 = 180◦ − ∠A1 RA2
de este modo, RA1 TA2 es cíclico y ∠A1 RT = ∠A1 A2 T = ∠A2 RS. Como RS biseca a A1 A2 (¿por qué?),
deducimos que RT es la R-simediana del △A1 RA2 , por lo que A1 A2 es la A2 -simediana del △RA2 T ; dado
el hecho i), por consiguiente, PT debe ser tangente a (A1 RA2 ) en T .
13.5 Problemas resueltos 119

P S
A1

A2
T

Figura 13.6. Tangentes relacionadas con puntos antihomólogos.

Teorema 13.1 (Monge). Sean Ω1 , Ω2 y Ω3 tres circunferencias. Los exsimilicentros de Ωi y Ωi+1


(donde los índices son tomados módulo 3) están alineados.

O1 O2
P1
O3

P2
P3

Figura 13.7. Teorema de Monge.

Demostración. Sean Oi el circuncentro de Ωi , ri el radio de Ωi y Pi el exsimilicentro de Ωi , Ωi+1 . Por el


teorema de Menelao aplicado al △O1 O2 O3 , es suficiente probar que PP33 O 3 O1 P1 O2 P2
O1 P1 O2 P2 O3 = 1, pero,

P3 O3 O1 P1 O2 P2 r3 r1 r2
= =1
P3 O1 P1 O2 P2 O3 r1 r2 r3
la conclusión es inmediata.

Teorema 13.2 (Monge-d’Alembert). Dadas tres circunferencias Ω1 , Ω2 y Ω3 , el exsimilicentro de


Ω1 y Ω2 , el insimilicentro de Ω1 y Ω3 , y el insimilicentro de Ω2 y Ω3 son colineales.

Demostración. Similar a la prueba del teorema de Monge. Se deja como ejercicio al lector.

13.5 Problemas resueltos


¡Es momento de resolver algunos ejercicios! El primero es un resultado extremadamente útil.
120 Homotecia

O1
O2
P1

Q1
Q2
O3

Figura 13.8. Teorema de Monge-D’Alembert.

Ejemplo 13.1. (Olimpiada Balcánica de Matemática 2016, P2). Sea ABCD un cuadrilátero cíclico
con AB < CD. Las diagonales se intersectan en F y las rectas AD y BC en E. Sean K y L las
proyecciones ortogonales de F en AD y BC, respectivamente, y sean M, S y T los puntos medios de
EF, CF y DF, respectivamente. Pruebe que el segundo punto de intersección de los circuncírculos
de los triángulos MKT y MLS yace sobre CD.

D
K
T A
F
Z X
S M

C L B E

Solución. Sea Z el punto medio de CD. Observemos que el circuncírculo del △MKT es el círculo de los
9 puntos del △EFD; por ende, su punto de intersección con DE, digamos X, debe ser el punto medio
de ED. Notemos que los triángulos XT Z y EFC son homotéticos con centro de D y razón 12 , por lo que
∠XZT = ∠ECF. Además, el cuadrilátero XMT D es un paralelogramo, por lo que:

∠XMT = ∠ADT = ∠ADB = ∠BCF = ∠ECF = ∠XZT

luego, Z yace sobre el circuncírculo del △MKT . Análogamente podemos demostrar que M yace sobre el
circuncírculo del △MLS, por lo que Z es el punto de intersección deseado y además yace sobre CD.

Ejemplo 13.2. Sea O el centro de la circunferencia Γ1 . Sea ℓ una recta arbitraria que no pasa por
O. Una segunda circunferencia Γ2 es tangente a Γ1 en P y a ℓ en Q. La perpendicular a ℓ que pasa
por O corta a Γ1 en dos puntos, sea R el que está más alejado de ℓ. Probar que P, Q y R están
alineados.
13.5 Problemas resueltos 121

P
O1

Q

Figura 13.9. Una aplicación de homotecia inversa.

Solución. Sea O1 el centro de Γ2 . Notemos que OR ∥ O1 Q ya que ambas rectas son perpendiculares a ℓ.
Como R y Q yacen en distintos lados de OO1 , el paralelismo implica que ambos son puntos homólogos
con respecto al insimilicentro de Γ1 y Γ2 , a saber, P. Luego, QR pasa por P.

Ejemplo 13.3. (México 2014, P3). Sean Γ1 una circunferencia y P un punto fuera de Γ1 . Las
tangentes desde P a Γ1 tocan a la circunferencia en los puntos A y B. Considera M el punto medio
del segmento PA y Γ2 la circunferencia que pasa por los puntos P, A y B. La recta BM interseca
de nuevo a Γ2 en el punto C, la recta CA interseca de nuevo a Γ1 en el punto D, el segmento DB
interseca de nuevo a Γ2 en el punto E y la recta PE interseca a Γ1 en el punto F (con E entre P y
F). Muestra que las rectas AF, BP y CE concurren.

D
A

C
F
E M
Q
K B P

Figura 13.10. Problema 3 de la OMM 2014.

Solución. Sea Q el segundo punto de intersección de BC y Γ1 . Como MA es tangente a Γ1 , notemos que:


MQ · MB = MA2 = MA · MP = MC · MB
así que M también es punto medio de CQ. Siendo así, AQPC es un paralelogramo. Luego, conseguimos:
∠CQA = ∠QCP = ∠BCP = ∠BAP = ∠PBA = 180◦ − ∠ACP = ∠QAC
122 Homotecia

de modo que QCA es isósceles en A, por lo que AQBD es un trapecio isósceles. Esto se traduce en
BE ∥ QA ∥ PC. Además, ∠BEP = ∠BCP = ∠QAC = ∠BDC, por tanto QE ∥ AD, así que forzosamente
P, Q y E están alineados. Tomando en cuenta la definición de F, podemos inferir que EF ∥ CA y que
AQFD es un trapecio isósceles. Esto conlleva a que ∠BFQ = ∠BAQ = ∠PAC = ∠QPA, luego BF ∥ PA.
En resumen, los lados correspondientes de △BEF y △PCA son paralelos, por lo que estos triángulos
son homotéticos; por consiguiente, AF, CE y PB concurren en el centro de homotecia de estos triángulos.

Ejemplo 13.4. Sea ABCD un cuadrilátero convexo, de modo que existe una circunferencia tangen-
te externamente a AB, BC, CD y DA, y D es el punto más cercano a esta circunferencia. Sean E y
F las intersecciones de AB y CD, y de BC y AD, respectivamente. Sean K y L las intersecciones de
las bisectrices de ∠DAE con DE y de ∠DCF con DF, respectivamente. Demostrar que AC, KL y
la bisectriz de ∠ADE concurren.

Solución. Sea Γ la circunferencia en cuestión y O su centro. Asimismo, consideremos a Γ1 y Γ2 como


los incírculos de △DAE y △DCF, respectivamente.
Veamos que AO es la bisectriz de ∠DAE, así que K yace sobre AO. De este modo, K es el punto
de intersección de DE, una tangente interna común de Γ1 , Γ2 , con la recta que une los centros de estos
círculos; por consiguiente, K es el insimilicentro de Γ y Γ1 . Análogamente podemos concluir que L es
el insimilicentro de Γ y Γ2 . No hace falta decir que A y C son los exsimilicentros de Γ, Γ1 y Γ, Γ2 ,
respectivamente.

L O2 C
O
K D
O1

E A B

Figura 13.11. Los teoremas de Monge y Monge-D’Alembert en juego.

Sea P el exsimilicentro de Γ1 y Γ2 . Por el teorema de Monge-D’Alembert, P yace sobre KL. Además,


según el teorema de Monge, P también está sobre AC. Esto implica que P es el punto de intersección de
AC y LK. La recta que une los centros de Γ1 y Γ2 es la bisectriz interna de ∠ADE, y por definición de P,
tal recta debe pasar por este punto también. El resultado sigue.

13.6 Problemas propuestos


Los siguientes ejercicios pueden ser resueltos mediante diversos métodos, pero es recomendable usar
los resultados que acá hemos obtenido (¡de eso se trata!).
13.6 Problemas propuestos 123

Problema 13.1. Empleando homotecia, demostrar que el baricentro, ortocentro y circuncentro de un


triángulo son colineales (recta de Euler).
Problema 13.2. Demostrar que el ortocentro del triángulo ABC es el exsimilicentro del circuncírculo de
ABC y su círculo de los nueve puntos.
Problema 13.3 (Resultado útil). El incírculo de △ABC tiene centro I y toca a BC en E. AD es una altura
de △ABC; M es el punto medio de AD. Sea IA el A-excentro de △ABC. Demostrar que M, E, IA son
colineales.
Problema 13.4 (Resultado útil). Sean Ω y ω dos círculos tangentes en D, de forma que ω yace en el
interior de Ω. Se traza una cuerda de AB en Ω de modo que AB es tangente a ω en C. Probar que DC
biseca al arco AB
c que no contiene a AD.

Problema 13.5. (EGMO 2013, Problema 5). Sea Ω el circuncírculo del triángulo ABC. El círculo ω es
tangente a los lados AC y BC, y es tangente internamente al círculo Ω en P. Una recta paralela a AB que
pasa por el interior del triángulo ABC es tangente a ω en Q. Probar que ∠ACP = ∠QCB.
Problema 13.6. (Lista Corta Centroamericana 2015, G1). Sean Γ1 y Γ2 círculos que se cortan en dos
puntos distintos A y B. Sean C y D puntos sobre Γ1 y Γ2 tales que CB y DB son tangentes a Γ2 y Γ1 ,
respectivamente. Sea F un punto sobre Γ1 tal que AB = CF y G un punto sobre Γ2 tal que BG = DA. Sea
P el punto de corte de CF con BA, y Q el punto de corte de BG con DA. Muestra que el circuncírculo del
triángulo QFP es tangente a Γ2 .
Problema 13.7. (Centroamericana 1999, P3). Sea ABCDE un pentágono convexo. Si P, Q, R y S son los
baricentros respectivos de los triángulos ABE, BCE, CDE y DAE, demostrar que PQRS es un paralelo-
gramo y que su área es 2/9 del área de ABCD.
Problema 13.8. (México 2008, P6). Las bisectrices internas de los ángulos A, B y C de un triángulo
ABC concurren en I y cortan al circuncírculo de ABC en L, M, N, respectivamente. La circunferencia de
diámetro IL, corta al lado BC, en D y E; la circunferencia de diámetro IM corta al lado CA en F y G;
la circunferencia de diámetro IN corta al lado AB en H y J. Muestra que D, E, F, G, H, J están sobre una
misma circunferencia.
Problema 13.9. (Sharygin 2016, Ronda de Correspondencia) Un triángulo ABC es dado. Considere el
círculo tangente a (ABC) en A y externamente tangente al incírculo de △ABC en algún punto A1 . Los
puntos B1 , C1 son definidos similarmente.
Pruebe que las rectas AA1 , BB1 , CC1 concurren.
Sea A2 el punto de contacto del incírculo con BC. Demuestre que AA1 y AA2 son simétricas respecto
a la bisectriz interna del ∠A.
Problema 13.10. (Lista Corta IMO 2011, G4). Sea ABC un triángulo acutángulo con circuncírculo Ω.
Sea B0 el punto medio de AC y sea C0 el punto medio de AB. Sea D el pie de altura desde A y sea G el
baricentro del triángulo ABC. Sea ω el círculo que pasa por B0 y C0 que es tangente al círculo Ω en un
punto X ̸= A. Probar que los puntos D, G y X están alineados.
Problema 13.11. (EE. UU. 2013, Pre-Selectivo IMO, P1). Sea ABC un triángulo y D, E, F los puntos
medios de los arcos BC, CA, AB sobre el circuncírculo. La recta ℓa pasa por los pies de las perpendiculares
desde A hacia DB y DC. La recta ma pasa por los pies de las perpendiculares desde D hacia AB y AC.
Denote por A1 la intersección de ℓa y ma . Los puntos B1 y C1 se definen de forma similar. Probar que los
triángulos DEF y A1 B1C1 son semejantes.
124 Homotecia
CAPÍTULO 14

TEOREMAS SELECTOS

14.1 Una breve introducción a la razón cruzada


La razón cruzada es una invarianza fundamental en geometría euclidiana. Dados cuatro puntos ali-
neados A, B, C, D, podemos definir la razón cruzada como:
BA DA
(A,C; B, D) = : (14.1)
BC DC
donde las longitudes son dirigidas; esto es, establecemos una dirección particular como positiva y su
opuesta como negativa. Precisamente, ocurre que (A,C; B, D) > 0 si los segmentos AC y BD son disjuntos
o bien uno está contenido en el otro.

Definición 14.1. Sea P un punto no colineal con A, B, C, D. Las rectas PA, PB, PC, PD forman
un haz, representado como P(A,C; B, D).

A partir de la definición anterior, podemos probar nuestro primer resultado:

Proposición 14.1. Dado un haz P(A,C; B, D) y otra recta ℓ, sea A′ = PA ∩ ℓ. Los puntos B′ , C′ , D′
se contruyen de forma análoga. Entonces (A,C; B, D) = P(A,C; B, D) = (A′ ,C′ ; B′ , D′ ).

A′ B′ C′
D′

A B C D
Figura 14.1. El haz P(A,C; B, D).

Demostración. Al aplicar el teorema de la bisectriz generalizado a △APC con transversales PB y PD,


respectivamente, obtenemos:

BA PA sin ∠BPA DA PA sin ∠DPA


= · y = ·
BC PC sin ∠BPC DC PC sin ∠DPC
126 Teoremas selectos

por tanto:
BA DA sin ∠BPA sin ∠DPA
(A,C; B, D) = : = ·
BC DC sin ∠BPC sin ∠DPC
donde las longitudes son dirigidas. Lo anterior claramente implica la conclusión deseada.

El hecho anterior muestra el porqué la razón cruzada permanece constante: al considerar puntos
de intersección de una recta con un haz del cual no forma parte, los cuatro puntos resultantes conser-
van la razón cruzada de los cuatro puntos originales. Una forma común de representar este hecho es
P
(A,C; B, D) = (A′ ,C′ ; B′ , D′ ).

Una situación similar acontece al relacionar haces con circunferencias.

Proposición 14.2. Sean A, B, C y D puntos sobre una circunferencia Γ y P otro punto sobre Γ.
Luego
BA DA
|P(A,C; B, D)| =
:
BC DC
es decir, la razón cruzada (A,C; B, D) es independiente de P.

D
A

B C

A′ B′ C′ D′
Figura 14.2. Razón cruzada en una circunferencia.

Demostración. Usando la prueba del hecho anterior y por la ley del seno en circunferencias deducimos
que:

BA DA sin ∠BPA sin ∠DPA
BC : DC = sin ∠BPC · sin ∠DPC = |(A,C; B, D)| = |P(A,C; B, D)|

La conclusión sigue.

En efecto, si AC y BD se cortan P(A,C; B, D) es negativo, y positivo en caso contrario. La proposición


anterior indica que, al considerar los puntos comunes correspondientes A′ , B′ , C′ , D′ del haz P(A,C; B, D)
con otra recta, entonces (A′ ,C′ ; B′ , D′ ) = (A,C; B, D); a su vez, esto significa que al proyectar un haz dado
sobre una circunferencia o viceversa, la razón cruzada mantiene su carácter invariante.
14.2 Teoremas selectos de concurrencia y colinealidad 127

Otras propiedades
1. Si r es un número real y A, B, C son puntos distintos que yacen sobre una misma recta ℓ, entonces
existe un único punto D en ℓ tal que (A,C; B, D) = r.
2. Se tiene que (A,C; B, D) = (B, D; A,C).
3. El inverso de la proposición 14.1 también es cierto; es decir, si (A,C; B, D) = (A′ ,C′ ; B′ , D′ ), y
A, B, C, D y A′ , B′ , C′ , D′ son dos cuartetas de puntos colineales, entonces AA′ , BB′ , CC′ , DD′
concurren.

14.2 Teoremas selectos de concurrencia y colinealidad


14.2.1 Teoremas de Pascal y Brianchon

Teorema 14.1 (Pascal). Sea ABCDEF un hexágono inscrito (no necesariamente convexo) en un
círculo Γ. Entonces, los puntos P = AB ∩ DE, Q = BC ∩ EF y R = CD ∩ FA están alineados.

A
Q
P
D R E K
L
C

B
F

Figura 14.3. Teorema de Pascal.

Demostración. Defínanse K = DE ∩ BC y L = DC ∩ EF. Por las proposiciones 14.1 y 14.2, se obtiene


que:
B F
(D, E; P, K) = (D, E; A,C) = (D, L; R,C)
luego, tomando en cuenta el inverso de la proposición 14.1, se concluye que EL, PR y KC concurren en
Q. La colinealidad sigue inmediatamente.
Además de su versión original, una de las razones que explican la tremenda utilidad del teorema de
Pascal es que también es válido en casos extremos (como las que se muestran en la figura 14.4); esto es,
cuando se cuenta con menos de 6 puntos sobre la circunferencia de referencia.
Tales situaciones degeneradas ocurren cuando dos de ellos coinciden, en cuyo caso la recta que los
une está dada por la tangente correspondiente. Sin embargo, aún en estas circunstancias, continuamos
trabajando con hexágonos repitiendo los vértices respectivos. Por ejemplo, el panel izquierdo de la figura
14.4 refleja la aplicación del resultado al hexágono ABCDEE (pentágono ABCDE). Entretanto, las gráfi-
cas central y derecha corresponden a los hexágonos ABCCDD (cuadrilátero ABCD) y AABBCC (triángulo
ABC).
128 Teoremas selectos

P
R
C R R
A
A A

D Q B C
P
E C P D

B B
Q
Q

Figura 14.4. El teorema de Pascal admite versiones degeneradas.

Una técnica útil para no olvidar los pares de rectas que determinan los puntos colineales es tomar los
vértices consecutivos cuyas posiciones son las mismas al considerarlas módulo 3. Es decir:

A B C D E F
A B D E
B C E F
C D F A

Más interesante aún, el teorema de Pascal continúa siendo cierto para cualquier permutación de los
vértices de ABCDEF. Al emplearlo, debe tenerse suficiente cuidado con el orden de los puntos para que
obtengamos la colinealidad deseada. Vale mencionar que el resultado también aplica para cónicas; sin
embargo, la versión para circunferencias es por mucho la más frecuente en olimpiadas.
Por otro lado, un resultado íntimamente relacionado al teorema de Pascal (no a primera vista) viene
dado por:

Teorema 14.2 (Brianchon). Sea ABCDEF un hexágono circunscrito (no necesariamente convexo)
a un círculo Γ. Luego, las rectas AD, BE y CF son concurrentes.

A
B

F P
C

E D

Figura 14.5. Teorema de Brianchon.


Resulta que los teoremas de Pascal y Brianchon son duales bajo la transformación polar, en la que
14.2 Teoremas selectos de concurrencia y colinealidad 129

profundizaremos al estudiar polos y polares. Esto significa que, al tomar la colinealidad proporcionada
por teorema de Pascal y aplicarle la transformación polar con respecto al círculo de referencia, inmedia-
tamente obtenemos la concurrencia que el teorema de Brianchon implica.
Al igual que con el primer hecho, el teorema de Brianchon está sujeto a versiones degeneradas. En
este caso, los vértices “degenerados” coinciden con los puntos de tangencia entre el polígono y el círculo
inscrito. En efecto, al aplicar el teorema de Brianchon al hexágono ABCDEF con D, F y B puntos de
tangencia (como lo muestra el panel derecho de la figura 14.6), redescubrimos que los puntos de tangencia
del incírculo de △ACE concurren (en el punto de Gergonne).

A
E
A D
B C B
C F P F
P P

F E D A B E D C
Figura 14.6. Casos degenerados del teorema de Brianchon.

En el caso de Brianchon, las rectas están formadas por los puntos cuyas posiciones son las mismas
módulo 3.

A B C D E F
A D
B E
C F

14.2.2 Teorema de Pappus


El teorema de Pappus, con el mismo sentido pero mayor simplicidad que el teorema de Pascal, tam-
bién brinda una tríada de puntos alineados.

Teorema 14.3 (Pappus). Sean A, B, C tres puntos colineales (no necesariamente en ese orden) y
D, E, F tres puntos alineados (no necesariamente en ese orden). Luego, los puntos de intersección
de las rectas AE, BD; AF, CD y BF, CE yacen sobre una misma recta.

Demostración. Consideremos los puntos S = CD ∩ BF, R′ = CE ∩ PQ, O = CA ∩ FD, K = PQ ∩ AO y


L = CP ∩ FO. Observemos que:

A C P F
(C, S; R′ , E) = (K, Q; R, P) = (O, D; E, L) = (O, B; A,C) = (E, R; S,C) = (C, S; R, E)

por consiguiente R ≡ R′ y el resultado sigue.


130 Teoremas selectos

C Q
B P
A
R
P Q R B C A

D E F D F E

Figura 14.7. Teorema de Pappus.

C
B
A
Q R
K P
S

O L D E F

Figura 14.8. Prueba del teorema de Pappus.

A B C

D E F

Otra idea posible de demostración es considerar las dos rectas como una cónica degenerada, por lo
que la colinealidad surge al aplicar el teorema de Pascal.
Una forma sencilla para no olvidar qué rectas determinan los puntos de intersección es la siguiente.
Cabe destacar que el teorema es válido sin importar el orden de los dos pares de tríadas de puntos que
estamos considerando, como lo escenifica el diagrama derecho de la figura 14.7.

14.2.3 Teorema de Desargues

Consideremos dos triángulos arbitrarios ABC y DEF. Antes de establecer el principal resultado, es
necesario proporcionar dos definiciones fundamentales.

Definición 14.2. Los triángulos ABC y DEF están en perspectiva con respecto a un punto (diga-
mos O), si las rectas que unen sus vértices correspondientes —a saber, AD, BE y CF—, concurren
en O.
Este punto recibe el nombre de perspector de △ABC y △DEF.
14.2 Teoremas selectos de concurrencia y colinealidad 131

Definición 14.3. Los triángulos ABC y DEF están en perspectiva con respecto a una recta si
los puntos de intersección de los pares de lados correspondientes de ambos triángulos —es decir,
P = AB ∩ DE, Q = CA ∩ FD y R = BC ∩ EF—, están alineados.
La recta PQ es conocida como la perspectriz de △ABC y △DEF.

B A B A

C P C
Q
R
E F E F

D D
(a) Perspectiva con respecto a un punto. (b) Perspectiva con respecto a una recta.

Teorema 14.4 (Desargues). Los triángulos ABC y DEF están en perspectiva con respecto a un
punto si y solo si están en perspectiva con respecto a una recta.

Es decir, si △ABC y △DEF poseen un perspector, también deben tener una perspectriz y viceversa.

B A

P J C
K Q
R
E L F

D
Figura 14.10. Teorema de Desargues.

Demostración. Sean J = BC ∩ AD, K = PQ ∩ AD y L = EF ∩ AD. Además, definamos R′ = BC ∩ PQ


y R′′ = EF ∩ PQ. Por la propiedad 1, sabemos que P, Q y R están alineados si y solo si (P, Q; K, R′ ) =
(P, Q; K, R′′ ). Pero:
A D
(P, Q; K, R′ ) = (P, Q; K, R′′ ) ⇐⇒ (B,C; J, R′ ) = (P, Q; K, R′ ) = (P, Q; K, R′′ ) = (E, F; L, R′′ )

por lo que (P, Q; K, R′ ) = (P, Q; K, R′′ ) si y solo si AD, BE y CF concurren, como requeríamos.
132 Teoremas selectos

Notemos además que el hecho de que △ABC y △DEF estén en perspectiva respecto a O implica, de
acuerdo con el teorema de Desargues, que △BPE y △CQF estén en perspectiva respecto a la recta AD
y viceversa. Similarmente, si △ABC y △DEF están en perspectiva respecto a PQ, los triángulos △BPE
y △CQF también están en perspectiva respecto a R. Es decir, un par de triángulos en perspectiva nos
brinda otro par de triángulos con la misma propiedad.

O D
B

D
B C P
P Q Q C
A

E F E F R
A
(a) Las rectas BC, EF y PQ son paralelas. (b) Las rectas BE, CF y AD son paralelas.
Figura 14.11. Versiones degeneradas.

Al igual que los teoremas anteriores, es muy común reconocer versiones degeneradas del teorema de
Desargues en problemas de olimpiada. La figura 14.11a ilustra el caso en que un par de lados corres-
pondientes son paralelos, BC ∥ EF. Según el resultado, P∞ = BC ∩ EF debe estar sobre PQ, por tanto
PQ también es paralela a BC y EF. Entretanto, el gráfico 14.11b ilustra la situación particular en la que
BE ∥ CF. Por un argumento similar al anterior, también concluimos que AD es paralela a estas rectas.

14.3 Problemas resueltos

Ejemplo 14.1. (Iberoamericana 2014, P5) Sea ABC un triángulo acutángulo y H su ortocentro. Sea
D el pie de la altura desde A a BC. Sean M y N los puntos medios de BH y CH, respectivamente.
Las rectas DM y DN cortan a AB y AC en X y Y respectivamente. Si P es la intersección de XY con
BH y Q la intersección de XY con CH, demostrar que H, P, D, Q están sobre una circunferencia.

Solución. Notemos que los triángulos XMB y Y NC están en perspectiva con respecto a la recta AD. Así,
por el teorema de Desargues, las rectas XY, MN y BC deben concurrir. Sin embargo, al ser MN base
media de BC inferimos que MN ∥ BC; por tanto XY también debe ser paralela a estas rectas.
Observemos que el triángulo BMD es isósceles en M, ya que este punto es el circuncentro de BHD.
Como XP ∥ BD, es sencillo concluir que XPDB es un trapecio isósceles (y cíclico, de paso). Entonces:

∠DHQ = ∠DHC = ∠ABC = ∠XBD = ∠QPD

como requeríamos.
14.3 Problemas resueltos 133

H
X P Q Y

M N

B D C
Figura 14.12. Problema 5 de la OIM 2014.

Ejemplo 14.2. (APMO 2008, P3) Sea Γ el circuncírculo del triángulo ABC. Un círculo que pasa
por A y C corta a los lados BC y BA en D y E, respectivamente. Las rectas AD y CE intersecan a Γ
por segunda vez en G y H, respectivamente. Las tangentes a Γ por A y C intersecan a la recta DE
en L y M, respectivamente. Probar que las rectas LH y MG se cortan en Γ.

G M
P

H D
E
L

Q A C

Figura 14.13. Un cuadrilátero cíclico innecesario gracias al teorema de Pascal.

Solución. Sean Q = DE ∩ AC, P = BQ ∩ Γ, P ̸= B, L′ = PH ∩ AA y M ′ = PG ∩ CC. Por el teorema de


Pascal en los hexágonos BPHCAA y BPGACC inferimos que Q, L′ , E, D, M ′ son colineales, así que
L ≡ L′ y M ′ ≡ M; por tanto, QB, LH y MG concurren en P, el cual está sobre Γ.

Ejemplo 14.3. (Lista Corta IMO 2017, G1) Sea ABCDE un pentágono convexo tal que AB =
BC = CD, ∠EAB = ∠BCD y ∠EDC = ∠CBA. Probar que la perpendicular desde E a BC y los
segmentos AC y BD son concurrentes.

Solución. Denotemos por J y K los puntos de corte de BA y DE, CD y AE, respectivamente. Las condi-
ciones angulares junto a AB = BC = CD implican que ABCK y BCDJ son romboides. Por el teorema de
Pitot, ambos cuadriláteros tienen un incírculo. Debido a que comparten los ángulos ∠ABC, ∠BCD y el
134 Teoremas selectos

lado BC, sus incírculos deben coincidir, i.e. ABCDE es un pentágono circunscrito con incentro I. Sea F
el pie de la perpendicular desde E a BC. Tenemos que:

∠DEF = 270◦ − ∠EDC − ∠BCD = 270◦ − ∠ABC − ∠EAB = ∠FEA

de modo que E, I y F son colineales. Luego, por el teorema de Brianchon al hexágono ABFCDE, se
obtiene que EF, AC y BD son concurrentes.

C
F

B D
I

E
A

J
Figura 14.14. Problema G1 de la lista corta de la IMO 2017.

Ejemplo 14.4. (IMO 2019, P2) En el triángulo ABC, el punto A1 está en el lado BC y el punto
B1 está en el lado AC. Sean P y Q puntos en los segmentos AA1 y BB1 , respectivamente, tales
que PQ es paralelo a AB. Sea P1 un punto en la recta PB1 distinto de B1 , con B1 entre P y P1 , y
∠PP1C = ∠BAC. Análogamente, sea Q1 un punto en la recta QA1 distinto de A1 , con A1 entre Q y
Q1 , y ∠CQ1 Q = ∠CBA. Demostrar que los puntos P, Q, P1 , y Q1 son concíclicos.

Q1

S T

P1 C
B1 A1

P Q

A B
Figura 14.15. Problema 2 de la IMO 2019.
14.4 Problemas propuestos 135

Solución. Sin pérdida de generalidad, supongamos que BC ≥ AC. Definamos los puntos S = BA1 ∩ PB1
y T = AB1 ∩ QA1 . Por el teorema de Pappus aplicado a los puntos P, B1 , S y T, A1 , Q concluimos que
A, B y el punto de intersección de PQ y ST son colineales. Sin embargo, PQ ∥ AB, así que forzosamente
ST ∥ AB. De este modo:

∠PP1C = ∠BAC = ∠STC y ∠QQ1C = ∠CBA = ∠T SC

lo que implica que STCP1 y T Q1 SC son cíclicos, por ende C, P1 , Q1 , S y T yacen sobre la misma
circunferencia. Entonces ∠SP1 Q1 = ∠ST Q1 = ∠PQQ1 ; por consiguiente, PQQ1 P es cíclico.

14.4 Problemas propuestos


Problema 14.1. Sea r un número real y A, B, C puntos distintos que yacen sobre una misma recta ℓ.
Probar que existe un único punto D en ℓ tal que (A,C; B, D) = r.

Problema 14.2. Verificar que (A,C; B, D) = (B, D; A,C).

Problema 14.3. ¿Cuántas razones cruzadas están asociadas a cuatro puntos alineados A, B, C, D?
¿Cuántos de estos valores son diferentes?

Problema 14.4. Demuestre el converso de la proposición 14.1; es decir, si (A,C; B, D) = (A′ ,C′ ; B′ , D′ ),
y A, B, C, D y A′ , B′ , C′ , D′ son dos cuartetas de puntos colineales, entonces AA′ , BB′ , CC′ , DD′
concurren.

Problema 14.5. Sea ABCD un cuadrilátero con incírculo. Sean M, N, P y Q los puntos de tangencia del
incírculo con los lados AB, BC, CD y DA, respectivamente. Probar que las rectas AC, BD, MP y NQ
concurren.

Problema 14.6. Sea ABCD un cuadrilátero cíclico. Las tangentes por A y C al circuncírculo de ABCD se
cortan en P, mientras que las tangentes por B y D se intersecan en Q. Sea R la intersección de AB y CD,
y sea S la intersección de AD y BC. Demostrar que P, Q, R y S son colineales.

Problema 14.7. (Centroamericana 2010, P2). Sea ABC un triángulo y L, M y N los puntos medios de
BC, CA y AB, respectivamente. La tangente al circuncírculo de ABC en A interseca a LM y LN en P y Q,
respectivamente. Mostrar que CP es paralela a BQ.

Problema 14.8. (Ronda de Correspondencia, Sharygin 2020, P15). Un círculo que pasa por los vértices
B y D del cuadrilátero ABCD corta a AB, BC, CD y DA en K, L, M y N, respectivamente. Un círculo
que pasa por K y M corta a AC en P y Q. Probar que L, N, P y Q son concíclicos.

Problema 14.9. (Balcánica 2015, P2) Sea ABC un triángulo escaleno con incentro I y circuncírculo ω.
Las rectas AI, BI,CI cortan a ω por segunda vez en D, E, F, respectivamente. Las parelelas desde I con
respecto a los lados BC, AC, AB cortan a EF, DF, DE en K, L, M, respectivamente. Probar que los puntos
K, L, M son colineales.

Problema 14.10. (Iberoamericana 2019, P4). Sea ABCD un trapecio con AB ∥ CD, inscrito en una cir-
cunferencia Γ. Sean P y Q dos puntos sobre el segmento AB (A, P, Q, B están en este orden y son
diferentes) tales que AP = QB. Sean E y F los segundos puntos de intersección de las rectas CP y CQ
con Γ, respectivamente. Las rectas AB y EF se cortan en G. Demostrar que DG es tangente a Γ.
136 Teoremas selectos

Problema 14.11. (Lista Corta IMO 2006, G6). Los círculos ω1 y ω2 con centros O1 y O2 son tangentes
externamente en un punto D, y tangentes internamente a un círculo ω en E y F, respectivamente. La
recta t es la tangente común de ω1 y ω2 que pasa por D. Sea AB el diámetro de AB de ω perpendicular a
t, de modo que A, E y O1 están a un mismo lado de t. Demostrar que las rectas AO1 , BO2 , EF y t son
concurrentes.

Problema 14.12. (Campeones Rumanos de Matemática 2016, P1). Sea ABC un triángulo y sea D un
punto sobre el segmento BC, D ̸= B y D ̸= C. El círculo ABD corta al segmento AC de nuevo en el punto
interior E. El círculo ACD corta al segmento AB de nuevo en el punto interior F. Sea A′ la reflexión de
A con respecto a la recta BC. Las rectas A′C y DE se cortan en P, y las rectas A′ B y DF se cortan en Q.
Probar que las rectas AD, BP y CQ son concurrentes (or paralelas).

Problema 14.13. (EE. UU. 2015, Selectivo IMO, P6). Sea ABC un triángulo no equilátero y sean
Ma , Mb , Mc los puntos medios de los lados BC, CA, AB, respectivamente. Sea S un punto sobre la
recta de Euler de ABC. Denótese por X, Y, Z las segundas intersecciones de Ma S, Mb S, Mc S con el
círculo de los nueve puntos. Probar que AX, BY y CZ son concurrentes.
PARTE V

ELEMENTOS DE GEOMETRÍA
PROYECTIVA
CAPÍTULO 15

DIVISIÓN ARMÓNICA

15.1 ¿Qué es la división armónica?


Por mucho, el caso de la razón cruzada con mayor frecuencia en competiciones matemáticas es
(A,C; B, D) = −1. Cuando esta igualdad ocurre, se dice que los puntos A, B, C y D forman una cuarteta
armónica o que A, C son conjugados armónicos de B, D y viceversa1 .

Definición 15.1. Sean A, B, C, D puntos colineales en ese orden. Si (A,C; B, D) = −1 (o equiva-


BA DA
lentemente BC = − DC en términos de longitudes dirigidas), se dice que la cuarteta (A,C; B, D) es
armónica.

Naturalmente, las proposiciones 14.1, 14.2 y los ejercicios de la sección anterior son válidos para
cuartetas armónicas. Su utilidad será visible a lo largo de esta sección.
El siguiente hecho brinda una relación extremadamente instructiva entre cevianas concurrentes y
cuartetas armónicas.

Lema 15.1. Sea APC un triángulo. Los puntos B, E, F yacen en los lados BC, CA, AB, respec-
tivamente. La recta FE intersecta a BC en D. La cuarteta (A,C; B, D) es armónica si y sólo si
AE, PB y CF son concurrentes.

P
F
E

A B C D
Figura 15.1. Cevianas concurrentes involucran cuartetas armónicas.

Demostración. Es una aplicación trivial de los teoremas de Ceva y Menelao. Los detalles son dejados
como ejercicio al lector.

En este momento, coloquemos especial interés al caso en que B coincide con el punto medio del
segmento BC. Es sencillo derivar que EF ∥ AC, por tanto, el punto D no existe en términos euclidianos;
no obstante, si hablamos respecto al plano proyectivo real, dos pares de rectas paralelas se cortan en un
1 Recuerde el ejercicio ejercicio 14.2.
140 División armónica

punto al infinito, simbolizado como P∞ , y se añade la existencia de una recta al infinito que contiene
todos los puntos P∞ . De este modo, puede establecerse el siguiente hecho crucial.

Lema 15.2. D es el punto medio de BC si y solo si los puntos B, C dividen armónicamente a


D, P∞ ; es decir, (B,C; D, P∞ ) = −1.

La deducción anterior nos permite empalmar paralelismos y cuartetas armónicas eficazmente. Véase
el siguiente resultado.

Proposición 15.1. Considere el haz P(A,C; B, D). La paralela a PA por B corta a PB y PD en X y


Y , respectivamente. Luego C es el punto medio del segmento XY si y sólo si (A,C; B, D) = 1.

A B C D
X

Figura 15.2. Cuartetas armónicas y paralelas.

Demostración. De acuerdo al lema 15.2, C es punto medio de XY si y sólo si (X,Y ;C, P∞ ) = −1, así
que P(X,Y ;C, A) = P(X,Y ;C, P∞ ) = −1. Proyectando este haz sobre AD descubrimos que la propiedad
deseada de C sucede si y únicamente si (A,C; B, D) = −1.
Para evitar la labor engorrosa de decir “proyectar” siempre que deseemos considerar los puntos de
P
intersección de un haz P(A,C; B, D) con otra recta ℓ, se utiliza la notación =. En términos compactos,
podemos redactar la demostración anterior como sigue:
P
CX = CY ⇐⇒ −1 = (X,Y ;C, P∞ ) = (A,C; B, D)
Con este ejemplo, la conveniencia de tal símbolo debe quedar más que clara.
¡Ahora es el turno de resolver un ejercicio de olimpiada! De hecho, este ejercicio también lo resolvi-
mos anteriormente (véase el ejemplo 11.1.

Ejemplo 15.1. (Sharygin 2016, Ronda final, Grado 10, P6) Un triángulo ABC es dado. El punto
K es el pie de la bisectriz externa del ángulo A. El punto M es el punto medio del arco AC del
circuncírculo que no contiene a B. El punto N está sobre la bisectriz del ángulo C y cumple que
AN ∥ BM. Probar que M, N y K son colineales.

Solución. Usando los teoremas de las bisectrices interior y exterior en △KBA, es simple deducir que
(K, A; IC , IB ) = −1, donde IC , IB son los C, B-excentros de △ABC, respectivamente. Sea A′ = AN ∩ KI e
I el incentro de △ABC. Obtenemos que:
I
−1 = (K, A; IC , IB ) = (A′ , A; N, P∞ )
15.1 ¿Qué es la división armónica? 141

por consiguiente, el lema 15.2 implica que N es el punto medio de A′ A. Es sabido que M es punto medio
de IIB y dado que AA′ ∥ IIB , se concluye que MN pasa por K.

IB

A
M
IC
N
I
A′
K B C
Figura 15.3. Armónicos y paralelas en acción.

15.1.1 Cuadriláteros armónicos

Definición 15.2. Un cuadrilátero cíclico ABCD es armónico si y sólo si (A,C; B, D) = −1.

Siguiendo la proposición 14.1, si los puntos A, B, C y D son concíclicos en ese orden, la definición
previa implica lo siguiente:

Proposición 15.2. El cuadrilátero cíclico ABCD será armónico si y sólo si los productos de sus
lados opuestos son iguales, es decir AB ·CD = BC · DA.

El próximo lema permite reconocer cuadriláteros armónicos bajo la presencia de tangentes.

Lema 15.3. P es un punto exterior a un círculo ω y sus tangentes tocan a ω en B, D, respec-


tivamente. La recta ℓ pasa por P y corta a ω en A y C de modo que PC > PB y A, B, C, D
están sobre ω en ese orden, entonces ABCD es armónico. Además, si Q = AC ∩ BD, se tiene que
(A,C; Q, P) = −1.

Demostración. Nótese que △PBA ∼ △PCB y △PDA ∼ △PCD, luego:

AB PB PD DA
= = =
BC PC PC CD
B
lo cual implica que ABCD es armónico, por ende (A,C; Q, P) = (A,C; D, B) = −1, donde la recta BB
equivale a la tangente a ω por B. El hecho sigue.

Revirtiendo los pasos, se observa que si ABCD es armónico, las tangentes a ω por B y D deben
concurrir con AC en el cuarto punto armónico P. La misma propiedad vale para las tangentes por A, C y
142 División armónica

A
Q C
P

D
Figura 15.4. El cuadrilátero armónico ABCD.

la recta BD. Más adelante, aprenderemos que BD es la P-polar respecto a ω y P es el BD-polo respecto
a la misma circunferencia.
Reconocer cuadriláteros armónicos suele ser la clave en algunos problemas difíciles. Como muestra,
analicemos el siguiente problema.

Ejemplo 15.2. (Cuenca del Pacífico 2013, P5). Sea ABCD un cuadrilátero cíclico inscrito en ω y
sea P un punto en la prolongación de AC tal que PB y PD son tangentes a ω. La tangente en C
corta a PD en Q y a AD en R. Sea E el segundo punto de corte de AQ y Γ. Probar que B, E y R son
colineales.

B P

Q
E

A D R F

Figura 15.5. Solución del quinto problema de la APMO 2013 mediante cuadriláteros armónicos.

Solución. Se visualiza de inmediato que ABCD y ACED son cuadriláteros armónicos gracias al lema
15.3. Sea R′ = BE ∩ AD y F = BC ∩ AD, entonces:

B C
(A, R′ ; D, F) = (A, E; D,C) = −1 = (A,C; D, B) = (A, R; D; F)

lo cual implica que R = R′ y a su vez la colinealidad deseada.


15.1 ¿Qué es la división armónica? 143

15.1.2 Círculos de Apolonio


Existe una manera sorprendente de conectar cuartetas armónicas con perpendicularidad. Esto lo ex-
plica la próxima premisa.

Lema 15.4. Los puntos A, B, C y D están alineados en este orden. Sea P un punto que no está
sobre la recta AD. Luego, dos de las siguientes condiciones implican la tercera:
(a) (A,C; B, D) = −1.
(b) La recta PB es la bisectriz interior de ∠APC.
(c) La recta DP es perpendicular a PB (i.e. PD es la bisectriz exterior de ∠APC).

Demostración. Simple manipulación de los teoremas de las bisectrices interna externa, junto con el hecho
BA DA
BC = − DC , en términos de longitudes dirigidas.

A B C D

Figura 15.6. El P-círculo de Apolonio de △APC.

¡Y sí; para cualquier punto Q ̸= B, D que esté sobre la circunferencia de diámetro BD se tendrá
que ∠AQB = ∠BQC, por consiguiente, AP : PC = AB : BC = AQ : QC! Esta igualdad no es una mera
casualidad. En efecto, el círculo con diámetro BD recibe el nombre de P-círculo de Apolonio del triángulo
APC y en términos formales, se define como sigue.

Definición 15.3. El P-círculo de Apolonio de △APC es el lugar geométrico de los puntos X tales
que la razón de las distancias de X a los extremos del segmento AC es una constante positiva; es
decir, XA : XC = r; r ∈ R+ .

Es imprescindible mantener en mente que este círculo especial es el circuncírculo del triángulo for-
mado por los pies de las bisectrices externa e interna de ∠APC y el vértice P.
Por otro lado, ¡también podemos conectar potencias con la división armónica!

Lema 15.5. Si (A,C; B, D) = −1 y M es el punto medio de AC entonces:


(a) MA2 = MC2 = MB · MD
(b) DC · DA = DB · DM y,
(c) DB · BM = CB · BA.
144 División armónica

A M B C N D

Q
R
Figura 15.7. Potencia en cuartetas armónicas.

Demostración. Sea P un punto de intersección de los círculos con diámetros AC y BD. Note que ∠APB =
∠BPC = ∠CPD = 45◦ por ser (APC) y (BPD) los P-círculos de Apolonio de △BPD y △APC, respecti-
vamente. Además, es sencillo inferir que:

∠NMP = 2∠DAP = 90◦ − 2∠PDA = 90◦ − ∠PNM

por tanto, MP es tangente a (BPD), de donde surge el resultado (a).


Por otro lado, por el converso del lema 15.3, el segmento que une los puntos de contacto de las
tangentes desde D a (APC) pasa por B. Si Q es uno de tales puntos, tendremos que ∠MBQ = 90◦ =
∠MQD, por lo que DB · DM = DQ2 = DC · DA, por lo que (b) queda probado.
Finalmente, como MP es tangente a (BPD), si R = PB ∩ (APC), R ̸= P se tiene que ∠PRM =
∠MPR = ∠PDM, así que MPDR es cíclico y por tanto DB · BM = PB · BR = CB · BA, como se requería
para (c).
Vale mencionar que los conversos de las propiedades i), ii) y iii) del lema 15.5 también son ciertos; en
otras palabras, si DC · DA = DB · DM, MA2 = MB · MD o DB · BM = CB · BA entonces (A,C; B, D) = −1.
Solamente se invierte la demostración anterior para verificar lo antes dicho.

15.2 Ejemplos

Ejemplo 15.3. (Sharygin 2017, Ronda final, Grado 10, P5). Sean BB′ , CC′ las alturas de un trián-
gulo acutángulo ABC. Dos círculos que pasan por A y C′ son tangentes a BC en puntos P y Q.
Probar que A, B′ , P, Q son concíclicos.

Solución. El eje radical de (AC′ P) y (AC′ Q) es la recta AC′ y biseca a su tangente común BC, esto es,
BP = BQ. Sea A′ el pie de altura desde A a BC. Ya que BP2 = BC′ · BA = BA′ · BC, por el resultado (a)
del lema 15.5 se deduce que (P, Q; B,C) = −1; luego, usando el hecho (b) de la misma proposición se
obtiene que:
CQ ·CP = CA′ ·CB = CB′ ·CA
por ende AB′ QP es cíclico.
15.2 Ejemplos 145

B′

C′

P B A′ Q C
Figura 15.8. Alturas y círculos tangentes a BC.

Ejemplo 15.4. (Iberoamericana 2010, P3). La circunferencia Γ inscrita al triángulo escaleno ABC
es tangente a los lados BC, CA y AB en los puntos D, E y F, respectivamente. La recta EF
corta a la recta BC en G. La circunferencia de diámetro GD corta a Γ en R (R ̸= D). Sean P y
Q (P ̸= R, Q ̸= R) las intersecciones de BR y CR con Γ, respectivamente. Las rectas BQ y CP se
cortan en X. La circunferencia circunscrita a CDE corta al segmento QR en M y la circunferencia
circunscrita a BDF corta al segmento PR en N. Demostrar que las rectas PM, QN y RX son
concurrentes.

R E

F M
N I
P Q
X
G B D C

Figura 15.9. Problema 3 de la Olimpiada Iberoamericana de Matemáticas 2010.

Solución. Las rectas AD, BE y CF concurren en el punto de Gergonne de △ABC, así que por el lema
14.2 se tiene (G, D; B,C) = −1. Dado que ∠GRD = 90◦ , el lema 15.4 permite obtener:2

∠PRD = ∠BRD = ∠DRC = ∠DRQ


2 Reconózcase a (GRD) como el R-círculo de Apolonio de △BRC.
146 División armónica

[ surge que ID es la mediatriz de PQ y se deduce que PQ ∥ BC.


por tanto D es punto medio del arco PDQ,
Sea P∞ el punto al infinito correspondiente para PQ y BC. Ya que R(P, Q; X, P∞ ) = −1, de acuerdo al lema
15.2 se infiere que RX es la R-mediana de △PRQ.
Por otra parte, (CDE) y (BDF) tienen diámetros CI y BI respectivamente, lo cual indica que IM ⊥ RQ
y IN ⊥ PR. Como RP y RQ son cuerdas de Γ, se deduce que M, N son puntos medios de RQ, RP y por
consiguiente, QN, MN, RX concurren en el baricentro de △PRQ.
Se culmina esta sección resolviendo un problema de la lista corta de la IMO 2017.

Ejemplo 15.5. (Lista Corta IMO 2017, G5). Sea ABCC1 B1 A1 un hexágono convexo tal que AB =
BC, y suponga que los segmentos AA1 , BB1 y CC1 tienen la misma mediatriz. Las diagonales AC1
y A1C se cortan en D, y denote por ω al círculo ABC. ω corta al círculo A1 BC1 nuevamente en E
(E ̸= B). Probar que las rectas BB1 y DE se cortan en ω.

Solución. Por hipótesis, AA1 BB1 , BB1C1C y AA1C1C deben ser trapecios isósceles. Según el teorema
del eje radical, AC, A1C1 y BE concurren en un punto J. Por ser BA = BC, la recta BJ es la bisectriz
exterior de ∠AEC. Sea K el punto de corte de la bisectriz interna de ∠AEC con AC; por el lema 15.4
(J, K; A,C) = −1. Veamos que:
AD AA1 AJ AK
= = =
DC1 CC1 CJ KC
i.e. DK ∥ C1C, por tanto ∠KDA = ∠CC1 A = ∠CA1 A = ∠CDK. De este modo, D yace sobre el E-círculo
de Apolonio de △EAC, es decir, EKDJ es cíclico. Mediante manipulación angular simple se demuestra
que BC es tangente a (CEJ); luego, si F = ED ∩ BB1 concluimos que:

∠BCE = ∠CJE = ∠KJE = ∠KDE = ∠BFE

por tanto, F yace sobre ω.

B
E

J A K C

D
A1

B1 C1
Figura 15.10. Trapecios isósceles y círculo de Apolonio.
15.3 Problemas propuestos 147

15.3 Problemas propuestos


Problema 15.1. (Sharygin 2018, Ronda de Correspondencia, P18) Sean C1 , A1 , B1 puntos en los lados
AB, BC, CA del triángulo ABC, tal que AA1 , BB1 , CC1 concurren. Los rayos B1 A1 y B1C1 cortan al
circuncírculo de ABC en A2 y C2 respectivamente. Probar que A, C, el punto común de A2C2 y BB1 y el
punto medio de A2C2 son concíclicos.

Problema 15.2. (Iberoamericana 2015, P4) En el triángulo acutángulo ABC, el punto D es el pie de la
perpendicular desde A sobre el lado BC. Sea P un punto en el segmento AD. Las rectas BP y CP cortan
a los lados AC y AB en E y F respectivamente. Sean J y K los pies de las perpendiculares desde E y F
sobre AD respectivamente. Demostrar que

FK FJ
=
KD JD

Problema 15.3. (México 2015, P5) Sea I el incentro de un triángulo acutángulo ABC. La recta AI corta
por segunda vez al circuncírculo del triángulo BIC en E. Sean D el pie de la altura desde A sobre BC y J
la reflexión de I con respecto a BC. Mostrar que los puntos D, J y E son colineales.

Problema 15.4. (Cuenca del Pacífico, Problema 4) Sea ABC un triángulo acutángulo. Denote por D el
pie de altura desde A, M el punto medio de BC y H el ortocentro de ABC. Sea E el punto de intersección
del circuncírculo Γ de ABC con la semirrecta MH, y F el punto de intersección distinto de E de la recta
ED y Γ. Probar que
BF AB
=
CF AC
Problema 15.5. (Lista Corta IMO 2004, G8) Sea ABCD un cuadrilátero cíclico, M el punto medio del
lado CD y N un punto sobre el circuncírculo del triángulo ABM, tal que N ̸= M y AN AM
BN = BM . Si E =
AC ∩ BD, F = BC ∩ AD, demostrar que E, F, N son colineales.

Problema 15.6. (Lista Corta IMO 2002, G7) El incírculo Ω del triángulo acutángulo ABC es tangente a
BC en K. Sea AD una altura de △ABC y sea M el punto medio de AD. Si N es el otro punto común de Ω
y KM, probar que Ω y el circuncírculo de △BCN son tangentes en N.

Problema 15.7. (IMO 2010, Problema 4, generalizado) El punto P yace dentro del △ABC. Las rectas
AP, BP, CP cortan al circuncírculo del △ABC en K, L, M, respectivamente. La tangente al circuncírculo
en C corta a AB en S. Probar que SC = SP si y sólo si MK = ML.

Problema 15.8. ([Link]. 2017, Selectivo IMO, P2) Sea ABC un triángulo con altura AE. El A-excírculo
toca al lado BC en D, y corta al circuncírculo en dos puntos F y G. Probar que es posible seleccionar
puntos V y N en las rectas DG y DF tales que el cuadrilátero EVAN es un rombo.

Problema 15.9. ([Link]. 2018, P5) En el cuadrilátero cíclico convexo ABCD, se sabe que las rectas AC
y BD se cortan en E, las rectas AB y CD se intersecan en F, y las rectas BC y DA se encuentran en
G. Suponga que el circuncírculo de △ABE corta a la recta CB en B y P, y el circuncírculo de △ADE
interseca a la recta CD en D y Q, donde C, B, P, G y C, Q, D, F son colineales en ese orden. Probar que
si las rectas FP y GQ se cortan en M, entonces ∠MAC = 90◦ .
148 División armónica

Problema 15.10. (Vietnam 2018, P2) Sea ABC un triángulo escaleno y un punto D en el lado BC. Se
escogen puntos E y F en los lados AB y AC tales que ∠DEB = ∠DFC, respectivamente. Las rectas DF y
DE intersecan a AB y AC en M y N, respectivamente. Denote por (I1 ) e (I2 ) los circuncírculos de △DEM
y △DFN en este orden. El círculo (J1 ) toca internamente a (I1 ) en D y a AB en K. El círculo (J2 ) toca
internamente a (I2 ) en D y a AC en H. La segunda intersección de (I1 ) e (I2 ) es P. La segunda intersección
de (J1 ) y (J2 ) es Q.
i) Probar que D, P y Q se encuentran sobre una misma línea.
ii) Los circuncírculos de △AEF y △AHK se intersecan en A y G. El círculo (AEF) también corta a
AQ en A y L. Probar que la tangente a (DQG) por D encuentra a EF en un punto sobre (DLG).
CAPÍTULO 16

POLOS Y POLARES

16.1 Introducción

Definición 16.1. Sea Γ una circunferencia centrada en O, con radio r y un punto A ̸= O coplanar
con Γ. Construyamos un punto A′ sobre el rayo OA tal que OA · OA′ = r2 . La recta ℓ que pasa por
A′ y es perpendicular a OA se nombra como la A-polar con respecto a Γ. A su vez, A se llama el
ℓ-polo con respecto a Γ.

O A′ A O A A′

ℓ ℓ

Figura 16.1. La A-polar y el ℓ-polo respecto a Γ.

La construcción de la A-polar es sencilla. En caso de que A esté fuera de Γ, ℓ es la recta que une los
puntos de contacto de las tangentes a Γ trazadas desde A con esta misma circunferencia, mientras que
A′ = ℓ ∩ OA. Al contrario, si A yace en el interior de Γ, la construcción se revierte: A′ se define como el
punto de intersección de las tangentes a Γ que pasan por los puntos donde la perpendicular a OA por A
cortan a Γ.
Queda como ejercicio al lector corroborar que ambas construcciones verdaderamente cumplen que
OA · OA′ = r2 . Es evidente que cuando A yace sobre Γ entonces A = A′ y su polar corresponde a la
tangente a Γ que pasa por él mismo.

16.2 Resultados útiles

Teorema 16.1 (La Hire). Sea B otro punto coplanar con Γ, diferente de A y O. Si A yace en la
B-polar, entonces B está sobre la A-polar.
150 Polos y polares

P
B′

O A′ A

Figura 16.2. Teorema de La Hire

Demostración. Sea P un punto sobre Γ tal que PB es tangente a Γ, B′ = OB ∩ PA y A′ el pie de la


perpendicular por B a OA. Note que OA′ BP y A′ ABB′ son cuadriláteros cíclicos, de modo que:

∡PAO = ∡B′ AA′ = ∡B′ BA′ = ∡OBA′ = ∡OPA′

lo cual implica que OP es tangente al círculo (PAA′ ), luego r2 = OP2 = OA · OA′ , por tanto BA′ coincide
con la A-polar.

A pesar de su simplicidad, el teorema de La Hire es uno de los hechos más valiosos para la resolución
de problemas concernientes a concurrencia y/o colinealidad. Otro teorema relevante es el siguiente.

Teorema 16.2 (Brocard). Sea O el circuncentro del cuadrilátero cíclico ABCD. Los puntos P, Q y
R son los puntos de intersección de los pares de rectas AC, BD; AB, CD y BC, AD, respectivamen-
te. Entonces O es el ortocentro de △PQR. Además, QR, PQ y PR son las polares correspondientes
a P, R, Q.

R
A

B
P

D C Q

Figura 16.3. Teorema de Brocard


16.2 Resultados útiles 151

Demostración. De acuerdo al lema 14.2, conocemos que R(D,C; P, Q) = −1, por tanto:

(A, B; RP ∩ AB, Q) = −1 = (D,C; RP ∩ DC, Q)

Por el inverso del lema 15.3, se concluye que RP ∩ AB y RP ∩ DC están sobre la Q-polar respecto a
(ABCD), así que PR es tal polar. De forma similar podemos mostrar que QP y RQ son las R, P-polares
respecto a (ABCD), respectivamente. Recordando la definición 16.1, se infiere que OP ⊥ QR, OQ ⊥ RP
y OR ⊥ PQ, por lo que O es el ortocentro de △PQR.

Definición 16.2. Un triángulo es llamado autopolar si cada lado es la polar del vértice opuesto
respecto a un mismo círculo de referencia.

Siguiendo la definición anterior, es claro que △PQR es autopolar. Además, O, P, Q y R forman un


grupo ortocéntrico, pues cada uno de ellos es el ortocentro del triángulo formado por los demás tres
puntos. El teorema de Brocard no descubre otra cuestión más que los cuadriláteros cíclicos brindan
polares escondidas, ángulos rectos y cuartetas armónicas de forma natural. Considerar los puntos de
intersección de los pares de lados opuestos y de las diagonales puede ser el truco para arribar a una
solución. El próximo problema es un ejemplo perfecto de lo antes mencionado.

Ejemplo 16.1. (Iberoamericana 2016, P3). Sea ABC triángulo acutángulo cuya circunferencia cir-
cunscrita es Γ. Las tangentes a Γ por B y C se cortan en P. Sobre el arco AC
c que no contiene a B
se toma un punto M, distinto de A y de C, tal que la recta AM corta a la recta BC en K. Sean R el
punto simétrico de P con respecto ala recta AM y Q el punto de intersección de las recta RA y PM.
Sean J el punto medio de BC y L el punto donde la recta paralela por A a la recta PR corta a la
recta PJ. Demostrar que los puntos L, J, A, Q y K están sobre una misma circunferencia.

A
L Q
R

B J F C K
D
E

P
Figura 16.4. Un problema clásico de polares.

Solución. Es claro que ∠LAK = ∠LJB = 90◦ , de modo que ALJK es inscrito. Sean E = AP ∩ Γ, E ̸=
A, D = KE ∩ Γ, D ̸= E y F = ME ∩ AD. Note que BC es la P-polar respecto a Γ. Como K yace sobre BC
152 Polos y polares

y la K-polar pasa por F según el teorema de Brocard, entonces PF es tal K-polar, por tanto M, D y P son
colineales. También por Brocard, KF es la P-polar, así que F está sobre BC. Derivamos que:

∠KDQ = ∠EAM = ∠PAK = ∠KAQ

por tanto ADKQ también es cíclico.


Usando la propiedad (c) del lema 15.5 se consigue que KF ·FJ = CF ·FB = DF ·FA, por lo que AJDK
es cíclico. En conclusión, ALJK, ADKQ y AJDK son cuadriláteros inscritos, por lo que A, L, J, D, K, Q
son concíclicos.
Un hecho a primera vista trivial pero de enorme utilidad es el descrito a continuación.

B
P

ℓA
C
O ℓC

ℓB

Figura 16.5. Dualidad concurrencia-colinealidad bajo la transformación polar.

Proposición 16.1. Las polares de puntos coplanares dados respecto a un círculo Γ son concurren-
tes si y sólo tales puntos son colineales.

Demostración. Es una aplicación directa del teorema de La Hire.


La proposición anterior expone manifiestamente el concepto de dualidad entre rectas y puntos: la co-
linealidad entre algún conjunto de puntos puede ser intercambiada a la concurrencia de rectas y viceversa
mediante la transformación polar. Es una idea simple pero tremendamente provechosa al momento
de resolver problemas que involucren los dos conceptos antes mencionados. Como muestra, veamos el
siguiente ejemplo1 .

Ejemplo 16.2. (Sharygin 2017, Ronda de Correspondencia, P23). Una recta m es tangente al
incírculo del △ABC. Las rectas que pasan por el incentro I y son perpendiculares a AI, BI, CI
cortan a m en A′ , B′ , C′ , respectivamente. Probar que AA′ , BB′ , CC′ concurren.

Solución. Considere la transformación polar respecto al incírculo de △ABC y sea DEF el triángulo
tangencial de △ABC como se visualiza en la figura previa. Definamos a P como el punto común de m y
el incírculo. Por la proposición 16.1, es suficiente probar que los polos correspondientes de AA′ , BB′ y
1 Los teoremas de Pascal y Brianchon constituyen un ejemplo claro del concepto dual, así como Ceva y Menelao.
16.3 Polares y círculos ortogonales 153

A′
A

B′
F P
S C′
R
T
I
E

B D C
Figura 16.6. Polares concurrentes y recta de Simson.

CC′ , digamos R, S y T , están alineados. Por el teorema de La Hire, R está sobre EF. Además, PR es la
A′ -polar, por lo que PR ⊥ A′ I, pero EF ∥ A′ I, así que PR ⊥ EF. De modo similar, PS ⊥ DF, PT ⊥ DE
y S ∈ FD, T ∈ DE. De acuerdo al teorema de Simson obtenemos que R, S, T yacen sobre la P-recta de
Simson respecto a △DEF. La conclusión sigue.

16.3 Polares y círculos ortogonales

Definición 16.3 (Círculos ortogonales). Si los radios de dos círculos ω1 y ω2 que unen un mismo
punto de intersección son perpendiculares, se dice que ω1 y ω2 son ortogonales.

O1 Q O2

R ω2
ω1
Figura 16.7. Un par de círculos ortogonales

En otras palabras, la definición anterior explica que para un par de círculos ortogonales, los radios
O1 P y O2 P son tangentes a ω2 y ω1 , respectivamente, como se revela en la figura 16.7. Particularmente,
esto nos brinda una caracterización alternativa bastante obvia de circunferencias ortogonales.
154 Polos y polares

Proposición 16.2. Dos círculos ω1 y ω2 son ortogonales si y sólo si r22 = O2 P2 = O2 Q · O2 R,


donde r2 es el radio de ω2 , O2 es el centro de ω2 y Q, R son puntos sobre ω1 tales que Q, R y O2
están alineados.

Posiblemente, se reconozca que se ha trabajado con circunferencias de este tipo previamente. En


efecto, en el lema 15.5, los círculos (APC) y (BPD) son ortogonales.

O A′ A

Figura 16.8. Polares y círculos ortogonales

La exposición anterior permite establecer una propiedad sublime que involucra polares con la orto-
gonalidad de círculos.

Lema 16.1. Suponga que un punto B yace sobre la A-polar respecto al círculo Γ. Entonces el
círculo de diámetro AB es ortogonal a Γ.

Demostración. Sea A′ el punto de corte de la A-polar con OA y r el radio de Γ. Es claro que ∠BA′ A = 90◦ ,
así que A′ está sobre el círculo de diámetro AB. Como OA′ · OA = r2 por la definición 16.1, siguiendo la
proposición 16.2 se infiere el resultado.
Resolvamos el problema a continuación mediante esta idea.

Ejemplo 16.3. (Irán 2018, Selectivo IMO, P3). En el triángulo ABC sea M el punto medio del
lado BC. Sea ω un círculo interior a △ABC y tangente a AB y AC and E y F, respectivamente.
Las tangentes desde M a ω cortan a este círculo en P y Q, tal que B y P están al mismo lado de
AM. Sean X e Y los puntos de intersección de las rectas PM, BF y QM, CE, respectivamente. Si
BC = 2PM, probar que XY es tangente a ω.

Solución. La condición BC = 2PM implica que P y Q se encuentran sobre la circunferencia con centro
M y radio BM = MC.
Sean D = BP ∩CQ y H = CP ∩ BQ. Como D está sobre la H-polar respecto a (BPQC) por el teorema
de Brocard y ∠HPD = ∠HQD = 90◦ , gracias al lema 16.1 se deduce que (DPHQ) y (BPCQ) son
ortogonales. Observe que ω y (BPCQ) también son ortogonales, pero es simple asegurarse que para cada
cuerda PQ sobre (BPCQ) que no sea un diámetro, existe un sólo círculo ortogonal a este que pasa por P
y Q, lo cual implica que ω y (DPHQ) deben coincidir; es decir, D y H yacen sobre ω.
Según el teorema de Brocard, la C-polar respecto a ω pasa por B y es claro que también pasa por
F, así que BF es tal C-polar. Como PX es tangente a ω, se sigue que CP es la X-polar respecto a ω de
16.4 Más ejemplos 155

A
D
F
E Q

P
X H Y

B M C
Figura 16.9. Problema 3 del Selectivo IMO 2018 de Irán.

acuerdo al teorema de La Hire. Similarmente, BP es la Y -polar, por ende, H es el polo de XY . Puesto que
H yace sobre ω, XY debe ser tangente a ω en H.

16.4 Más ejemplos

Ejemplo 16.4. (Sharygin 2017, Ronda Final, Décimo grado, P7). Un cuadrilátero ABCD está cir-
cunscrito alrededor del círculo ω centrado en I y está inscrito en el círculo Γ.a Las rectas AB y CD
se cortan en el punto P, mientras que las rectas BC y AD se encuentran en el punto Q. Mostrar que
los círculos PIQ y Γ son ortogonales.
a Un cuadrilátero cíclico que a la vez contiene un incírculo recibe el nombre de cuadrilátero bicéntrico.

I
K

P D C
Figura 16.10. El cuadrilátero bicéntrico ABCD.
156 Polos y polares

Solución. Por el teorema de Brocard, sabemos que P está sobre la Q-polar respecto a Γ; de este modo,
basta con probar que ∠QIP = 90◦ tomando en cuenta al lema 16.1.
Como I es el incentro de △DQC, obtenemos que ∠QID = 90◦ + ∠C 2 ; sin embargo, observemos que:
◦ ∠A+∠D
∠AID = 180 − 2 y que además, al ser ABCD cíclico:
∠B + ∠C ∠A + ∠D
∠ABI + ∠DCI = = 180◦ −
2 2
i.e. ∠AID = ∠ABI + ∠DCI. Luego, si trazamos la tangente ℓ a (AIB) por I y consideramos un punto K
sobre ℓ en el lado opuesto a BC obtendremos que:
∠KID = ∠AID − ∠AIK = ∠AID − ∠ABI = ∠DCI
por tanto, ℓ también es tangente a (CID). Concluimos que (AIB) y (CID) son tangentes externamente en
I y que ℓ es su eje radical, pero P tiene igual potencia a estos dos círculos, así que P está sobre ℓ, es decir
∠PID = ∠C ◦
2 , por ende ∠QIP = 90 .

Ejemplo 16.5. (Iraní de Geometría 2016, Nivel Avanzado, P4). En un cuadrilátero convexo ABCD,
las rectas AB y CD se cortan en el punto E y las rectas AD y BC se cortan en el punto F. Sea P el
punto de intersección de las diagonales AC y BD. Suponga que ω1 es un círculo que pasa por D y
es tangente a AC en P. También suponga que ω2 es un círculo que pasa por C y es tangente a BD
en P. Sea X el punto de intersección de ω2 y BC. Suponga que los círculos ω1 y ω2 se intersecan
en Q por segunda vez. Demostrar que la perpendicular por P a la recta EF pasa por el circuncentro
del triángulo XQY

B
Y

A
X Y′
P
X′ O
G

E D C
Q

Figura 16.11. Cuarto problema del nivel avanzado de la IGO 2016

Solución. Denotemos por Y ′ el punto común de PX, BC y X ′ el de Y P y AD. Notemos que:


∠Y X ′ X = ∠PX ′ X = ∠PDX + ∠X ′ PD = ∠PQX + ∠Y QP = ∠Y QX
Análogamente, podemos conseguir que ∠YY ′ X = ∠Y QX, lo cual implica que X ′ y Y ′ están sobre el
círculo (XQY ). Sea G = XY ∩ X ′Y ′ . Sabemos que F(G, P; X,Y ) = −1 = F(E, P; D,C) según el lema
16.5 Problemas propuestos 157

15.1, así que E, G y F están alineados. Además, por el teorema de Brocard GF es la P-polar respecto
a (XQY ); por consiguiente, siendo O el circuncentro de △XQY podemos concluir que OP ⊥ EF de
acuerdo a la definición 16.1.

16.5 Problemas propuestos


Problema 16.1. (Balcánica 2018, P1) Un cuadrilátero ABCD está inscrito en un círculo k con AB > CD,
y AB ∦ CD. El punto M es la intersección de las diagonales AC y BD, y la perpendicular desde M a AB
corta a AB en un punto E. Si EM biseca a ∠CED, probar que AB es un diámetro de k.

Problema 16.2. (Corea del Sur 2018, P4) El triángulo ABC satisface ∠C = 90◦ . Un círculo que pasa por
A y B corta a AC en G (̸= A, C) e interseca a BC en D (̸= B). AD corta a BG en H y la mediatriz de
AD corta a la mediatriz de AB en E. Una recta que pasa por D es perpendicular a DE y corta a ℓ en F.
Si el circuncírculo de △CFH interseca a AC y BC en P (̸= C) y Q (̸= C) respectivamente, probar que
PQ ⊥ FH.

Problema 16.3. (Centroamericana 2015, P3) Sea ABCD un cuadrilátero cíclico con AB < CD, y sea P
el punto de intersección de la rectas AD y BC. El circuncírculo del triángulo PCD corta a la recta AB en
puntos Q y R. Sean S y T los puntos donde las tangentes desde P al circuncírculo del ABCD tocan a dicha
circunferencia.
i) Probar que PQ = PR.
ii) Mostrar que QRST es un cuadrilátero cíclico.

Problema 16.4. El incírculo del △ABC toca a los lados BC, CA, AB en D, E, F, respectivamente. Sea
I el incentro de ABC y M el punto medio de BC. Demuestra que EF, DI, AM concurren.

Problema 16.5. (Sharygin 2017, Ronda de Correspondencia, P15) Sea ABC un triángulo acutángulo
con incírculo ω e incentro I. ω toca a AB, BC, CA en D, E, F respectivamente. Los círculos ω1 y ω2
centrados en J1 y J2 , respectivamente están inscritos en ADIF y BDIE. Sea M = J1 J2 ∩ AB. Probar que
CD ⊥ IM.

Problema 16.6. Sea ABC un triángulo, y sean D, E, F los puntos de contacto de su incírculo ω con los
lados BC, CA y AB, respectivamente. Sea K el punto de intersección de AD con ω distinto de D, y sea M
el punto de intersección de EF con la perpendicular a AD que pasa por K. Mostrar que AM ∥ BC.

Problema 16.7. (Cuenca del Pacífico 2016, P3) Sean AB y AC dos rayos distintos que no yacen en una
misma recta, y sea ω una circunferencia con centro O que es tangente al rayo AC en E y al rayo AB en
F. Sea R un punto sobre el segmento EF. La recta que pasa por O paralela a EF corta a AB en P. Sean
N = PR ∩ AC, y M el punto de intersección de AB con la paralela por R paralela a AC. Probar que MN es
tangente a ω.

Problema 16.8. ([Link]. 2018, Preselectivo IMO, P3) Sea ABC un triángulo agudo con incentro I,
circuncentro O y circuncírculo Γ. Sea M el punto medio de AB. El rayo AI corta a BC en D. Denote por ω
y γ los circuncírculos de △BIC y △BAD, respectivamente. La recta MO interseca a ω en X e Y , mientras
que la recta CO corta a ω en C y Q. Asuma que Q yace en el interior de △ABC y ∠AQM = ∠ACB.
Considere las tangentes a ω por X e Y y las tangentes a γ por A y D. Dado que ∠BAC ̸= 60◦ , probar que
estas cuatro rectas concurren en Γ.
158 Polos y polares

Problema 16.9. ([Link]. 2016, Preselectivo IMO, P2) Sea ABC un triángulo escaleno con ortocentro H
y circuncentro O. Denote por M, N los puntos medios de AH, BC. Suponga que el círculo γ con diámetro
AH corta al circuncírculo de ABC en G ̸= A, y corta a la recta AN en Q ̸= A. La tangente a γ por G corta
a la recta OM en P. Demostrar que los circuncírculos de △GNQ y △MBC se intersecan en un punto T
sobre NP.

Problema 16.10. ([Link]. 2013, Selectivo IMO, P3) Sea ABC un triángulo escaleno con ∠BCA = 90◦ y
D el pie de altura desde C. Sea X un punto sobre el segmento CD. Sea K un punto sobre el segmento AX
tal quue BK = BC. Similarmente, sea L un punto sobre el segmento BX tal que AL = LC. El circuncírculo
del triángulo DKL corta al segmento AB en un segundo punto T ̸= D. Demostrar que ∠ACT = ∠BCT .

Problema 16.11. (Iraní de Geometría 2018, Nivel Avanzado, P4) El cuadrilátero ABCD está circunscrito
alrededor de un círculo. Las diagonales AC y BD no son perpendiculares. Las bisectrices de los ángulos
entre estas diagonales intersecan a AB, BC, CD y DA en puntos K, L, M y N, respectivamente. Dado que
KLMN es cíclico, probar que también lo es ABCD.
CAPÍTULO 17

INVERSIÓN

17.1 Primeros hechos

Definición 17.1. Dada una circunferencia Γ de centro O y radio r, la inversión f de centro O y


radio r es una transformación del plano que asigna a cada punto A ̸= O otro punto A′ , de modo que
A′ está sobre el rayo OA y satisfacen la siguiente relación fundamental:

OA · OA′ = r2 (17.1)

Los puntos A, A′ son llamados inversos respecto a Γ e imágenes uno del otro bajo f , i.e. f (A) = A′ .

Particularmente, el inverso de O es el punto al infinito1 P∞ . Observe que la definición 16.1 se traduce


en que la A-polar es la perpendicular a OA por el inverso de A. La construcción del A-inverso debe ser
clara a estas alturas.

O A′ A

Figura 17.1. Un par de puntos inversos

Gracias a la expresión 17.1, es sencillo convencerse que si A es un punto exterior a Γ, entonces A′


yace es interno a Γ y viceversa. Además, A es su propio inverso si y sólo si yace sobre Γ.
Una característica fundamental de la inversión es que es una involución; es decir, el inverso de A′ es
A. Dicho de otra forma, f ( f (A)) = A.
¿Por qué invertir un problema? Pues bien, al aplicar una inversión bajo las condiciones “adecuadas”
que abordaremos en lo que sigue, obtendremos una versión alternativa más asequible de tratar y resol-
ver. Como la inversión es involutiva, al invertir hacia atrás el ejercicio alterno resuelto obtendremos
2 2
1 Por convención, r∞ = 0 y r0 =∞. Se asume que cada recta y ningún círculo en el plano pasa por este punto. En este caso,
la definición de P∞ difiere de la dada en el capítulo anterior, donde a cada clase de rectas paralelas le corresponde un punto al
infinito.
160 Inversión

el problema original, mismo que también debe ser verdadero; por consiguiente, basta con solucionar el
problema inverso.
En lo que sigue, indagaremos lo que ocurre al invertir rectas y círculos; además, denotaremos por X ′
al inverso de X.

17.2 Rectas y círculos

Proposición 17.1 (Inversión y rectas). Sea ℓ una recta. Luego,


i) si ℓ pasa por O, entonces f (ℓ) = ℓ, i.e. ℓ es su propio inverso.
ii) si ℓ no pasa por O, su imagen es una circunferencia γ que pasa por O y tiene diámetro OA′ ,
donde A′ es el inverso de la proyección ortogonal de O (digamos A) sobre ℓ.

B
B′

O A′ A

Figura 17.2. La imagen de una recta que no pasa por O pasa por O.

Demostración. El inciso i) es trivial y se deja como ejercicio. Para el hecho ii), sea B ̸= A un punto sobre
ℓ y B′ su inverso. Se tiene que OB · OB′ = r2 = OA · OA′ , luego ABB′ A es cíclico y entonces ∠OB′ A′ =
∠BAO = 90◦ , por tanto B está sobre γ. Debido a que B es un punto arbitrario sobre ℓ, lo anterior debe
cumplirse para cualquier punto en ℓ y la conclusión sigue.
El converso de la proposición 17.1 también es cierto. De hecho,

Proposición 17.2. La circunferencia γ pasa por O, entonces su inverso es una recta.

B′
B

O A A′

Figura 17.3. La imagen de un círculo que pasa por el centro de inversión es una recta.
17.2 Rectas y círculos 161

Demostración. Sea A un punto sobre γ tal que OA es un diámetro, y sea A′ = f (A). Escoja un punto arbi-
trario B ̸= A sobre γ y defina a B′ como su inverso. Como en la prueba anterior, se infiere que A′ ABB′ es
cíclico, de donde surge que ∠B′ A′ O = ∠OBA = 90◦ , así que el lugar geométrico de B es la perpendicular
a OA por A′ .

La proposición 17.2 conforma uno de los incentivos más fuertes para invertir: considerar una inver-
sión con centro en un punto por el que pasen varios círculos los transformará en rectas; y ciertamente,
las rectas son mucho más manejables que las circunferencias. En caso que γ no pase por O, su imagen
bajo f es otra circunferencia2 .

Proposición 17.3. Sea AB un diámetro del círculo γ tal que O, A, B están alineados. Si γ no pasa
por O, entonces f (γ) es otro círculo γ ′ con diámetro A′ B′ , con A′ = f (A) y B′ = f (A).

P′

O A B B′ A′

Figura 17.4. El inverso de un círculo que no pasa por O es otro círculo.

Demostración. Considere un punto arbitrario P (̸= A, B) sobre γ y defina a P′ como su inverso. Es claro
que OP · OP′ = OA · OA′ = OB · OB′ = r2 , por ende APP′ A′ y BB′ PP′ son cuadriláteros inscritos. Usando
ángulos dirigidos, se deriva que:

∡B′ P′ A′ = ∡B′ A′ P′ + ∡P′ B′ A′ = ∡APO + ∡P′ PB = 90◦

lo cual significa que P′ yace sobre el círculo de diámetro A′ B′ , como se deseaba.

El siguiente resultado es, sin duda alguna, imprescindible al abordar inversión en un problema que
involucre figuras tangentes.

Proposición 17.4. La inversión preserva intersecciones y tangencias.

Lo primero es evidente; de este modo, si dos objetos λ1 y λ2 son tangentes en A, necesariamente A′


debe ser el único punto común de f (λ1 ) y f (λ2 ) por. En caso de que A coincida con el centro de inversión,
entonces A′ es P∞ . Un ejemplo de lo anterior son dos circunferencias tangentes en O, cuyas imágenes son
dos rectas paralelas (¿por qué?) cuyo único punto común es P∞ (consulte la figura 17.5).
2 ¡Así qué chiste!
162 Inversión

O λ1 λ2

f (λ2 ) f (λ1 )

Figura 17.5. Los inversos de dos círculos tangentes en O son dos rectas paralelas.

Ejercicios

Lema 17.1 (La inversión fija circunferencias ortogonales). Mostrar que, dadas dos circunferencias
ortogonales ω1 y ω2 , entonces ω1 es su propio inverso con respecto a ω2 y viceversa.

Problema 17.1. Pruebe que si γ y γ ′ son dos círculos inversos respecto a un punto O, entonces O es el
exsimilicentro de γ y γ ′ .
Problema 17.2. Dado un triángulo ABC, demostrar que:
i) Si H es su ortocentro y D, E, F son √los pies de alturas desde A, B, C, respectivamente, entonces
la inversión con centro A y radio r = AH · AD transforma F a B y E a C.
ii) Si DEF es su triángulo tangencial, la inversión respecto al incírculo envía (ABC) al círculo de los
nueve puntos de △DEF.

17.3 Un primer ejemplo


A estas alturas, resolver un problema mediante las proposiciones antes abordadas sería esclarecedor.
En efecto, centremos nuestra atención en el ejercicio a continuación.

Ejemplo 17.1. (Sharygin 2016, Ronda de Correspondencia, P17). Sea D un punto arbitrario sobre
el lado BC del triángulo ABC. Los círculos ω1 y ω2 pasan por A y D de modo que BA es tangente a
ω1 y CA es tangente a ω2 . Sea BX la segunda tangente desde B hasta ω1 y CY la segunda tangente
desde C hasta ω2 . Probar que BC es tangente al circuncírculo del triángulo XDY .

Proveamos motivación para invertir y recorramos detalladamente las transformaciones necesarias.


Primero, observemos que invertir con centro A parece una buena estrategia, pues es un punto común
de ω1 y ω2 , al mismo tiempo en que podríamos aprovechar que las rectas AB y AC quedan fijas en
consonancia a la proposición 17.1. En efecto, por la proposición 17.2, las imágenes de ω1 y ω2 son dos
rectas ℓ1 y ℓ2 tales que ℓ1 ∥ AB y ℓ2 ∥ AC, respectivamente. Además, puesto que la inversión conserva
intersecciones, el inverso de D es el punto de corte de estas rectas, i.e. D′ = ℓ1 ∩ ℓ2 .
Evidentemente, B′ y C′ yacen sobre AB y AC. Examinemos qué ocurre con X ′ y Y ′ . En primer lugar,
X ′ está sobre ℓ2 . Dado que BB′ X ′ X es inscrito y ∠BAX = ∠AXB, entonces ∠X ′ B′ A = ∠B′ AX ′ , en otras
palabras, X está sobre la mediatriz de AB′ . Similarmente, Y ′ yace en la mediatriz de AC′ .
Por otra parte, la imagen de BC es el círculo (AB′C′ ) según la proposición 17.1. Siendo así, la aserción
“BC tangente a (XDY )” se traduce a “círculos (AB′C′ ) y (X ′ D′Y ′ ) son tangentes en D′ ” de acuerdo a la
17.3 Un primer ejemplo 163

B D C

Y
Figura 17.6. El círculo (XDY ) es tangente a BC.

proposición 17.4. Debemos tomar en cuenta otro detalle: como D está sobre el segmento BC, necesaria-
mente D′ debe encontrarse sobre el arco Bd
′C′ opuesto a A en (A′ B′C′ ).

C′
Y′ X′
B′ D′
B D C

Y
Figura 17.7. Invirtiendo en A al problema 17.1.

Hemos obtenido las imágenes de cada punto dado en el problema, así que es momento de resolver el
problema inverso. Por claridad, enunciemos separadamente el ejercicio alternativo obtenido.

Enunciado inverso. Dado un △AB′C′ , sea D′ un punto arbitrario en BC


d′ que no contiene a A en el círculo
(AB C ). Sean X e Y puntos tales que X D ∥ AB , Y D ∥ AC y X A = X ′ B′ , Y ′ A = Y ′C′ . Demostrar que
′ ′ ′ ′ ′ ′ ′ ′ ′ ′

los círculos (AB′C′ ) y (X ′ D′Y ′ ) son tangentes en D′ .

La inversión nos ha permitido llegar al corazón del problema, pues la nueva versión es completamente
trivial. Sea O el circuncentro de △AB′C′ . Como X ′ está sobre la mediatriz de AB′ y D′ X ′ ∥ AB′ , entonces
∠OX ′ D′ = 90◦ . Análogamente, ∠D′Y O = 90◦ , por tanto (X ′ D′Y ) tiene diámetro OD′ . La conclusión es
inmediata. Redactemos la solución de forma compacta.

Solución al ejercicio 17.1. Apliquemos una inversión con centro A y radio unitario. En el problema
inverso, X ′ e Y ′ son puntos sobre las mediatrices de AB′ y AC′ , respectivamente, y D′ un punto en el arco
164 Inversión

O
X′

B′ C′
Y′

D′
Figura 17.8. Diagrama inverso del ejercicio 17.1

′C′ que no contiene a A en el circuncírculo de △AB′C′ , tal que D′ X ′ ∥ AB′ y D′Y ′ ∥ AC′ . Es suficiente
Bd
probar que (AB′C′ ) y (X ′ D′Y ′ ) son tangentes en D′ .
Defina a O como el centro de (AB′C′ ). Las condiciones anteriores implican que OX ′ ⊥ DX ′ y OY ′ ⊥
D′Y ′ , por tanto, O, X ′ , D′ , Y ′ yacen sobre un círculo con diámetro OD′ . De este modo, D, O y el circun-
centro de △X ′ D′Y ′ son colineales, lo cual asegura la tangencia precisada.

Comúnmente, no es necesario incurrir en todos los detalles relativos a la obtención del problema
alternativo. Basta con reescribir las hipótesis proporcionadas en el enunciado original.

17.4 Longitudes y ángulos


Usualmente, poco puede decirse acerca de lo que sucede con las longitudes al invertir. La siguiente
propiedad puede ser de interés.

Proposición 17.5 (Fórmula de la distancia). Dados dos puntos A, B y sus inversos correspondien-
tes A′ , B′ , se tiene que:
A′ B′ r2
=
AB OA · OB

Demostración. Simple manipulación de longitudes tomando en cuenta que △AOB ∼ △B′ OA′ .

Este hecho puede resultar útil al aplicar inversión a problemas donde hay condiciones expresables en
razones. El ejemplo a continuación es un resultado clásico.
17.4 Longitudes y ángulos 165

Teorema 17.1 (Ptolomeo). Cuatro puntos A, B, C y D son concíclicos en ese orden si y sólo si se
cumple la igualdad

AB ·CD + AD · BC = AC · BD (17.2)

i.e. la suma de los productos de lados opuestos equivale al producto de las diagonales.

A
B

A′
B′

D C′ C

Figura 17.9. Teorema de Ptolomeo.

Demostración. Consideremos la inversión con centro D y radio r > 0. Multiplicando la expresión dada
r2
por AD·BD·CD , se obtiene que 17.2 es cierta si y sólo si:

r2 r2 r2
AB · + BC · = AC · (17.3)
AD · BD AD ·CD AD ·CD
Sean A′ , B′ , C′ los inversos de A, B, C, respectivamente. Por la proposición 5, la expresión 17.3 implica
que A′ B′ + B′C′ = A′C′ , lo cual es verdadero si y sólo si A′ , B′ , C′ son colineales según la desigualdad
triangular; es decir, si ABCD es cíclico.

Por otra parte, la inversión tampoco es eficaz para el manejo de ángulos en general, a pesar de brindar
cuadriláteros cíclicos. Por supuesto, hay excepciones a esta regla. Consideremos el siguiente problema.

Problema 17.3. (IMO 1996, P2). Sea P un punto interior al triángulo ABC tal que:

∠APB − ∠ACB = ∠APC − ∠ABC (17.4)

Sean D y E los incentros de los triángulos APB y APC, respectivamente. Demostrar que AP, BD y CE
concurren.

Demostración. Invirtamos respecto a un círculo con centro A y radio r arbitrario. Primero analicemos qué
ocurre con la incómoda condición angular. Dado que CC′ P′ P, BB′ P′ P y BB′C′C son cíclicos, podemos
derivar que:

∠B′C′ P′ = ∠CC′ P′ − ∠CC′ B′ = ∠CPA − ∠CBA = ∠APB − ∠ACB = ∠P′ B′ B − ∠C′ B′ B = ∠P′ B′C′

luego, la expresión 17.4 implica que △C′ P′ B′ es isósceles en P′ .


166 Inversión

F E
D P C

B′ C′

P′
Figura 17.10. Invirtiendo el problema 2 de la IMO 1996

Ahora, usando la fórmula de la distancia, es sencillo concluir que:

AB AP′ AP′ AC
= ′ ′= ′ ′=
BP P B PC CP

luego, de acuerdo al teorema de la bisectriz, BD y CE se intersecan en un punto F sobre AP, como


deseábamos.

17.5 Inversión negativa

Definición 17.2. Dado un círculo Γ en el plano, sea h la transformación compuesta por la reflexión
r respecto a O seguida por la inversión g respecto a Γ, de modo que h asigna a cada punto A ̸= O
otro punto A′ sobre la recta OA y a un lado distinto respecto a O, tal que la siguiente igualdad es
satisfecha:
OA · OA′ = r2
La composición h = r ◦ g recibe el nombre de inversión negativa respecto a Γ.

El inverso negativo de A puede construirse de dos maneras: considerando el inverso A′ en Γ del reflejo
A′′de A respecto a O, o bien, tomando el reflejo A′ a través de O del inverso de A respecto a Γ. En adición,
puesto que la reflexión por O no es más que una rotación de 180◦ alrededor de O, la inversión negativa
hereda las propiedades de la inversión “ordinaria”, e.g. el inverso negativo λ ′ de un círculo λ que no pasa
por O será el reflejo de su inverso λ ′′ con respecto a O:
Consideremos el siguiente ejemplo.
17.5 Inversión negativa 167

A′′ A′ O A

Figura 17.11. A y A′ son inversos negativos respecto a Γ.

Γ
λ
λ′ λ ′′

Figura 17.12. Inverso negativo de λ

Ejemplo 17.2. (IMO 2015, P3) Sea ABC un triángulo acutángulo con AB > AC. Sea Γ su circun-
círculo, H su ortocentro, y F el pie de altura desde A. Sea M el punto medio de BC. Sea Q el punto
sobre Γ tal que ∠HQA = 90◦ , y sea K el punto en Γ tal que ∠HKQ = 90◦ . Asuma que los puntos
A, B, C, K y Q son todos diferentes y están sobre Γ en este orden. Probar que los circuncírculos
de los triángulos KQH y FKM son tangentes.

Solución. Sea D el pie de altura desde C, tenemos que AH · HF = CH · HD. Es conocido que Q, H y M
están alineados y dado que ∠AQH = ∠HFM = 90◦ , entonces AQFM es cíclico y por tanto QH · HM =
AH · HF.

Consideremos la inversión negativa h centrada en H y con radio r = AH · HF. En base a las poten-
cias anteriores podemos derivar que h(A) = F, h(Q) = M y h(C) = D, así que la imagen de Γ es el círculo
de los nueve puntos de △ABC, digamos γ. Sea K ′ = h(K). Evidentemente, K ′ está sobre γ. Además, el
inverso de (KQH) es K ′ M y la imagen de (FKM) bajo h es (AK ′ Q), por consiguiente, es suficiente probar
que K ′ M es tangente a (AQK ′ ).
Sea N el punto medio de AH y notemos que QKMK ′ es cíclico. Como MN es diámetro de γ, tenemos
∠NK ′ M = 90◦ = ∠HMK ′ , por lo que basta mostrar que el centro de (AQK ′ ) está sobre NK ′ . La última
igualdad permite deducir que NK ′ ∥ MH. Ya que MH ⊥ AQ entonces K ′ N ⊥ AQ, pero N es el circuncentro
de △AQH, así que K ′ N es la mediatriz de AQ y por supuesto, se concluye que el centro de (AQK ′ ) está
sobre K ′ N, como requeríamos.
168 Inversión

N K
D

K′
B M F C

Figura 17.13. Problema 3 de la IMO 2015.



17.6 Inversión bc
Otro tipo
√ de inversión cuya aparición es cada vez más frecuente en las competiciones es la llamada
inversión bc. Establezcamos el siguiente resultado.

Proposición
√ 17.6. Sea ABC un triángulo. Llamemos por f la inversión con centro A y radio
r = AB · AC y sea g la reflexión con respecto a la bisectriz interna del ángulo ∠BAC. Definamos
la siguiente función:
h = g◦ f
entonces, se tiene que h(B) = C.

Demostración. Observemos que AB · AB′ = AB · AC = AC′ · AC, por lo que AB′ = AC y AB = AC′ . Como
△B′ AC es isósceles, se sigue que C = g(B′ ), de donde h(B) = C.

C′

B D C

B′
Figura 17.14. La composición h = g ◦ f fija a los otros dos vértices B y C.

17.6 Inversión bc 169

La proposición anterior muestra que la composición especial h deja fijos a los vértices B y C, por
tanto, es claro que el inverso del circuncírculo de △ABC es la recta BC. Es más, para cualquier recta ℓ
que pase por A, tendremos que f (ℓ) = ℓ, por lo que h(ℓ) será la reflexión de ℓ con respecto a la bisectriz
interna de ∠A; por ende, cualquier par de rectas isogonales con respecto a los lados AB y AC son imágenes
una de la otra bajo h. √
El poder de la inversión bc yace en que empareja una cantidad importante de puntos, rectas y
círculos conocidos en △ABC, como veremos en los dos grandes lemas a describir.

K
H O
N

B M C

M′

O′ N′
H′

K′

Figura 17.15. Los inversos del ortocentro, circuncentro y puntos relacionados bajo h.

Lema 17.2 (Imágenes del circuncentro, ortocentro y puntos relacionados).


√ Sea ABC un triángu-
lo escaleno y consideremos la inversión en A con radio r = AB · AC seguida de una reflexión
respecto a la bisectriz de ∠A. Luego, h intercambia:
a) El circuncentro O de △ABC con la reflexión O′ de A respecto a BC.
b) El circuncírculo de △BHC y el circuncírculo de △BOC.
c) El ortocentro H y el segundo punto de intersección H ′ de AO y (BOC).
d) El pie K de la perpendicular por H a la A-mediana con el punto de intersección K ′ de las
tangentes por B y C a (BAC).
e) El punto medio N de la A-cuerda simediana y el simétrico N ′ de A respecto al punto medio
M del lado BC.
f) El punto medio M del lado BC y segundo punto de intersección M ′ de la A-cuerda simediana
con (ABC).

En general, si deseamos demostrar que h(X) = Y , basta con asegurar que △ABX ∼ △AYC. Proce-
damos a probar a). Conocemos que AH y AO son simétricas respecto a la bisectriz interna de ∠A, i.e.
∠BAO′ = ∠BAH = ∠OAC. Además, OB = BO′ y AO = OC, por ende inferimos que △ABO′ ∼ △AOC,
de donde surge la propiedad.
170 Inversión

Como H está sobre (BO′C), el primer inciso implica b); el cual, a su vez, permite verificar a c).
Agregando, es claro que K ′ pertenece a (BOC) y es sencillo ver que K está sobre (BHC), así que b)
también muestra d). Asimismo, N está sobre (BOC) y N ′ sobre (BHC), por lo que e) es cierto. Por
último, como BC y (BAC) son inversos, junto a ∠BAM ′ = ∠MAC podemos deducir f ).
El segundo lema asocia al incentro y excentros de △ABC.

IB

M
A
IC

M′ B E D T′ C
T


D′ E

IA

Figura 17.16. Invirtiendo respecto a h al incentro, excentros y puntos relacionados.

Lema 17.3 (Incentro, excentros y puntos afines). La composición de inversión y reflexión h tam-
bién intercambia:
a) Los puntos de intersección D y D′ del rayo AI con BC y (ABC), respectivamente.
b) El incentro I y el A-excentro IA .
c) El B-excentro IB con el C-excentro IC .
d) El punto medio M del arco BACd y el pie de la bisectriz externa de ∠A, digamos M ′ .
e) El A-incírculo mixtilíneo y el A-excírculo.
f) El punto de tangencia T del A-incírculo mixtilíneo con (BAC) y el punto de contacto T ′ del
A-excírculo con BC.
g) El punto de tangencia E del incírculo con BC y el segundo punto de intersección E ′ de
(DD′ T ′ ) y (BAC).

El primer hecho es trivial. En el caso de b), c) y d) es suficiente probar que △BAI ∼ △IA AC, △IC AB ∼
△CAIB y que △M ′ AB ∼ △CAM, lo cual es meramente manipulación angular. En el caso del inciso e),
debido a que la inversión preserva tangencia, la imagen del A-excírculo debe ser tangente a AB, AC y a
(BAC), así que precisamente debe coincidir con el A-incírculo mixtilíneo. El hecho f ) se deriva inmedia-
17.7 Más ejemplos 171

tamente de e) y recordando nuevamente que BC y (BAC) son inversos bajo h.


A partir de f ), inferimos que ∠BAT = ∠T ′ AC, de donde no es difícil ver que △BT E ∼ △ATC, así
que ∠BET = ∠DD′ T = ∠ACT , de modo que EDD′ T es cíclico; por tanto, el inverso de E debe ser el
segundo punto de intersección de (DD′ T√′ ) con (BAC), como habíamos mencionado.
Por supuesto, el uso de la inversión bc será más claro si intentamos un problema.

Ejemplo 17.3. (Lista Corta IMO 2016, G7, modificado). Sea I el incentro de un triángulo no
equilátero ABC, IA el A-excentro, IA′ la reflexión de IA en BC, y lA la reflexión de la recta AIA′ en AI.
Defínanse puntos IB , IB′ y la recta lB análogamente. Sea P el punto de corte de lA y lB . Probar que
P yace en la recta OI, donde O es el circuncentro del triángulo ABC.

IB
IA′
A

I O
P
B C

IB′

A′
IA
Figura 17.17. Séptimo problema de geometría euclidiana de la lista corta de la IMO 2016.

Solución. Notemos que AC · BC = CIB ·CIA = CIB′ ·CIA′ y:

∠IA′ CA = ∠IA′ CB − ∠C = ∠BCIA − ∠C = ∠IBCA − ∠C = ∠IB′ CA − ∠C = ∠IB′ CB

de donde △BCIB′ ∼ △AIA′ C. Como ∠PBA = ∠IB′ BC y ∠PAB = ∠CAIA′ , inferimos que △BPA ∼ △BIB′ C ∼
△AIA′ C, por tanto AP · AIA′ = AB · AC.√
Invirtamos con centro A y radio AB · AC, seguido de una reflexión respecto a AI. Hemos deducido
que P e IA′ son inversos. Por los lemas 17.2 y 17.3, I se traslada en IA y O en A′ , el reflejo de A en BC; por
consiguiente, es suficiente probar que A, A′ , IA′ , IA′ son concíclicos, lo cual es obvio puesto que AA′ IA IA′
es un trapecio isósceles.

17.7 Más ejemplos


¡Resolvamos dos problemas más!
172 Inversión

Ejemplo 17.4. (Lista Corta ELMO 2013, G5). Sean ω1 y ω2 dos círculos centrados en O y P, y
supoonga que se intersecan en dos puntos X e Y tales que ∠OXP = ∠OY P = 90◦ . El diámetro AB
de ω1 es seleccionado de modo que B está estrictamente dentro de ω2 . Los dos círculos tangentes
a ω2 y que pasan por O y A tocan a ω2 en F y G. Demostrar que FGOB es cíclico.

F′
X
F
B
O
A A′ P

G′
Figura 17.18. Un par de círculos ortogonales e inversión.

Solución. Invirtamos respecto a ω1 . Por la definición 16.3 reconocemos a lo inmediato que ω1 y ω2 son
ortogonales. Según el lema 17.1, ¡esto significa que ω2 coincide con su inverso respecto a ω1 ! Luego, G′
y F ′ son los segundos puntos de corte de OF y OG con ω2 , respectivamente. Es más, las imágenes de
los círculos (AOF) y (AOG) son las rectas AF ′ y AG′ , mismas que deben ser tangentes a ω2 ; asimismo,
la recta F ′ G′ corresponde al inverso de (FOG), por lo que es suficiente probar que F ′ , B y G están
alineados.
Sea A′ el inverso de A respecto a ω2 , el cual debe estar sobre ω1 . Como F ′ G′ es la A-polar respecto a
ω2 , A′ yace en F ′ G′ y F ′ G′ ⊥ AP, pero también BA′ ⊥ AP, por tanto B está sobre F ′ G′ .

Ejemplo 17.5. (Taiwán 2017, Mock IMO, P1). Sean B, C, X, Y cuatro puntos sobre un círculo
dado. Asuma que A es el punto medio de BC, y Z es el punto medio de XY . Sean L1 y L2 las
perpendiculares a BC que pasan por B y C, respectivamente. La perpendicular a AX por X interseca
a L1 y L2 en X1 y X2 , respectivamente. Similarmente, la perpendicular a AY por Y corta a L1 y L2
en Y1 y Y2 , respectivamente. Suponga que X1Y2 corta a X2Y1 en P. Mostrar que ∠AZP = 90◦ .

A primera vista, parece contraproducente invertir debido a que contamos con solo un círculo “ex-
plícito”; sin embargo, ¡visualicemos que en realidad contamos con cinco círculos: (BY1YA), (BX1 XA),
(CY2YA), (CX2 XA) y (BCXY ), cuatro de ellos pasando por A! Desde luego, ahora tenemos suficiente
motivación para invertir con centro A y radio AB = AC.

Solución. Invirtamos respecto al círculo con diámetro BC. Las tripletas Y ′ , Y1′ , B; Y, Y2′ , C; X ′ , X2′ , C y
X ′ , X1′ , B son puntos colineales; el cuadrilátero BCX ′Y ′ es cíclico y además:

∠BY1′ A = ∠BX1′ A = ∠AY2′C = ∠AX2′ C = 90◦


17.7 Más ejemplos 173

Y2

X1 X
Z X2
Y
Y1

P
B A C
Figura 17.19. Aparentemente, un problema libre de inversión.

por consiguiente, P′ corresponderá al segundo punto de intersección de las circunferencias con diámetro
AL y AK, donde L = CY ′ ∩ BY ′ y K = BY ′ ∩CX ′ .
Por otro lado, Z ′ X ′ AY ′ debe ser cíclico. Por medio de la fórmula de la distancia, obtenemos que:

Y Z Y ′ Z ′ AX ′
1= = ′ ′· ′
ZX Z X AY
así que, de acuerdo a la proposición 15.2, Z ′ X ′ AY ′ debe ser armónico.
Evidentemente, es suficiente probar que ∠AP′ Z ′ = 90◦ . Denotemos por P′′ el pie de la perpendicular
desde A a KL.

K
X′

Z′

Y′ L
X2′
X1′ Y2′

Y1′ M
J B A C
P′
Figura 17.20. Diagrama inverso del problema 17.5.

Como P′′ es el segundo punto de intersección de (KX2′ AY1′ ) y (LY2′ AX1′ ), inferimos que P′′ = P′ , luego
K, L y P′ son colineales. Sean M = BC ∩ KL y J = X ′Y ′ ∩ BC. Como (J, M; B,C) = −1 tendremos que
JY ′ · JX ′ = JB · JC = JM · JA por el lema 15.5, por tanto M está sobre (Y ′ Z ′ X ′ A). Finalmente, deducimos
que:
M(Y ′ , X ′ ; Z ′ , A) = −1 = M(Y ′ , X ′ ; K, A)
luego, M, K, Z ′ y P′ están alineados, por ende ∠AP′ Z ′ = 90◦ .
174 Inversión

17.8 ¿Cuándo es recomendable invertir?


Como pudiste haberte dado cuenta a lo largo del capítulo, la utilidad de la inversión yace en que, bajo
las condiciones correctas, nos brinda un problema más sencillo de resolver. La habilidad de reconocer
tales escenarios se adquiere por medio de la práctica suficiente. Sería en extremo útil señalar algunas
circunstancias en las que invertir puede ser la clave del ejercicio.

La más obvia de todas: hay varias circunferencias que pasan por un mismo punto. Naturalmente,
no hay razón para creer que su construcción en el problema será explícita: pueden estar ocultos
como cuadriláteros cíclicos con un vértice común.
Contamos con un par de círculos ortogonales. Sabemos que la inversión respecto a uno de ellos
preserva al otro, lo cual es genial.
Tenemos figuras tangentes. Seleccionando un centro de inversión conveniente, la tangencia se trans-
forma en paralelismo.
Hay un puñado de potencias con un punto común (es decir, expresiones del tipo PA · PB). Si A y B
están a un mismo lado de P, considerar la inversión ordinaria puede ser el truco; en caso contrario,
toma en cuenta una inversión negativa respecto a P. Una fuente rica de potencias es la configuración
ortocentro-pies de alturas-vértices.
Problemas que involucren puntos notables como el incentro, los excentros,√el ortocentro, el cir-
cuncentro y demás puntos relacionados. Siempre ten a mano la inversión bc en estos casos, o
simplemente úsala si deseas dejar fijo a un triángulo específico.

En general, la inversión resulta muy poco útil si tienes demasiadas rectas no concurrentes (¿para
qué querer círculos?) y pocos círculos, o ángulos sin algún rayo que pase por el centro. De haber una
cantidad considerable de puntos, círculos o rectas, la inversión puede consumir tiempo precioso; ¡úsala
inteligentemente!

17.9 Problemas resueltos


Problema 17.4. (Indonesia 2018, P1) Sean Γ1 y Γ2 dos círculos tangentes en el punto A y Γ2 está en el
interior de Γ1 . Sea B un punto sobre Γ2 y AB corta a Γ1 en C. Sea D un punto sobre Γ1 y P un punto
sobre la recta CD. La recta BP interseca a Γ2 en Q. Probar que A, D, P, Q yacen sobre una misma
circunferencia.

Problema 17.5. (Sharygin 2016, Ronda Final, Grado 10, P2) Sean I e Ia el incentro y el A-excentro del
triángulo ABC, respectivamente. Sea A′ el antípoda de A en el círculo ABC y A1 el pie de altura desde A
sobre BC. Probar que ∠IA1 Ia = ∠IA′ Ia .

Problema 17.6. (Rusia 2009, Grado 10, P2) En el triángulo ABC con circuncírculo Φ, la bisectriz interna
del ángulo ∠A interseca a BC en Dy Ω por segunda vez en E. EL círculo de diámetro DE corta a Ω
nuevamente en F. Probar que AF es una simediana del triángulo ABC.

Problema 17.7. (EGMO 2013, P5) Sea Ω el circuncírculo del △ABC. El círculo ω es tangente a los
lados AC y AB, y es internamente tangente al círculo Ω en P. Una paralela a AB que corta a ABC en su
interior es tangente a ω en Q. Demostrar que ∠ACP = ∠QCB.
17.9 Problemas resueltos 175

Problema 17.8. (CGMO 2018, P2) Los puntos P y D yacen en los segmentos AB y AC del triángulo ABC
tales que DE ∥ BC. Sean O1 y O2 los circuncentros de △ABE y △ACD, respectivamente. La recta O1 O2
corta a AC en P y a AB en Q. Sea O el circuncentro de △APQ y M la intersección de la prolongación de
AO y BC. Probar que M es el punto medio de BC.
Problema 17.9. (Balcánica 2009, P2) Sea MN una recta paralela al lado BC de un triángulo ABC, con
M sobre el lado AB y N sobre el lado AC. Las rectas BN y CM se cortan en P. Los circuncírculos de los
triángulos BMP y CNP se cortan en un punto Q ̸= P. Probar que ∠BAQ = ∠CAP.
Problema 17.10. (Lista Corta ELMO 2018, G1) Sea ABC un triángulo agudo con ortocentro H, y sea
P un punto sobre el círculo de los nueve puntos de ABC. Las rectas BH, CH cortan a los lados opues-
tos AC, AB en E, F, respectivamente. Suponga que los círculos (EHP), (FHP) interseca a las rectas
CH, BH por segunda vez en Q, R, respectivamente. Mostrar que, a medida que P varía a lo largo del
círculo de los nueve puntos de ABC, la recta QR pasa por un punto fijo.
Problema 17.11. (Lista Corta IMO 2003, G4) Sean Γ1 , Γ2 , Γ3 , Γ4 círculos distintos tales que Γ1 , Γ3
son tangentes externamente en P, y Γ2 , Γ4 son tangentes externamente en el mismo punto P. Suponga
que Γ1 y Γ2 , Γ2 y Γ3 , Γ3 y Γ4 , Γ4 y Γ1 se encuentran en A, B, C, D, respectivamente, y que todos estos
puntos son diferentes de P. Probar que
AB · BC PB2
=
AD · DC PD2
Problema 17.12. (ELMO 2014, P5) Sea ABC un triángulo con circuncentro O y ortocentro H. Sean
ω1 y ω2 los circuncírculos de los triángulos BOC y BHC, respectivamente. Suponga que el círculo con
diámetro AO corta a ω1 de nuevo en M, y la recta AM corta a ω1 nuevamente en X. Similarmente, suponga
que el círculo con diámetro AH interseca a ω2 por segunda vez en N, y la recta AN corta a ω2 por segunda
vez en Y . Probar que MN ∥ XY .
Problema 17.13. En un triángulo ABC con circuncírculo Γ, la bisectriz interna de ∠A corta a BC y Γ en
A′ y D, respectivamente. Denote por M y N los puntos medios de los lados AB y AC, respectivamente. Sea
M ′ el segundo punto de intersección del círculo (DBA′ ) y DM, mientras que N ′ es el segundo punto de
intersección del círculo (DCA′ ) y DN. Sea ω el círculo con diámetro A′ D, T el punto donde A′ M ′ corta
por segunda vez a ω y Q la reflexión de D en BC. Probar que D, N ′ , T y Q son concíclicos.
Problema 17.14. Dado un cuadrilátero cíclico ABCD cuyas diagonales AC y BD se intersecan en P, sean
D y E puntos sobre los círcuncírculos de los triángulos APB y CPD tales que ∠BPD = ∠CPE y D, P, E
no están alineados. Si O es el circuncentro de ABCD, demostrar que OD = OE.
Problema 17.15. (Sharygin 2016, Ronda de Correspondencia, P12) Sea BB1 una simediana del triángulo
acutángulo no isósceles ABC. El rayo BB1 corta al circuncírculo de △ABC en L. Sean AHA , BHB y CHC
las alturas del △ABC. El rayo BBH corta al circuncírculo de △ABC en T . Probar que HA , HC , T y L son
concíclicos.
Problema 17.16. (Lista Corta IMO 2017, G3) Sea O el circuncentro del triángulo agudo escaleno ABC.
La recta OA interseca las alturas de △ABC por B y C en P y Q, respectivamente. Las alturas se cortan en
H. Probar que el circuncentro de △PQH está sobre una mediana de △ABC.
Problema 17.17. ([Link]. 2016, Selectivo IMO, P2) Sea ABC un triángulo escaleno con circuncírculo
Ω, y suponga que el incírculo de ABC toca a BC en D. La bisectriz del ∠A corta a BC y Ω en E y F. El
circuncírculo de DEF interseca al A-excírculo en S1 , S2 y a Ω en T ̸= F. Probar que AT pasa por S1 o S2 .
176 Inversión

Problema 17.18. (Sharygin 2016, Ronda final, Grado 10, P8) Sea ABC un triángulo no isósceles, con
AA1 una bisectriz interior y A2 el punto de contacto del incírculo con el lado BC. Los puntos B1 , B2 ,C1 ,C2
son definidos similarmente. Sea O e I el circuncentro e incentro de ABC. Probar que el centro radical del
los circuncírculos de los triángulos AA1 A2 , BB1 B2 , CC1C2 yace en la recta OI.

Problema 17.19. ([Link]. 2018, Selectivo IMO, P3) Sea ABC un triángulo escaleno con circuncentro
O, y sea T un punto sobre la recta BC tal que ∠TAO = 90◦ . El círculo con diámetro AT interseca al
circuncírculo de △BOC en dos puntos A1 y A2 , con OA1 < OA2 . Los puntos B1 , B2 , C1 , C2 se definen de
forma análoga.
i) Probar que los segmentos AA1 , BB1 y CC1 son concurrentes.
ii) Probar que los segmentos AA2 , BB2 y CC2 concurren en la recta de Euler del △ABC.

Problema 17.20. (IMO 2018, P6) Un cuadrilátero convexo ABCD satisface AB ·CD = BC · DA. El punto
X en el interior de ABCD es tal que ∠XAB = ∠XCD y ∠XBC = ∠XDA. Demostrar que

∠BXA + ∠DXC = 180◦

Problema 17.21. (Campeones Rumanos de Matemática 2018, P6) Fije una circunferencia Γ, una recta ℓ
tangente a Γ, y otra circunferencia Ω disjunta de ℓ tal que Γ y Ω se ubican en lados opuestos de ℓ. Las
tangentes a Γ desde un punto variable X en Ω intersecan a ℓ en los puntos Y y Z. Pruebe que, conforme
X varía sobre Ω, el circuncírculo del triángulo XY Z es tangente a dos circunferencias fijas.

También podría gustarte